Сохранен 1058
https://2ch.hk/math/res/10508.html
24 декабря Архивач восстановлен после серьёзной аварии. К сожалению, значительная часть сохранённых изображений и видео была потеряна. Подробности случившегося. Мы призываем всех неравнодушных помочь нам с восстановлением утраченного контента!

МАТЕМАТИКА ДЛЯ НАЧИНАЮЩИХ, ТРЕД 26.5: в ожидании вики

 Аноним 06/02/17 Пнд 21:17:31 #1 №10508 
250px-Euclid.jpg
Продолжение треда http://arhivach.org/thread/233955/

В этом треде мы изучаем математику, ну или начинаем это делать. Если ты школьник или студент, и тебя есть задача, то в здесь тебе помогут её решить или хотя бы скажут, в каком направлении двигаться для её решения.
Также приветствуется обсуждения самого процесса изучения и учебников/задачников.

Анально модерируемый альтернативный мертвый тред для начинающих: https://2ch.hk/math/res/9338.html
Обсуждение вузов и математического образования: https://2ch.hk/math/res/9453.html
Мемасики сюда постите: https://2ch.hk/math/res/7199.html

Архивы тредов
http://pastebin.com/kiRZGVHW
ВНИМАНИЕ! ВНИМАНИЕ!! ВНИМАНИЕ!!!

On-line LaTex. Формулы пишем в нём, а после прикрепляем картинками к посту
http://www.codecogs.com/latex/eqneditor.php
Львовский Набор и вёрстка в системе latex
http://www.mccme.ru/free-books/llang/newllang.pdf

Если ты только зашел в тред и хочешь спросить, какую книжку прочитать, то ответ, скорее всего, будет в этих списках, анон.

Список от ОП-а, бывшего тут до меня. Был составлен на протяжении 13 тредов, к ознакомлению обязателен.
http://pastebin.com/4iMjfWAf
Список от анона с dxdy. Довольно внушителен, тоже рекомендуется к прочтению. Является дополнением к списку старго ОП-а.
http://pastebin.com/YP1uaUyd
Goodbook.txt список книг с dxdy, рекомендованный тамошними обитателями.
http://pastebin.com/4FngRj6n
Литература - НМУ
https://docs.google.com/spreadsheets/d/1UWwIIAFwSwOQLK3m--LOaMOvHUivFDEz-JAnLa87i7Q/edit#gid=0
ОП-список 2. Составляйте список в реальном времени! Предлагайте в тред книги, критикуйте уже имеющиеся!
http://pastebin.com/szzZfkCM
Форчановский список, книги на английском.
http://4chan-science.wikia.com/wiki/Mathematics
Список с видеолекциями(в разработке):
http://pastebin.com/S3d7Jj6J
Качать книги тут:
http://libgen.io
А статьи тут(в разработке):
http://pastebin.com/3BfHPskz
Мемасы(в разработке):
http://pastebin.com/e38Yuj5V


СПИСКИ В РАЗРАБОТКЕ, НУЖНА ПОМОЩЬ АНОНА, ПОЖЕЛАНИЯ ОСТАВЛЯЕМ В ТРЕДЕ
Аноним 06/02/17 Пнд 21:18:01 #2 №10509 
>>10444
Том 1. Анализ.
Том 2. Дифференциальные уравнения.
Том 3. Линейная алгебра.
Том 4. Вероятность, информация, статистика.
Том 5. Функциональный анализ.
Том 6. От Диофанта до Тьюринга.
Том 7. Оптимизация.
Том 8. Теория групп.
Том 9. ТФКП.
Том 10. Перебор и эффективные алгоритмы.
Том 11. Уравнения математической физики.
Том 12. Контрпримеры и парадоксы.
Том 13. Топология.
Том 14. Теория чисел.
Том 15. Нелинейные операторы и неподвижные точки.
Том 16. Теория множеств от Кантора до Коэна.
http://rgho.st/8ytVrBpz9
Аноним 06/02/17 Пнд 21:18:25 #3 №10510 
>>10466
В. Босс - это полнейший пиздец, все эти книженции пригодны лишь для повторения материала перед экзаменом в каком-нибудь ПТУ для даунов
НО НИКАК НЕ ДЛЯ ИЗУЧЕНИЯ МАТЕМАТИКИ

Напомню также, что этот ваш Опойцев на своем канале загонял сумасшедшую ерунду про баллистическую теорию (лженауку отвергнутую больше века назад)
Очень стремный мужик. Его отношение к РАН также под вопросом.
Аноним 06/02/17 Пнд 21:39:43 #4 №10512 
>>10509
http://rgho.st/private/8ytVrBpz9/bd7c27b7cba52c55ce62bbbe6f86dac2
Аноним 06/02/17 Пнд 21:47:48 #5 №10515 
p0004.png
p0005.png
босс p0006.png
босс p0007.png
>>10510
>загонял сумасшедшую ерунду
Это норма. Ленг вон был диссидентом СПИДа, ну а про Фоменко и говорить нечего. Тем не менее, учебники они сделали хорошие.
>в каком-нибудь ПТУ для даунов
Скажи, в каком ПТУ для даунов изучают пикрелейтед? inb4 мехмат
Аноним 06/02/17 Пнд 22:38:34 #6 №10522 
Я нуб, который решил посмотреть теорему CAP. И возник такой вопрос. Почему нельзя сделать базу данных, где можно будет для каждого типа данных или как там, задавать два пункта из трех в виде свойства? Типа, надо тебе - бери CA, CP, AP и в таком духе, кароч. А само это хранить в виде свойства возле данных, для каждых данных по разному. А эти свойства будут идти с доступностью и устойчивостью к разделению, то есть PA (AP), ибо согласованность данных между узлами там не особенно и нужна да и сможет разруливать с алгоритмами.
Аноним 06/02/17 Пнд 22:52:35 #7 №10524 
>>10522
Теорема САР - это вообще не математика (с точки зрения математики, оно не является теоремой). Это недоказанная догма информатиков, ну а информатика - это вещь, гм, самобытная.
Аноним 06/02/17 Пнд 22:54:52 #8 №10525 
Возьмём некоторое число (например, 4321). Меняем число таким образом 752—>257, 4321—>1234. Отнимем от первого числа получившееся второе, то есть 4321-1234=3087. Повторяем подобную операцию с новым числом (3087-7803= - 4716), и так далее (-4716+6174=1458;...
1458-8541=-7083
-7083+3807=-3276
-3276+6723=3447
3447-7443=-3996
-3996+6993=2997
2997-7992=-4995
-4995+5994=999
999-999=0)

В итоге мы получим ноль (как в этом примере) или наткнемся на бесконечный цикл.
Так вот, как доказать, что в ходе таких преобразований с любым числом мы получим или ноль, или бесконечный цикл?
Аноним 06/02/17 Пнд 23:15:57 #9 №10527 
>>10522
"Теорема" сар - это на самом деле не совсем теорема. Конкретно по твоему вопросу: потому что разные буковки подразумевают разную реализацию. То есть грубо говоря тебе надо будет каждый тип данных хранить в отдельной базе, и постоянно держать их поднятыми, все эти разные базы. Не очееь эффективно и не нужно в большинстве случаев. Но вообще, примерно то, что ты описал, существует, просто немного в другом виде.

Алсо, вопрос вообще не про математику, да.
Аноним 06/02/17 Пнд 23:16:49 #10 №10528 
>>10525
Вместо 1234 берешь Ax1000 + Bx100 + Cx10 + D, ну ты понел.
Аноним 06/02/17 Пнд 23:21:48 #11 №10530 
>>10525
смотри с конца же.
Аноним 06/02/17 Пнд 23:26:39 #12 №10531 
>>10528
Я пробовал так, там хуйня получается
>>10530
В смысле?
Аноним 06/02/17 Пнд 23:31:29 #13 №10532 
>>10525
Обозначим твою операцию '. Числа a и -a будем называть симметричными. Если x - число из n знаков, то 10^n будем называть порядком числа x. Замкнутые шары с центром в нуле будем называть хорошими.

Лемма 1. Порядок x' не превосходит порядок x.
Лемма 2. Число x и число -x' лежат в хорошем шаре с радиусом, равным порядку x. Назовем этот шар окружающим x.
Лемма 3. Если лежащие в некотором хорошем шаре числа a и b таковы, что a > 0, b<0, то сумма a+b также лежит в этом хорошем шаре.

Пусть x - число, с которого ты начинаешь.
Рассмотрим последовательность чисел x1 = x-x', x2 = x1-x1', x3 = x2-x2', и так далее.

Рассмотрим соответствующую последовательность окружающих шаров B1, B2, B3, ... Последовательность их радиусов не возрастает.

Либо последовательность шаров является стягивающейся и тогда по теореме о вложенных отрезках имеет общую точку, а именно 0, либо она является стабилизирующейся.

Рассмотрим первый случай. Поскольку x-x' не выводит нас из целых чисел, а расстояние между x и x' становится сколь угодно малым, делаем вывод, что рано или поздно x=x' и имеем нуль.

Рассмотрим второй случай. Количество целых чисел, содержащихся в данном шаре, конечно, а количество шаров бесконечно. Это означает, что последовательность иксов зацикливается.
Аноним 06/02/17 Пнд 23:48:23 #14 №10534 
>>10510
Лекции норм, он пару тонких моментов таки проартикулировал.
Аноним 06/02/17 Пнд 23:57:54 #15 №10535 
>>10532
>Либо последовательность шаров является стягивающейся и тогда по теореме о вложенных отрезках имеет общую точку, а именно 0, либо она является стабилизирующейся.
Не факт же.
>Рассмотрим первый случай. Поскольку x-x' не выводит нас из целых чисел, а расстояние между x и x' становится сколь угодно малым, делаем вывод, что рано или поздно x=x' и имеем нуль.

Рассмотрим второй случай. Количество целых чисел, содержащихся в данном шаре, конечно, а количество шаров бесконечно. Это означает, что последовательность иксов зацикливается.
Можно пояснить?
Аноним 07/02/17 Втр 00:36:35 #16 №10537 
>>10535
Факт. xi и xi' лежат в Bi. Поэтому xi+1 тоже лежит в Bi, и Bi+1 - подмножество Bi.

>Можно пояснить?
В шаре BN лежит лишь конечное множество целых чисел, обозначим его M.
Последовательность иксов бесконечна.
Все последующие иксы целые и лежат в BN.
Значит, все иксы в последовательности скачут по точкам из M и хотя бы одна точка встречается бесконечно много раз.
Аноним 07/02/17 Втр 00:46:14 #17 №10538 
>>10537
Так они могут скакать в сторону нуля и не будут зацикливаться, а скакать будут так же в М и внутри того же шара.
Аноним 07/02/17 Втр 00:49:28 #18 №10540 
>>10537
А ебат, можно же сказать, что зацикленной точкой может быть и ноль?
Аноним 07/02/17 Втр 01:08:40 #19 №10542 
>>10538
Вот у тебя есть M = {-9, -8, -7, -6, -5, -4, -3, -2, -1, 0, 1, 2, 3, 4, 5, 6, 7, 8, 9}. Вот у тебя есть бесконечная последовательность x1, x2, x3, x4, x5, ... , где каждый xi - элемент M. Могут ли все xi быть различными?

>>10540
Там не может получиться что-то вроде 3, 3, 3, ... , потому что 3-3=0.
Аноним 07/02/17 Втр 01:19:59 #20 №10544 
>>10542
Я про то, что первый случай можно представить как 0-0=0, 0-0=0, ...

Как только доказать, что зацикленность может состоять только из двух чисел про ноль понятно? x1-x1'=x2; x2-x2'=x1. Более двух там, вроде, не может быть.
Аноним 07/02/17 Втр 01:27:53 #21 №10546 
>>10544
Это уже другая задача.
Аноним 07/02/17 Втр 01:30:24 #22 №10548 
Две команды бьют пенальти, кто первый забьет - сразу побеждает. Вероятность забить - 30%. У кого больше шансов, тот кто начинает? Как посчитать? Допустим у второй команды есть 70% шанс получить 30%-й шанс выиграть. То есть в самом начале шансы 30% у 1-й и 21% у второй выиграть в первом раунде? А как дальше считать? Продолжать умножать?
Аноним 07/02/17 Втр 03:37:03 #23 №10550 
Чета я стромозил. Очевидно что если обе команды не забьют, в следующем раунде те же шансы, 30% vs 21%. Короче в 60% побеждает 1-я.
Аноним 07/02/17 Втр 07:26:51 #24 №10552 
>>10546
Ну с нулем я же правильно сообразил?
формальная логика Аноним 07/02/17 Втр 09:00:37 #25 №10554 
Попробую ещё раз задать вопрос. Есть уравнение (1). Есть уравнение (2), являющееся частным случаем (1) -- например, задан один из коэффициентов. Есть утверждение A, делающееся относительно этих уравнений (например, относительно их решений). Всё, это всё, что дано.

(1) <= (2)

Правильно?

A(1) => A(2)

Правильно?

Т.е. я просто прошу проверить правильность двух утверждений. Меня смущает, что значки следовательно направлены в противоположные стороны.
Аноним 07/02/17 Втр 11:01:11 #26 №10559 
>>10554
Ты как-то расплывчато задачу ставишь. Если мы говорим о логике высказываний, то нужны конкретные утверждения.
> Меня смущает, что значки следовательно направлены в противоположные стороны.
<=> - такой значек тоже существует, например
Аноним 07/02/17 Втр 11:56:31 #27 №10562 
>>10554
>A(1) => A(2)
в общем случае, конечно, нет.
Аноним 07/02/17 Втр 12:15:13 #28 №10563 
>>10554
Вообще, наебашь себе таблицу истинности и всё станет ясно.
Аноним 07/02/17 Втр 15:36:30 #29 №10570 
>>10508 (OP)
В предыдущем треде, который 26, была ссылка на книжку по геометрии для даунов, с ветными ссылками в тексте и картинками отрезков разноцветных. Запостите опять её сюда а то я не успел скачать.
Аноним 07/02/17 Втр 15:43:59 #30 №10572 
>>10570
http://jemmybutton.livejournal.com/8502.html
Аноним 07/02/17 Втр 19:16:30 #31 №10581 
Что почитать по теории графов для программистишки?
Аноним 07/02/17 Втр 19:20:39 #32 №10585 
Clrs3.jpeg
>>10581
Классику.
Аноним 07/02/17 Втр 20:01:52 #33 №10601 
>>10554
Что значит знак <= между уравнением и его частным случаем?
Что может значить, например, запись x+y=2 <= x+2=2?
Даже если тебя смущает направленность импликации, что может значить запись x+y=2 => x+2=2?
Аноним 07/02/17 Втр 20:08:10 #34 №10602 
>>10581
Новиков/Хаггарти
Аноним 07/02/17 Втр 20:18:32 #35 №10604 
Как научиться доказывать полноту по Тьюрингу для чего-либо?
Аноним 07/02/17 Втр 21:23:47 #36 №10609 
Реально ли выучить математику как иностранный язык, и если да то как?
Аноним 07/02/17 Втр 22:19:00 #37 №10612 
>>10609
Общайся с носителями
Аноним 07/02/17 Втр 22:44:36 #38 №10614 
>>10510
Ты так шутишь? Попробуй найди мне хоть одного вменяемого математика котого после множеств и топологий не распидорасило
Аноним 08/02/17 Срд 00:34:25 #39 №10621 
blob
>>10614
а ну някни
Аноним 08/02/17 Срд 12:58:52 #40 №10631 
>>10604
Построить внутри этого чего-то машину тьюринга или что-то аналогичное.
Аноним 08/02/17 Срд 15:59:58 #41 №10638 
>>10621
И это все твои аргументы? Смешная картиночка?
Аноним 08/02/17 Срд 16:50:25 #42 №10643 
blob
А есть какая-то методичка по тому, как лучше хранить и представлять математику в мозгу и развивать представления? Ну, типа представлять натуральный ряд чисел как линию, но дальше сложнее и сложнее брать, например, даже ту же линейную алгебру. Есть ли такая методичка?
Аноним 08/02/17 Срд 17:09:24 #43 №10644 
>>10643
Тобы показать какой ты дурачок и какой у тебя даунский вопрос, напиши мне что ты предсталяешь под числом два? Не символ "2", а именно то что этот символ обозначает.
Аноним 08/02/17 Срд 17:24:22 #44 №10645 
>>10643
У некоторых синестетов это само собой получается. Методички нет, но ты всегда можешь создать свою систему.
>>10644
> напиши мне что ты предсталяешь под числом два? Не символ "2", а именно то что этот символ обозначает.
Скажем, мамку твою. А число 1 - Ашота. Складывая 2+1 я представляю, как Ашот сует твоей мамке, это мое представление символа "3".
Аноним 08/02/17 Срд 17:33:21 #45 №10646 
>>10614
Ты прочел Томаса в третий раз?
Аноним 08/02/17 Срд 18:11:22 #46 №10647 
Новый точечный рисунок.jpg
Я даун. Мне непонятно, как Зорич из определения множества натуральных чисел пришел к принципу мат индукции. Это, конечно, все ясно на интуитивном уровне, но строго доказать принцип я не смог.
Подмножество X множества R называется индуктивным, если вместе с каждым x принадлежащим R ему принадлежит также число x + 1
Аноним 08/02/17 Срд 18:21:31 #47 №10649 
>>10647
И еще. Есть ли у вас конфа в телеграмме?
Аноним 08/02/17 Срд 18:26:14 #48 №10650 
>>10601
классическая формальная логика не отвечает на вопрос "что значит". Это просто набор синтаксических правил, перебор символов.

В логике высказываний "x+y=2 => x+2=2 " равносильно "A=>B"
Для логики первого порядка формула составлена неверно, т.к. не определены предикаты и кванторы.

Аноним 08/02/17 Срд 19:01:47 #49 №10652 
>>10649
Она не нужна.
Аноним 08/02/17 Срд 20:27:35 #50 №10655 
Я даун-аутист, стало интересно, где вообще на практике применимо деление множеств на счетные/несчетные. В смысле, эта классификация хоть где-то применяется за пределами собственного определения и гимнастики для ума?
Аноним 08/02/17 Срд 20:39:04 #51 №10656 
>>10655
Это факт. Факты не могут быть полезными или бесполезными, они просто есть.
Аноним 08/02/17 Срд 20:48:21 #52 №10657 
>>10647
А с чем именно проблема возникла?
Аноним 08/02/17 Срд 21:24:41 #53 №10662 
444b8d7fb7[1].png
Аноны, я клинический идиот (особенно в математике), как из левого выражения получилось правое? Я думал, переносить из левой части равенства в правую можно только слагаемые.
Аноним 08/02/17 Срд 21:29:11 #54 №10663 
Я понял почему математика сложная.
Во первых, тут все описывается определениями. Это нужно для того чтобы было абсолютно, строго, по существу понятно о чем речь. НО, конкретная формулировка не сопровождается пояснениями другими словами. Грубо говоря, нужно читать пару учебников в котором авторы по разному определяют какие то понятие, что позволяет взглянуть на эти вещи под разными углами.
Во вторых, в самих определениях есть спираль эриксона. Это такая хуйня когда в сознании нужно держать несвязанные между собой понятия, которые постепенно вводятся в сознание, чтобы в самом конце склеить их все в одну кучу и понять о чем речь вообще. Преодолевается это расписыванием этих самых понятий своими словами и сверкой с оригиналом на соответствие.
Аноним 08/02/17 Срд 21:30:27 #55 №10664 
>>10662
Перенос происходит как умножение/деление/отнимание/суммирование ВСЕГО уравнения на какое то значение.
Аноним 08/02/17 Срд 21:31:00 #56 №10665 
>>10657
Я не очень понимаю, что я могу использовать как доказанное утверждение. Ну в голове строятся какие-то доказательства, но всегда кажется, что я сделал какое-то предположение-допущение относительно множества натуральных числе(или еще чего) и мое доказательство не совсем строго. "Что ты хочешь, мудак?", спросите вы Я бы хотел увидеть абсолютно строгое доказательство используя только аксиомы и теоретико-множественные свойства.
Аноним 08/02/17 Срд 21:34:25 #57 №10666 
>>10664
Хорошо, тогда на сколько было умножено левое выражение?
Аноним 08/02/17 Срд 21:35:18 #58 №10667 
>>10666
И левую и правую часть равенства поделили на левую дробь.
Аноним 08/02/17 Срд 21:40:08 #59 №10668 
>>10667
Спасибо, анон. У меня просто в понедельник пересдача финансовой математики, а я даже с базовой школьной математикой, как видишь, не так уж и дружу. Надеюсь, за оставшиеся 4 дня смогу подготовится для сдачи хотя бы на 3, больше мне не надо, преподша лютует. Коньяку с конфетами купить ей, что ли.
Аноним 09/02/17 Чтв 09:43:03 #60 №10686 
>>10655
Классы сложности вычислений, например. Тебе же не захочется высчитывать несчетное множество, да? От них, говорят, даже машины Тьюринга зависают
Аноним 09/02/17 Чтв 11:40:32 #61 №10691 
>>10663
Примерно к таким же выводам пришел ранее. Только это не исключительная черта математики, во многих интеллектуальных областях так.
Аноним 09/02/17 Чтв 13:18:21 #62 №10693 
Ребята, это что получается, наличие производной и дифференцируемость это не одно и то же? Решил по приколу почитать босса-опойцева, а тут такое откровение.

Я то думал что дифференцируемость это наличие производной, а оказывается то дифференцируемость это общее название наличия линейный частей в функциях, что в частном виде дает производную. Правильно, нет!?
Аноним 09/02/17 Чтв 15:42:02 #63 №10697 
>>10665
За абсолютно строгим доказательством тебе в https://2ch.hk/math/res/9895.html походу. Конкретно здесь тебе просто надо показать, что определение и левая часть под буквой бэ на картинке дают ощно и то же множество.
Аноним 09/02/17 Чтв 15:43:01 #64 №10698 
>>10693
>>10438

>Функция дифференцируема если имеет производную (производная это отдельное понятие) в точке и наоборот, эти понятия эквивалентны. Производную ты уже определил.

>А вот дифференцируемость функции в точке означает, наивно, что ее можно приблизить линейной функции. Т.е. взяв какую-то окрестность, ты можешь заменить функцию на ломаные линии, которые будут точно определять точки функции в такой окрестности.

>Более точно, функцию в точке x0 можно представить в виде:
>f(x0)+A(dx)+o(dx), где A - некая константа
Аноним 09/02/17 Чтв 17:42:10 #65 №10703 
>>10698
Я вообще то другой человек.
Поясни подробнее про дифференцируемость.
Аноним 09/02/17 Чтв 17:46:05 #66 №10705 
>>10703
Я на твой пост ответил отсылкой к посту другого анона. Какой еще "другой человек"? И что еще пояснить?
Аноним 09/02/17 Чтв 18:45:34 #67 №10707 
>>10705
Если дифференцируемость так определяется, то получается что векторный анализ, дифференциальная геометрия с топологией строится на подмене кривых ломаными при бесконечно малых размерах областей коколо точки.
Аноним 09/02/17 Чтв 19:49:04 #68 №10713 
>>10693
для одной переменной эти понятия эквивалентны.
Аноним 09/02/17 Чтв 20:00:59 #69 №10714 
>>10508 (OP)
Почему для комплексных чисел не определены операции больше-меньше?
Аноним 09/02/17 Чтв 20:08:05 #70 №10715 
>>10714
ну так определи, довен
Аноним 09/02/17 Чтв 21:01:01 #71 №10718 
>>10643
https://www.youtube.com/playlist?list=PLZHQObOWTQDPD3MizzM2xVFitgF8hE_ab
Аноним 09/02/17 Чтв 21:35:22 #72 №10719 
819822d6f5[1].png
Аноны, это опять >>10662-хуй, который не может в базовую математику. Поясните, пожалуйста, как именно на пике выразили rH?
Аноним 09/02/17 Чтв 22:32:24 #73 №10721 
>>10714
Ну можно по модулю сравнивать, например.
Аноним 09/02/17 Чтв 23:24:26 #74 №10723 
Selection006.png
>>10719
Аноним 10/02/17 Птн 00:27:48 #75 №10726 
>>10723
Спасибо, анон. А как степень mT перешла к соседнему множителю, превратив его в корень? Я не понимаю, пожалуйста, объясни.
Вот тебе и последствия игнорирования математики в школе, начиная класса эдак с седьмого.
Аноним 10/02/17 Птн 00:36:01 #76 №10727 
И, да, анончики, у кого-нибудь вообще был в универе курс финансовой математики? Наращивание процентов, дисконтирование, ренты-хуенты, Time value of money, все дела. В понедельник пересдача, готовлюсь, но в голове один хуй каша. Было бы охуенно, если кто-то, знакомый с темой, помог бы мне с подготовкой в том же скайпе например, был бы очень благодарен, могу даже денег на карту кинуть немного, я же хикка.
Тот же >>10723 >>10719 хуй.
Скайп dollararmdogttilda
Аноним 10/02/17 Птн 00:36:49 #77 №10728 
>>10727
Ой, то есть, вот мои сообщения, ну вы поняли.
>>10719
>>10662

Аноним 10/02/17 Птн 02:20:38 #78 №10730 
>>10726
Извлекли корень степени mT из левой и правой частей уравнения.

>>10727
> В понедельник пересдача, готовлюсь, но в голове один хуй каша.
Здесь это сочувствия не вызовет, скорее наоборот.
Аноним 10/02/17 Птн 04:00:50 #79 №10733 
Ребзя, в универе добавили новый предмет. Тензорный анализ. Нихуя не понимаю. Линейная алгебра была(обратные, союзные матрицы). Но сука, всю лекцию сегодня сидел и нихуя не понимал как даун. Подскажите литературу для того чтобы вкатиться в этот тензорный анализ.
Аноним 10/02/17 Птн 14:56:42 #80 №10741 
20170210144813.jpg
Помогите
Аноним 10/02/17 Птн 15:03:41 #81 №10743 
>>10508 (OP)
Как Эйнштейн вывел формулу E=mc^2 ?
Аноним 10/02/17 Птн 15:41:22 #82 №10745 
>>10741

Во втором равнобедренный треугольник.
Потом уравнение.
AB= 4(2)^1/2
AB^2=2AC^2
AC=((AB^2)/2)^1/2
Подставляй AB, считай.

Аноним 10/02/17 Птн 15:56:55 #83 №10746 
>>10741
В первом:
AB=(12^2+9^2)^1/2=15
sin(B)=y/AB=sin(30)=1/2
y/15=1/2
y=15/2=7,5
x=(12^2-7,5^2)^1/2=(144-56,25)^1/2=87,75^1/2
x=87,75^1/2
y=7,5
Аноним 10/02/17 Птн 16:19:41 #84 №10747 
>>10733
Мак-Коннел А.Дж. Введение в тензорный анализ с приложениями к геометрии, механике и физике
Аноним 10/02/17 Птн 16:23:32 #85 №10748 
>>10743
https://www.fourmilab.ch/etexts/einstein/E_mc2/e_mc2.pdf
Аноним 10/02/17 Птн 16:27:27 #86 №10749 
25422019-Math-geometry-background-with-formulas-Stock-Photo.jpg
Ребята с физмата, где найти полный перечень предметов, которые вы прошли за время обучения в институте, хочу сам освоить.

Чем сегодняшние программы отличаются от советской, там учились дольше и больше проходили, мне бы именно совковую версию.
Аноним 10/02/17 Птн 16:32:59 #87 №10751 
>>10748
Кроме того, спрашивали про конкретную формулу, про формулу энергии тела в движении, про формулу полной энергии, а не про формулу с гамма-фактором.

Откуда взялась E=m0c^2 тогда?
Почему энергия равна в покое именно такая формула?
Аноним 10/02/17 Птн 16:34:22 #88 №10752 
>>10751
Почему энергия в покое равна именно такой формуле?
fixed
Аноним 10/02/17 Птн 16:41:27 #89 №10753 
>>10749
Совковую ты вряд ли найдешь уже наверное, а нынешние есть на сайтах факультетов.
sage[mailto:sage] Аноним 10/02/17 Птн 18:43:02 #90 №10760 
>>10751
>Откуда взялась E=m0c^2 тогда?
Начнем с того, что нулик у массы не пишут. Масса она как бы одна. А саму формулу можно получить из формулы для релятивистской энергии движущегося тела.
Аноним 10/02/17 Птн 18:57:54 #91 №10762 
>>10760
E=m0c^2 где m0 - нулевая масса, как получить?
Аноним 10/02/17 Птн 18:58:24 #92 №10763 
>>10762
ну в смысле масса в состоянии покоя
sage[mailto:sage] Аноним 10/02/17 Птн 19:00:27 #93 №10764 
>>10763
Масса она и в покое и в движении одна. А так берешь формулу для энергии в сто и раскладываешь при малых u/c, либо считаешь кинетическую энергию, как работу, считаешь простой интеграл, она там и вылезает.
Аноним 10/02/17 Птн 22:22:38 #94 №10776 
Пацаны помогите дураку. Последний раз изучал теорвер 10 лет назад, будучи студентом. За это время мозг совсем засох. И мне не дает покоя одна задача, очень охота узнать ответ.

5 мешков с разноцветными шарами, необходимо достать черный
в 1ом : 128 шаров, 3 черных ; 3/128 ~ 0,023438
во 2ом: 40 шаров, 3 черных ; 3/40 ~ 0,075
в 3ем : 169 шаров, 9 черных ; 9/169 ~ 0,053254
в 4ом : 59 шаров, 3 черных ; 3/59 ~ 0,050847
в 5ом : 35 шаров, 3 черных ; 3/35 ~ 0,085714

У нас есть 12 независимых испытаний, каждое испытание мы рандомно берем один шар из каждого мешка.

по Бернулли за 12 попыток вероятности получаются:
p1 ~ 0,216667
p2 ~ 0,383995
p3 ~ 0,350027
p4 ~ 0,343673
p5 ~ 0,383827

Если вытягиваем черный шар из какого-либо мешка, то в последующих испытаниях уже не берем шар из этого мешка. Т.е допустим за 1-ую попытку не вытащили ни одного черного шара, за 2-ую повезло и вытащили сразу два: из 2-ого и 4-ого мешка, например, в 3-ей попытке уже берем шары только из 1,3,5 мешков и.т.д.
Т.е 32 возможных комбинаций каждую попытку. От, когда ничего не вытащили, до, когда за раз по нужному шару из каждого мешка.

Как мне посчитать вероятность того, что я достану из каждого мешка по черному шару за 12 независимых испытаний?
Аноним 11/02/17 Суб 00:15:14 #95 №10778 
>>10776
Я решу твою задачу на вероятности, если ты объянишь мне, что такое вероятности.
Аноним 11/02/17 Суб 00:31:04 #96 №10781 
Ты про это?
>по Бернулли за 12 попыток вероятности получаются:
>p1 ~ 0,216667
>p2 ~ 0,383995
>p3 ~ 0,350027
>p4 ~ 0,343673
>p5 ~ 0,383827

Я не уверен нужно ли оно тут. Я просто посчитал вероятности за 12 независимых испытаний для каждого мешка по формуле Бернулли. За одно испытание будут такие:
>3/128 ~ 0,023438
>3/40 ~ 0,075
>9/169 ~ 0,053254
>3/59 ~ 0,050847
>3/35 ~ 0,085714
Аноним 11/02/17 Суб 00:35:05 #97 №10782 
Если научишь меня вероятностям, я решу твою задачу, в таком плане.
Аноним 11/02/17 Суб 01:12:12 #98 №10783 
>>10776
Нихуя не понял.
Нужно узнать вероятность доставания черного шарика из мешка за 12 попыток или что?
Аноним 11/02/17 Суб 01:43:05 #99 №10784 
>>10783

Перед нами 5 мешков с шарами, у нас 12 независимых испытаний. Каждое испытание мы достаем шар из каждого мешка по очереди. Если мы достали черный шар из мешка, то в последующих испытаниях из этого мешка уже не берем.
Надо найти вероятность того, что после 12 испытаний у нас будет 5 черных шаров, т.е с каждого мешка мы вытащим по черному шару.
в 1ом мешке : 128 шаров, 3 черных
во 2ом: 40 шаров, 3 черных
в 3ем : 169 шаров, 9 черных
в 4ом : 59 шаров, 3 черных
в 5ом : 35 шаров, 3 черных
Аноним 11/02/17 Суб 14:10:31 #100 №10787 
>>10776
Ну так тебе же просто надо посчитать для каждого мешка в отдельности и перемножить, не?
Аноним 11/02/17 Суб 20:31:35 #101 №10800 
Мне кажется, что для отдельно взятого станка из тысяч в реале теория вероятностей не работает.
Аноним 12/02/17 Вск 00:18:31 #102 №10805 
>>10508 (OP)
Как выводятся правила для нахождения производных и интегралов.

Например, производная от числа, или x^2 итд.
Аноним 12/02/17 Вск 00:26:11 #103 №10806 
>>10800
> станка
Тебя каким макаром из заводотреда сюда занесло?
Аноним 12/02/17 Вск 09:42:27 #104 №10813 
>>10805
По определению производной же!
В школе должны были так первые производные брать. Вот допустим, от х^2. Пишешь предел при дельта х к нулю, сверху в числителе функция в точке икс ноль плюс дельта икс минус функция в точке икс ноль (приращение функции), в числителе само дельта икс (приращение аргумента). Вычисляешь правильно и видишь, откуда взялась табличная проиводная.
Аноним 12/02/17 Вск 09:53:35 #105 №10814 
saa.jpg
>>10508 (OP)
Аноны, кто знает, как получить угол в радианах, под которым лежит прямая, проходящая из А в B? Суть в том, что, стоя в точке А, я должен повернуться на определенное значение, чтобы по прямой дойти в точку B. Как высчитать?
Аноним 12/02/17 Вск 11:49:13 #106 №10816 
>>10813
А интеграл?
Аноним 12/02/17 Вск 11:53:35 #107 №10817 
>>10816
Интеграл - это обратное действие нахождению производной.
Аноним 12/02/17 Вск 11:57:26 #108 №10818 
023206765prevstill.jpg
Лучшие учебники по математике на ваш взгляд?
Аноним 12/02/17 Вск 12:14:47 #109 №10819 
элементы математики бурбаки.jpg
>>10818
Аноним 12/02/17 Вск 13:22:12 #110 №10822 
>>10818
Алгебра для аспирантуры Грийе, Пирс ассоциативные алгебры, Марголис спектр и алгебра стинрода, Гельфанд-Манин, топопология Прасолов, топология Казарян, Мацумура, Вайбель К-теория, Уэллс про теорему об индексе, Манин введение в схемы и алгебры хопфа.
Аноним 12/02/17 Вск 16:36:42 #111 №10834 
>>10822
И всё что ли? На этом вся матемака заканчивается?
Аноним 12/02/17 Вск 17:12:12 #112 №10835 
>>10814
Ну дострой прямоугольный треугольник. Координаты точек известны, стороны ты знаешь, все.
Аноним 12/02/17 Вск 18:25:10 #113 №10846 
>>10819
Все книги Бурбаки читать?
Аноним 12/02/17 Вск 18:29:31 #114 №10847 
Вопрос по производной.

Поворачиваем секущую к касательной, в конце у нас получится угол, тангенс которого и есть производная.

А тангенсы всех углов до того конечного угла, они то же производные?
Аноним 12/02/17 Вск 18:30:42 #115 №10848 
>>10834
У тебя есть какая-то другая? Знаешь или придумал область, которая никак не охватывается книгами из этого списка? Не, ну правда, мне интересно.
12/02/17 Вск 18:39:53 #116 №10850 
>>10848
>У тебя есть какая-то другая? Знаешь или придумал область, которая никак не охватывается книгами из этого списка? Не, ну правда, мне интересно.
Комбинаторика, теория Рамсея.
Аноним 12/02/17 Вск 18:40:47 #117 №10851 
14784311403040.jpg
14806206704640.jpg
>>10848
да, их полно


Аноним 12/02/17 Вск 18:57:59 #118 №10854 
>>10847
Что, никто производных не знает даже? Это же начала анализа.
Аноним 12/02/17 Вск 19:06:11 #119 №10857 
>>10854
https://www.youtube.com/watch?v=yqyzD4bfw7k
https://www.youtube.com/watch?v=qoHWa0eJHq4
https://www.youtube.com/watch?v=WfibjWbInC4
Аноним 12/02/17 Вск 19:27:47 #120 №10861 
>>10857
Я конкретный вопрос задал. А ты сам не знаешь ответа и даёшь ссылки, которые я и сам могу найти.
Аноним 12/02/17 Вск 19:36:35 #121 №10863 
Прокомментируйте данную статью с точки зрения математика.
http://magazines.russ.ru/October/2007/7/va5.html
Аноним 12/02/17 Вск 19:44:04 #122 №10865 
Если Производная - это предел, значит это бесконечное множество тангенсов углов. Всё верно?
И лежит производная в конкретном промежутке от x0 до x0+дельта x?
Аноним 12/02/17 Вск 19:45:10 #123 №10866 
>>10865
Почитай определение предела.
Аноним 12/02/17 Вск 19:49:43 #124 №10868 
>>10866
Мы говорим про производную, геометрический смысл которой tg угла.
Аноним 12/02/17 Вск 19:57:40 #125 №10870 
>>10868
Почитай определение производной.
Аноним 12/02/17 Вск 20:18:42 #126 №10873 
>>10868
> Если Производная - это предел, значит это бесконечное множество тангенсов углов.
> значит это бесконечное множество тангенсов углов
Что я сейчас прочитал?
Аноним 12/02/17 Вск 20:25:59 #127 №10874 
>>10873
бесконечное количество точек -> бесконечное количество производных -> бесконечное количество углов и тангенсов углов.
И все разные, но лежат на некотором промежутке от и до.
Аноним 12/02/17 Вск 20:36:29 #128 №10875 
>>10874
Тогда уж бесконечное множество пар (точка, тангенс угла).
Аноним 12/02/17 Вск 21:03:03 #129 №10876 
Производная.
А в той точке, где дельта x = дельта y = 0, получается в пределе 0/0.
Значит производная в предельной точке x0 равна неопределённости.
Аноним 12/02/17 Вск 21:06:13 #130 №10877 
>>10876
Ты опять выходишь на связь?
Аноним 12/02/17 Вск 21:06:21 #131 №10878 
>>10876
Ты тралишь?
sage[mailto:sage] Аноним 12/02/17 Вск 22:06:13 #132 №10879 
>>10863
Абсолютная софистика и говноедство, та же вера (но в обратную сторону). Обсуждалось на dxdy когда-то, там же про подобную хуету от Геделя.
вообще говоря, достаточно взглянуть на автора
Аноним 12/02/17 Вск 22:18:12 #133 №10881 
>>10863
Лень читать, о чём там, вкратце?
sage[mailto:sage] Аноним 12/02/17 Вск 22:36:59 #134 №10882 
>>10881
Я опираюсь на одно серьезное математическое положение. В 1930 году Курт Гедель доказал две теоремы, которые в переводе с математического языка на человеческий означают примерно следующее: любая система аксиом, достаточно богатая для того, чтобы с ее помощью можно было определить арифметику, будет либо неполна, либо противоречива. Неполная система – это такая, в которой можно сформулировать утверждение, которое средствами этой системы нельзя ни доказать, ни опровергнуть; противоречивая система – это такая система, в которой можно сформулировать утверждение, которое средствами этой системы можно и доказать, и опровергнуть. Поскольку окружающая нас природа не содержит противоречий, понятно, что любая система аксиом, описывающая природу, будет неполна. Бог же, по определению, есть конечная причина всех причин. С точки зрения математики это означает, что введение аксиомы о Боге делает всю нашу аксиоматику полной. Если есть Бог, значит, любое утверждение можно либо доказать, либо опровергнуть, ссылаясь так или иначе на Бога. Но по Геделю полная система аксиом неизбежно противоречива, то есть, если мы считаем, что Бог существует, мы вынуждены придти к выводу, что в природе возможны противоречия. А поскольку противоречий нет, иначе бы весь наш мир рассыпался от них, нам приходится придти к выводу, что существование Бога несовместимо с существованием природы, то есть Бога быть не может.

Бог — по определению — всемогущий и всеведущий творец всего сущего. Он может — хотя бы теоретически — дать ответ на любой вопрос о своём творении (либо указать на бессмысленность вопроса). Следовательно, если есть бог, то аксиоматика полна: любой вопрос получает однозначный ответ.
Аноним 12/02/17 Вск 23:20:53 #135 №10883 
>>10879
А автор кто?
Аноним 13/02/17 Пнд 00:35:06 #136 №10884 
>>10882
у меня было круче и ближе к жизни, но мне в лом писать/вспоминать
Аноним 13/02/17 Пнд 00:38:09 #137 №10885 
>>10876
Ну так что опять никто ничего не занет на самом деле, как обычно? Все только горазды языком чесать, да спорить друг с другом у кого гребешок красивше?
Аноним 13/02/17 Пнд 00:53:03 #138 №10886 
>>10876
Давай возьмем простейшую функцию, которая подходит под твое описание: у = х. Если дельта икс равно нулю, то и дельта игрек равно нулю, верно? Подходит под твое описание?

Теперь возьмем определение производной. Ф'(х) = предел при дх -> 0 от (ф(х+дх) - ф(х))/дх. Не забывая, что ф(х) = х, упрощаем: (х + дх - х)/дх = дх/дх = 1. То есть ф'(х) = предел при дх -> 0 от 1. Чему равен предел от единицы при дх стремящемся к нулю? Правильно, единице. То есть производная функции ф(х) = х есть функция г(х) = 1.

В какой точке функция г(х) = 1 "равна неопределенности"?
Аноним 13/02/17 Пнд 01:03:37 #139 №10887 
>>10886
Ты моё описание видать не понял.

дельта x стремится к нулю!
А что если достигает? Тогда мы получаем деление на ноль.
Аноним 13/02/17 Пнд 01:07:32 #140 №10888 
>>10887
Я тебе выше об этом и написал. Перечитай первый абзац. Алсо, когда убедишь себя в том, что описание выше корректно, прочитай определение предела.
Аноним 13/02/17 Пнд 01:10:17 #141 №10889 
>>10888
ты привёл пример заведомо такой, что у тебя не получается деление на 0, ну молодец чего, а теперь возьми заведомо такой, чтобы оно было.
Аноним 13/02/17 Пнд 01:37:54 #142 №10890 
>>10747
Благодарю.
Аноним 13/02/17 Пнд 03:27:16 #143 №10891 
>>10863
>внутренняя структура Троицы соответствует общеизвестному математическому объекту – вектору в трехмерной системе координат. В зависимости от выбранной системы может показаться больше одна из проекций вектора: кто поклоняется Отцу, кто общается с Сыном, на кого нисходит Дух Святой. Но во всех проявлениях присутствуют все три координаты/ипостаси. Нет никаких противоречий между математикой и догматом Троицы.

Ахинею не комментирую.
Аноним 13/02/17 Пнд 05:43:13 #144 №10893 
IMG20170213094237.jpg
>>10508 (OP)
Сап, матчан, есть функция, где j - мнимая единица, а w (0;+inf). С чего начинать?
Аноним 13/02/17 Пнд 06:38:01 #145 №10894 
IMG20170213103900.jpg
>>10893
Чего достиг, но что это дает? Как найти модуль и аргумент у дробной функции?
Аноним 13/02/17 Пнд 06:55:43 #146 №10895 
IMG20170213105700.jpg
>>10894
Правильно иду?
Аноним 13/02/17 Пнд 07:16:00 #147 №10896 
WAT.jpg
Надеюсь не ошибся тредом

Объясните тупому гуманитарию, как работает математика.

Допустим, Играю в покер.
В банке: 100
Противник ставить:100
Для уравнивания нужно:100. Можно или уравнять или сбросить.

Так вот, если считать от лица противника: 100/(100+100банк)=1/2.
1/2 - это то, как часто должна срабатывать ставка. Если он ставит 100 и выигрывает в половине случаев (ну или больше) банк 100 - то ставка оправдана.

Идем дальше. Наше слово. Нам нужно доставить 100, чтобы выиграть банк в 200.
100/(100+200)=1/3=33,3%
То есть, нам надо вколировать то что он ставит в 33,3%.

Я не понимаю что за хуета. То есть - если он всегда то должен выигрывать в 50% или больше, а если мы коллим то лишь в 33%.

Автоприбыльность бета для него - если мы сбрасываем в 50% случаев, но нам то надо только в 33% уравнивать.

Что за баг матрицы? Ваша математика не работает?

Я тупой, объясните на пальцах
Аноним 13/02/17 Пнд 11:48:16 #148 №10898 
Пацаны, книжка Громова вышла
http://biblio.mccme.ru/node/5780/shop
алсо, порылся еще на сайте, наконец перевели на руснявый кирпич Айзенбада, а в ближайшее время будет переиздание Атьи-Макдональда
Аноним 13/02/17 Пнд 11:48:23 #149 №10899 
>>10886
А dy в твоём случае y=x чему равен?
Аноним 13/02/17 Пнд 11:49:08 #150 №10900 
>>10898
Философия идёт нах.
Аноним 13/02/17 Пнд 11:54:09 #151 №10901 
>>10886
Итого, мы в тоём случае к тангенсу угла касательной даже не притронулись, а работаем с тангенсом угла секущей, но ведь производная - это тангенс угла секущей.
Аноним 13/02/17 Пнд 12:04:36 #152 №10902 
>>10885
Всем похуй на тебя, иди учи сначала, может научишься правильно вопросы задавать, на слабо он взять решил.
Аноним 13/02/17 Пнд 12:04:50 #153 №10903 
И ещё по производным.

Из рисунка можно было бы сделать вывод, что:
tg угла секущей = dy/Δx
Но чтобы получить секущую Δx стал нулём и dy=0, тогда о какой dy/Δx = 0/0 речь? Выходит что производная = неопределённости.
Аноним 13/02/17 Пнд 12:10:03 #154 №10904 
>>10903
Но чтобы получить касательную Δx стал нулём...
fixed
Аноним 13/02/17 Пнд 12:11:49 #155 №10905 
opredelenieproizvodnoismyslproizvodnoiclipimage081.jpg
Короче, всё перепишу.

И ещё по производным.

Из рисунка можно было бы сделать вывод, что:
tg угла касательной = dy/Δx.
Но чтобы получить касательную Δx стал нулём и dy=0, тогда о какой dy/Δx = 0/0 речь? Выходит что производная = неопределённости.
Аноним 13/02/17 Пнд 13:07:31 #156 №10914 
opredelenieproizvodnoismyslproizvodnoiclipimage081.jpg
Ещё по производным.

f ' (x) = tg φ = lim{Δx->0}tg α
Я и мой друг договорились о встрече. Он должен перевести свою дорогу на некоторый угол, чтобы она примкнулась к моей неподвижной: α->φ.

Он знает про формулы производной, нужно чтобы его предел совпал с тангенсом моей дороги, тогда и углы совпадут.

График фукции - не прямая, например, на y=x и секущая и касательная и сам график функции лежат на одной прямой, поэтому там мы просто говорим о секущей и подразумеваем касательную, у нас же это всё разные вещи.

Мой друг раскладывает tg α = Δy/Δx, упрощает и получает некоторую функцию f(x).

А я тем временем просто стою и жду его на своей дороге, под углом φ.

Он берёт предел lim{Δx->0}f(x) и забивает это перемещение в бортвой компьютер переводного механизма, дорога начинает поворот, но так получается, что замирает на конкретной точке, а дальше не идёт, его дорога не доходит до моей и мы не можем встретиться.

Может ли произойти подобный случай? Какая функция нам понадобится?
Аноним 13/02/17 Пнд 13:12:54 #157 №10915 
>>10914
Что у вас за транспортная система такая
Аноним 13/02/17 Пнд 14:06:20 #158 №10917 
bigstock-Ways-Of-Numbers-35114981.jpg
>>10886

Извини, но когда ты сделал:
дх/дх = 1/1 = 1

Ты на самом деле сделал это:
0/0 = 1/1 = 1

И ты такой же молодец как и 99,9% всего населения планеты, хорошо, что я не из вашего числа.
Аноним 13/02/17 Пнд 14:37:04 #159 №10919 
Так интегралы и дифференциальные уравнения это о производной функции? А я так пугался этих слов... Поздравьте меня с чудным открытием. Страх перед математикой уходит, даже удовольствие какое-то появляется, представьте себе. Спасибо за внимание.
Аноним 13/02/17 Пнд 14:38:52 #160 №10921 
>>10917
dx/dx=1 если что.
Аноним 13/02/17 Пнд 14:58:33 #161 №10922 
>>10921
У него написано "дх" я цитирую и я должен повторить. И так ему самому будет проще и быстрее найти нужное место в тексте.

если что...
Аноним 13/02/17 Пнд 15:10:33 #162 №10924 
>>10889
0/0 - это разве не деление на ноль?
Аноним 13/02/17 Пнд 15:59:18 #163 №10926 
>>10924
В реальности там деление на ноль, но он этого сам не понял, так как получил 1, когда сократил то, что нельзя было сокращать.
>>10917
Аноним 13/02/17 Пнд 16:16:14 #164 №10927 
Итого, когда мы говорим: "Функция бесконечно мала в точке", в которой она строго равна нулю, на самом деле - это ещё пол беды, и это даже забавно выглядит lim{Δx->x0}f(x)=0.

А когда мы говорим, что lim{Δx->0}Δy/Δx=A, и мы эту A каждый раз спокойно и в два счёта находим, хотя знаем, что Δx=0, лично мне уже не смешно.
Аноним 13/02/17 Пнд 16:16:41 #165 №10928 
>>10917
Тебя весь тред просит, чтобы ты почитал определение предела, ну что, почитал?
Аноним 13/02/17 Пнд 16:27:48 #166 №10929 
>>10928
Скажи своими словами, как ты понимаешь предел?

Аноним 13/02/17 Пнд 16:33:39 #167 №10930 
>>10929
Ничего я тебе не скажу, пока не напишешь в тред определение предела. Только тогда с тобой имеет какой-то смысл говорить.
Аноним 13/02/17 Пнд 16:34:50 #168 №10931 
>>10930
Ну в таком случае иди перечитай мой тред про бесконечно малые.
Аноним 13/02/17 Пнд 18:37:02 #169 №10933 
>>10926
То есть дедение на ноль там есть, так? То есть пример я привел ровно такой, какой ты хотел, верно? Так в какой точке г(х)=1 "равна неопределенности"?
Аноним 13/02/17 Пнд 18:37:49 #170 №10934 
А вообще, предлагаю репортить этого поехавшего, а то он весь тредж будет вечно засирать.
sage[mailto:sage] Аноним 13/02/17 Пнд 18:53:04 #171 №10935 
>>10847
>А тангенсы всех углов до того конечного угла, они то же производные?
Нет.
Аноним 13/02/17 Пнд 20:27:51 #172 №10938 
!!!!НЕОБХОДИМО БЫСТРО (15 ДНЕЙ) ПОДНЯТЬ МАТЕМАТИКУ С УРОВНЯ ЕБАННОГО СЕМИКЛАССНИКА ДО УРОВНЯ ПРИМЕРНО 9-КЛАССНИКА!!!!
Все с азов, но не совсем. Шоб там сокращение дробей, логарифмы, суммы, и пр, и пр.
Аноним 13/02/17 Пнд 20:37:28 #173 №10939 
>>10938
Я когда читать начал, думал, что ды за деньги предлагаешь. А так всем что-то нужно ведь.
Аноним 13/02/17 Пнд 20:51:46 #174 №10940 
>>10938
Да вся школьная математика - мелочи, за 15 дней 2 класса изи должны проходиться. Можешь читать обычный школьный учебник.
Аноним 13/02/17 Пнд 20:53:28 #175 №10941 
ф.jpg
>>10940
Спасибо, успокоил
Аноним 13/02/17 Пнд 21:23:49 #176 №10943 
И все таки, как нашли первообразные интегралов?
С производными все понятно, показали на одном примере что и как. И с интегралами что?
Понятно что их можно вывести опираясб на производные. А можно ли их вывести без них?
Аноним 13/02/17 Пнд 21:26:08 #177 №10944 
!!!!НЕОБХОДИМО БЫСТРО (15 ДНЕЙ) ПОДНЯТЬ МАТЕМАТИКУ С УРОВНЯ ЕБАННОГО СЕМИКЛАССНИКА ДО УРОВНЯ ПРИМЕРНО 9-КЛАССНИКА!!!!
Все с азов, но не совсем. Шоб там сокращение дробей, логарифмы, суммы, и пр, и пр.
Готов к обсуждению цен. Удаленно.
Аноним 13/02/17 Пнд 21:28:19 #178 №10945 
blob
> то чувство, когда забыл таблицу умножения
Аноним 13/02/17 Пнд 21:55:44 #179 №10946 
>>10941
Да не за что, главное на всякие там примеры не отвлекайся, тупо берёшь учебник и учишь.
sage[mailto:sage] Аноним 13/02/17 Пнд 21:56:22 #180 №10947 
>>10945
Нахуя ты её учил? Заняться нечем было?
Аноним 13/02/17 Пнд 22:51:29 #181 №10948 
Почему -22 = -4, а не 4? -2 х -2 = 4, минус на минус же, не? Вроде четная степень делает с минуса на плюс, а нечетная оставляет минус.
Аноним 13/02/17 Пнд 22:53:51 #182 №10949 
>>10948
(-2)^2 = 4
-(2^2) = -4
Аноним 14/02/17 Втр 00:23:11 #183 №10952 
Анончики, может есть среди вас кто-то, кто шарит в матанализе? Пусть n — натуральное число, а a и b — положительные числа. Через точку с координатами (a,b) проведена прямая, пересекающая оси абсцисс и ординат в точках u>0 и v>0 соответственно. Найдите уравнение прямой, для которого выражение u^n+v^n принимает наименьшее значение. Приведите это уравнение к виду αx+βy=1 и в ответе укажите выражение αx+βy.

Как такое решать?
Вот мои мысли:
У нас пересечения с x =(u,0) и y=(0,v), уравнение прямой y=kx+b, подставляем получаем b=v, k=-v/u, уравнение y=(-v/u)x+v.
Дальше, выражение принимает наименьшее значение, это нужно взять производную и приравнять к 0:
lnu
u^n+lnvv^n=0
u=v
(-lnv/lnu)^(1/n)
Аноним 14/02/17 Втр 00:24:01 #184 №10953 
>>10952
Или нет, я хз, по кругу хожу, жесть какая-то.
Аноним 14/02/17 Втр 00:25:42 #185 №10954 
Сколько времени понадобится для изучения алгебры и основ высшей математики, если уделять ей ~4 часа в день?
Аноним 14/02/17 Втр 01:21:55 #186 №10955 
>>10954

У тебя всего одна жизнь, никогда об этом не забывай.
Аноним 14/02/17 Втр 08:01:53 #187 №10957 
>>10861
>Но чтобы получить касательную, Δx стал нулём и dy=0
С хуя ли? Ты даже в рисунке разобраться не можешь
dy =/= 0. Если тебе интересно dy - это дифференциал функции, главная часть её приращения Δy. Дифференциал охуенен тем, что он линеен относительно приращения независимой переменной. Живо за 1ый том Фихтенгольца, если не пойдёт - за справочник по высшей математике Выгодского.
>>10914
>График фукции - не прямая, например, на y=x и секущая и касательная и сам график функции лежат на одной прямой, поэтому там мы просто говорим о секущей и подразумеваем касательную, у нас же это всё разные вещи.
Я познаю мир линейную функцию. Она этим и замечательна, что приращение независимой переменной приводит к линейному приращению зависимой переменной.
y=ax
y+Δy=a(x+Δx)=ax+aΔx
Δy=a
Δx
Увы, все остальные элементарные нелийненые функции этим замечательным своством не обладают зато обладают их дифференциалы, ради чего они и придумывались
>Он берёт предел lim{Δx->0}f(x) и забивает это перемещение в бортвой компьютер переводного механизма, дорога начинает поворот, но так получается, что замирает на конкретной точке, а дальше не идёт, его дорога не доходит до моей и мы не можем встретиться.
Ёбанное наркоманство, но геометрический смысл в том, что, да, ебанная секущая переходит в ебанную касательную
Аноним 14/02/17 Втр 09:17:51 #188 №10958 
>>10954
По ФГОС - ~7 лет.
Аноним 14/02/17 Втр 12:11:04 #189 №10960 
А до какого возраста есть смысл вкатываться в математику? Вот смотришь биографии разных ученых и там уже оказывается, до трех лет их накачивали математикой. В 4 года уже отдали в элитные ясли, где они решали диффуры. К 14 лет уже сформировался серьезный базис и они просто щелкали все эти топосы. И при всем этом кто-то из окружения обязательно рассказывал об уравнениях, формулах и так далее, читал там Пифагора/Эвклида/Эйнштейна/Лейбница.
Аноним 14/02/17 Втр 12:45:38 #190 №10961 
>>10960
Всё верно, после пяти уже поздно. Без занятий математикой до этого возраста мозг сформируется не так как нужно.
Аноним 14/02/17 Втр 13:23:31 #191 №10962 
Как найти бесконечную сумму 3n/4n ?
n=1..∞
Ясно, что будет рядом с 1/4n, где по геометрической прогрессии сумма 4/3
sage[mailto:sage] Аноним 14/02/17 Втр 13:57:40 #192 №10963 
>>10962
сумма n/4n n от 1 до бесконечности = 1/4n от 1 до бесконечности + 1/4n от 2 до бесконечности + 1/4n от 3 до бесконечности + ...
Аноним 14/02/17 Втр 14:42:03 #193 №10968 
Как нофапон влияет на занятия математикой?
Аноним 14/02/17 Втр 15:54:28 #194 №10977 
>>10949
Спас! Только пользуйся кнопками, которые абу ввел
Так непривычно без капчи, прямо год назад.
Аноним 14/02/17 Втр 16:22:42 #195 №10978 
>>10945
>учить таблицу умножения
Аноним 14/02/17 Втр 16:24:42 #196 №10979 
0,(9)=1?
статья на вики не убедительная
есть опровержения?
Аноним 14/02/17 Втр 16:29:27 #197 №10980 
>>10979
Нет. Может быть у тебя есть?
Аноним 14/02/17 Втр 16:35:19 #198 №10981 
>>10944
Зазубрить может и успеешь, а вот проникнуться не получится.
sage[mailto:sage] Аноним 14/02/17 Втр 16:41:39 #199 №10983 
>>10979
Что такое число? Подумай.
Аноним 14/02/17 Втр 16:47:27 #200 №10984 
>>10981
Да чо там проникаться, в школе вообще нихуя не делают, только дрочат примеры.
Аноним 14/02/17 Втр 17:43:34 #201 №10986 
подскажите основные темы по алгебре и геометрии за 8-9 класс плс
Аноним 14/02/17 Втр 17:44:33 #202 №10987 
>>10968
двачую вопрос
Аноним 14/02/17 Втр 17:57:41 #203 №10988 
>>10986
Оглавление любого учебника открой, мань
Аноним 14/02/17 Втр 18:03:12 #204 №10989 
>>10988
мне нужны основные темы
Аноним 14/02/17 Втр 18:09:18 #205 №10990 
>>10989
А в учебнике они какие?
Аноним 14/02/17 Втр 18:16:00 #206 №10991 
>>10990
прост там много той хуиты которая мне не пригодится - потому и спрашиваю. крч если не знаешь то ладно...
Аноним 14/02/17 Втр 19:11:41 #207 №10992 
blob
Как к этому готовиться?
Инверсия, наверное. Кубические уравнения. Что еще?
Аноним 14/02/17 Втр 19:13:49 #208 №10993 
>>10992
То есть, уравнения в 4-ой степени.
Фикс
Аноним 14/02/17 Втр 19:17:28 #209 №10994 
чел понимает, что дх стремится к нулю, но до этого берёт и [...].gif
Производные. Продолжение.

Вот смотрите, чел с матпрофи перед последним равно избавился от Δx=0, а в предыдущем действии он числитель разделил на Δx. И всё у всех нормально, все довольны и все так же делают уже сотни лет.

Человек взял да и выполнил деление (k0+0x0)/0=k0/0=k.
Вы посмотрите как всё заебись-то получается в этих ваших производных и в этих ваших пределах.

А теперь я поделю (второй предел с дробью):
+0^2=+ 0 отбрасываем
k/0=inf -бесконечность
и остаётся x0
Мы получили неопределённость вида: infx0, с которой уже ничего не сделать. И подобных примеров огромное количество.
С чем всех и поздравляю.
Аноним 14/02/17 Втр 19:30:54 #210 №10996 
>>10994
Через пол года все мировые фондовые рынки рухнут :(
Аноним 14/02/17 Втр 19:57:07 #211 №10997 
Как я уже сто раз повторял, в пределе тангенса стремящегося угла, у нас 0/0.
Производная должна появиться только там, где достигнут предел при Δx=0, а там и Δy=0.
Да есть такой тангенс dy/Δx и там не 0/0.
Но мы не можем приравнять к нему наш предел.
Мы не можем превратить наш предел неопределённости в тот определённый и конечный тангенс.

Мосты не сходятся.
Аноним 14/02/17 Втр 20:03:16 #212 №10998 
>>10952
x/(b^(n/(n+1))+a)+y/(b+ab^(1/(n+1) ))=1
Где ошибка? Бьюсь-бьюсь, уже код накатал, какие только значения не подставлял, не подходит нихера.
(b^(n/(n+1))+a)^n+(b+a
b^(1/(n+1) ))^n
Аноним 14/02/17 Втр 20:03:46 #213 №10999 
Это у тебя такой тролинг тупостью?
Аноним 14/02/17 Втр 20:04:25 #214 №11000 
>>10952
>уравнение прямой

>взять производную
Аноним 14/02/17 Втр 20:06:29 #215 №11002 
>>10994
Стремящееся это не равно, в том и прикол. Это 0.00000000000000000000000000000000000000000000000000000000000000000000000000000000000000000000000000000000000000000000000000000000000000....1.
Аноним 14/02/17 Втр 20:10:14 #216 №11003 
>>11000
Приглючилась касательная там. Тут уже нормально сделал и всё равно не то.
>>10998
Аноним 14/02/17 Втр 20:14:09 #217 №11004 
>>11002
Иди учи, что такое предел и производная.
Они у нас в точке, а не на неопределённом промежутке до нуля с бесконечностью точек.
И даже не в "бесконечной точке".
Δx=0, чётко и конкретно, а не 0,000...1.
Аноним 14/02/17 Втр 20:22:13 #218 №11005 
>>10997
Вообще, чисто теоретически мы приравниваем (даже не смотря на lim 0/0).
Мы, конретно не можем получить тот тангенс и производную из этого конкретного предела вот и всё.
Аноним 14/02/17 Втр 20:22:23 #219 №11006 
>>11004
На пикче Δx->0, а не Δx=0.
Аноним 14/02/17 Втр 20:30:52 #220 №11007 
>>11006
Ты почитал про производную? Она появляется в точке пересечения графика с касательной, т.е. при Δx=0.
А до этого (при Δx->0) секущие и производной там нет.
sage[mailto:sage] Аноним 14/02/17 Втр 20:36:27 #221 №11008 
тупой фрик.png
Почему этого долбоеба до сих пор не забанили?
Насколько же нужно быть тупым, чтобы не понять школьное определение производной
И, главное, потом еще обвинить все математической сообщество в лженаучности.
Он не хочет понимать, он хочет флеймить и притворяться тупым выблядком


Тут или клинический случай, или толстый тролль.
В обоих случаях нужен профилактический бан за яростный щипостинг. Моча - говносос
Аноним 14/02/17 Втр 20:45:14 #222 №11010 
>>11007
Если дифференциал равен нулю, то это прямая, параллельная оси Х(f(x)=const).
Аноним 14/02/17 Втр 20:46:15 #223 №11012 
>>10980
да, я раскрыл тайну, запили на тумбер высер
Аноним 14/02/17 Втр 20:46:33 #224 №11013 
>>10977
А у меня нет никаких кнопок, хм. Да и хуй с ними. Вернули 2007, ага.
Аноним 14/02/17 Втр 20:46:51 #225 №11014 
opredelenieproizvodnoismyslproizvodnoiclipimage081.jpg
>>11006
Вот смотри, когда точка L, окажется на точке K, скатываясь по синей кривой, т.е. когда Δx станет равен 0, и когда Δy станет равен 0, тогда и только тогда родится производная.

Производная равна MN/Δx=tg ф, но мы не знаем, чему равен MN. В этом и проблема.
Зато мы имеем lim{Δx->0}Δy/Δx, который равен производной.

И ты знаешь, Δx и Δy бывают разные, они не все равны между собой.
Пока секущая превращается в касательную, эти величины приобретают бесконечное количество разных величин.
У производной свои Δx и Δy, а у нас в нашем пределе - свои.

И при ближайшем рассмотрении оказывается, что Δx и Δy в lim{Δx->0}Δy/Δx оба равны нулю.
И получаем lim{Δx->0}0/0.

Но люди многие века делают так 0/0=1/1=1. А потом говорят, что это производная. Прикинь?
Аноним 14/02/17 Втр 20:47:05 #226 №11015 
>>11007
Ты опять выходишь на связь?
> Она появляется в точке пересечения графика с касательной, т.е. при Δx=0
> т.е. при Δx=0
Нет.
Аноним 14/02/17 Втр 20:50:24 #227 №11016 
>>11008
А ты его хоть зарепортил, дед-говноед? Не репорти, я последние пару дней в тред только за тем и захожу, чтобы поржать над очередным срывом покровов, лiл
Аноним 14/02/17 Втр 20:55:11 #228 №11018 
>>11014
Пойми же, не так производная определяется, нет никакого геометрического определения касательной, эта пурга про то, что производная - тангенс угла касательной, не более чем фигня для интуитивного понимания, но никак не строгое определение.
Аноним 14/02/17 Втр 20:57:48 #229 №11019 
>>11014
Если точнее, касательная определяется через производную, а не наоборот.
>>11018
Аноним 14/02/17 Втр 20:58:10 #230 №11020 
>>11008
У меня в своё время тоже разрыв шаблона вызвал данный факт 0.(9)=1, я доёбывался до всех, что раз 0.(9)=1, то 0.000000000...1=1-0.(9), в то же время 0.000000000...1/0.0000000...1=1, то есть
(1-0.(9))/(1-0.(9))=1, но 1-0.(9)=0, так как 1=0.(9), получается 0/0=1, значит 2=1+1=0/0+0/0=(0+0)/0=0/0=1. 2=1=любое другое число.
Но один чувак мне сказал, что с какого хуя 1-0.(9)=0.00000000...1? И что такая запись некорректна, как я собираюсь записать 0.0000000...1. Я сказал, что как 0.(0)1, на что он ответил, что после скобок в такой записи ничего стоять не может, ну и записать такое число можно как 0. 1-0.(9)=0, 1=0.(9). Тогда до меня наконец дошло.
Аноним 14/02/17 Втр 20:59:03 #231 №11021 
>>11014
>И при ближайшем рассмотрении оказывается, что Δx и Δy в lim{Δx->0}Δy/Δx оба равны нулю.
Ебанутый, с хуя ли Δy равен 0? Δy это отрезок LN, dy - дифференциал функции в точке K, dy - отрезок MN
>Производная равна MN/Δx=tg ф, но мы не знаем, чему равен MN. В этом и проблема.
В чём проблема? Что угол касательной ф разный в разных точках графика функции f(x)?
>И ты знаешь, Δx и Δy бывают разные, они не все равны между собой.
Нихуя не понял, что кому когда неравен.
>Пока секущая превращается в касательную, эти величины приобретают бесконечное количество разных величин.
Вот в этом и заключается предельный переход, что приращение Δx стремится к 0 (Δx->0). Касательная начинает поворачиваться, хорда KL становиться всё короче и короче, пока в пределе не слопнется в точку K.
Аноним 14/02/17 Втр 21:03:07 #232 №11022 
besk.png
>>11008
Давай с тобой вместе бесконечно малые пройдём.
Вот смотри на рисунок, там функция бесконечно мала в точках, в которых она строго равна нулю. И это всех устраивает. А меня нет.

А теперь сам логически прикинь, бесконечно малая - это процесс бесконечного приближения к нулю.
Т.е. это БЕСКОНЕЧНЫЙ процесс приближения к точке ноль, который никогда не должен кончаться.
Короче, это даже не точка.

А ТОЧКА 0 НИКОГДА НЕ БУДЕТ ДОСТИГНУТА!
Аноним 14/02/17 Втр 21:05:29 #233 №11023 
>>11015
как нет, ты что?
Аноним 14/02/17 Втр 21:09:31 #234 №11024 
>>11014
f(x)-f(a)=f`(a)*(x-a), левая часть записывается как dy правая как dx. Если у тебя dx=0, то это просто прямая, параллельная X, константа. У тебя не может быть изменения функции и dx=0 одновременно, нельзя бежать и стоять на месте одновременно.
Аноним 14/02/17 Втр 21:13:48 #235 №11025 
>>11023
Потому что Δx->0
Тебе бы подтянуть "язык ε - δ" и что такое окрестность точки.
Аноним 14/02/17 Втр 21:22:39 #236 №11026 
А ваша 0.00000000...1 "..." - бесконечность нулей.

Покажите мне эту штуку в реальности.
Этот процесс "бесконечность нулей" до конца жизни Вселенной не завершится. Вы же чисто теоретически приравниваете эти вещи туда-сюда, но не доделываете их.

Мы ведь говорим про реальность и до чистых теорий и волшебных моделей нам дела нет.

0.00000000...1 - вообще не конечная точка. Это чисто теоретическая штука.

Когда вы стремитесь к бесконечности, нет конечной точки, а если вдруг обнаружится, значит, это не бесконечность.

Теперь возьмём отрезок от 0 до 1, на котором бесконечное количество точек.
И вы начинаете перебирать точки, вы пытаетесь достигнуть 0 или 1 и ничего у вас не получается и не получится никогда, ведь их бесконечное количество. И даже если, вы знаете про 0 и 1, для вас ничего не меняется, ведь они находятся не в вашем точечном мире.
Аноним 14/02/17 Втр 21:23:16 #237 №11027 
>>11022
>БЕСКОНЕЧНЫЙ процесс приближения к точке ноль, который никогда не должен кончаться.
Возьми калькулятор и вводи туда
(0.9^2-30.9+2),(0.99^2-30.99+2),(0.999999^2-3*0.99999+2) и так далее.
>там функция бесконечно мала в точках, в которых она строго равна нулю
Зачем ты сравниваешь предел в данной точке со значением функции в данной точке? Первое это число, к которому стремится, ай да ну тебя нахер, я устал, сам разбирайся.
Аноним 14/02/17 Втр 21:26:18 #238 №11028 
>>11026
>Покажите мне эту штуку в реальности.
Хуиные молекулы в воздухе.
Аноним 14/02/17 Втр 21:26:42 #239 №11029 
>>11025
Производная равна пределу, предел достигается в тот момент когда секущая становится касательной, а в этот момент Δx=0 и всё тут, нет уже никакого стремления в данный момент времени, предел достигнут в конкретной точке.

Где в логике ошибка?
А у тебя логики даже нет.
Аноним 14/02/17 Втр 21:30:21 #240 №11031 
>>11027
Затем что мы говорим "функция бесконечно мала и тут же пишем y=0" и становится не понятно y->0 или y=0!
Мы все прекрасно знаем, что y=0, тогда не нужно и говорить, что "функция всё ещё стремится к нулю в точке ноль".
Неужели ты не согласен?
Аноним 14/02/17 Втр 21:33:01 #241 №11032 
>>11031
>Неужели ты не согласен?
Да разные вещи это, одно другому не мешает.
Аноним 14/02/17 Втр 21:33:25 #242 №11033 
>>11027
Калькулятор округляет.

Ты что хочешь, чтобы он до конца твоей жизни писал:
"Чувак, подожди, ещё чуть-чуть, ага вот ща, уже почти всё..."
Аноним 14/02/17 Втр 21:34:48 #243 №11034 
fih1.png
>>11029
>когда секущая становится касательной, а в этот момент Δx=0 и всё тут
Ага, ну давай-ка ты мне расскажешь про пределы и асимптоты обратной пропарциональности, и наконец поймёшь, что такое бесконечно приближаться, но не достигать.
>нет уже никакого стремления в данный момент времени, предел достигнут в конкретной точке.
Нет, пикрилейтед
>Где в логике ошибка?
Ошибка не в логике, а в твоём нематематичном ДНК.
Аноним 14/02/17 Втр 21:37:21 #244 №11035 
>>11032
Одно другому не мешает?

У нас функция в одной и той же точке и =0 и ->0, моей логике это мешает.
И проблема не в моей логике, а в отсутствии неё у вас.

Математики делают логические ошибки и неверно строят предложения, дают неверные определения, а в говорите, что всё хорошо.
Я же говорю: "Всё плохо!"
Аноним 14/02/17 Втр 21:38:21 #245 №11036 
>>11029
> предел достигается в тот момент
Гугли определение по Коши, ты совершенно не понимаешь сути предела. Если кратко, то там про промежуток, содержащий в себе бесконечное количество значений функции начиная с определйнного момента. Аналогия с горизонтом событий чёрной дыры мне нравится.
Аноним 14/02/17 Втр 21:39:45 #246 №11037 
>>11034
Что ты утверждаешь, что кто-то сочинял байки с логическими ошибками, чтобы "0/0=1/1=1" не вызывало подозрений?
Аноним 14/02/17 Втр 21:40:12 #247 №11038 
>>11035
>У нас функция в одной и той же точке и =0 и ->0
Функция равна, предел стремится. Это два разных объекта. Функция никуда не стремится. Предел стремится.
Аноним 14/02/17 Втр 21:41:46 #248 №11039 
>>11038
Хуйню сказал, значение функции стремится при стремлении аргумента к x0.
Аноним 14/02/17 Втр 21:42:30 #249 №11040 
>>11035
>У нас функция в одной и той же точке и =0 и ->0, моей логике это мешает.
Это хорошо. Чтобы твоя логика совсем успокоилась, повычисляй односторонние пределы 1/x при x->0 слева и справа.
>Математики делают логические ошибки и неверно строят предложения, дают неверные определения, а в говорите, что всё хорошо. Я же говорю: "Всё плохо!"
Срыв покровов итт.
>>11037
>Что ты утверждаешь, что кто-то сочинял байки с логическими ошибками, чтобы "0/0=1/1=1" не вызывало подозрений?
Ууу, да тут у нас человек, который ещё и не понимает, что такое неперырвность. И да, сочинитель "0/0=1/1=1" пока у нас только ты.
Аноним 14/02/17 Втр 21:43:31 #250 №11041 
>>11034
"бесконечно приближаться, но не достигать" - это вообще не точка. Сколько ещё повторять?
А нам конечная точка нужна.

Смотришь на картинку и не понимаешь что есть что.

Вы уже на личности перешли, а следовательно у вас закончились аргументы.
Аноним 14/02/17 Втр 21:45:50 #251 №11042 
>>11036
Предел - это значение функции =y0 в точке x->x0.
Аноним 14/02/17 Втр 21:47:57 #252 №11043 
>>11036
lim{x->x0}f(x)=A
Вот эта A - это и есть предел функции, т.е. f(x0)=A.
Я знаю, что такое предел, ты - нет.
Аноним 14/02/17 Втр 21:48:00 #253 №11044 
>>11041
>"бесконечно приближаться, но не достигать" - это вообще не точка. Сколько ещё повторять?
Ты молодец, на один шаг приблизился с истине. Теперь осталось научиться сравнивать бесконечно малые, и понять, почему при отбрасывании бесконечных малых более высокого порядка эквивалентные равенства дают точные значения. Самая мякотка анализа, без которой последний кажется вуду, которое хуй пойми как работает.
Аноним 14/02/17 Втр 21:50:31 #254 №11045 
>>11038
Мы говорим: "функция бесконечно мала", вот это место мне не нравится. Ещё сто раз повторить?
И слова "предел" там, между прочим, нет.
Аноним 14/02/17 Втр 21:52:02 #255 №11046 
>>11040
y=x Найдёшь производную строго по формуле?
Аноним 14/02/17 Втр 21:53:30 #256 №11047 
>>11044
Может примеры приведёшь?
Аноним 14/02/17 Втр 21:56:32 #257 №11048 
>>11043
>>11042
Число A называется пределом функции f (x) в точке x0, если эта функция определена в некоторой окрестности точки a за исключением, быть может, самой точки x0, и для каждого ε > 0 существует δ > 0 такое, что для всех x, удовлетворяющих условию |x – x0| < δ, x ≠ x0, выполняется неравенство |f (x) – A| < ε.
Обрати внимание на x ≠ x0
Аноним 14/02/17 Втр 21:58:40 #258 №11049 
>>11048
>x ≠ x0
f(x)=x^2-3x+2
f(x0=1)=0
limx0->1f(x)=0
Аноним 14/02/17 Втр 22:04:58 #259 №11050 
predelypokoshiclipimage016.jpg
>>11036
Аноним 14/02/17 Втр 22:07:53 #260 №11051 
>>11036
>ромежуток, содержащий в себе бесконечное количество значений функции начиная с определйнного момента
которое означает, что точки x n, начиная с некоторого номера n>N, лежат внутри интервала (a-ε, a+ε), т.е. попадают в какую угодно малую ε-окрестность точки а.

Последовательность, имеющая предел, называется сходящейся, в противном случае - расходящейся.

Понятие предел функции является обобщением понятия предел последовательности, так как предел последовательности можно рассматривать как предел функции xn = f(n) целочисленного аргумента n.
Аноним 14/02/17 Втр 22:14:24 #261 №11052 
kisel.jpg
>>11046
y=x (1) Наша функция
y+Δy=x+Δx (2) Функция, получившая приращение, после приращения независимой переменной
Выразим приращение функции
Δy = x+Δx-y (3)
Подставляем (1) в (3)
Δy = x+Δx-x = Δx (4) охуенно быть линейной функцией, уже на квадратичной функции будет не всё так красиво
По определению производной, подставив (4)
lim(Δx->0)Δy/Δx=Δx/Δx=1
Как бы предел не стремился к 0, он всегда равен 1.
arctan 1 = π/4 rad = 45º
45º совпадение? Не думаю.
Домашнее задание: вычислить производную x^2
Аноним 14/02/17 Втр 22:15:02 #262 №11053 
>>11048
Ты понимаешь, что x->x0, при их не равности это не точка вообще, а бесконечное стремление к x0?
И нам в предел просто нечего подставить.
А предел функции равен y0, я же уже писал про это всё не так давно. Неточности в определениях предела, вот и всё.
Аноним 14/02/17 Втр 22:16:53 #263 №11054 
>>11053
> это не точка вообще, а бесконечное стремление к x0?
>для каждого ε > 0 существует δ > 0 такое, что для всех x, удовлетворяющих условию |x – x0| < δ, x ≠ x0, выполняется неравенство |f (x) – A| < ε.
Да ладно?!1 Пиздиш11 Быть не может!1
Аноним 14/02/17 Втр 22:18:44 #264 №11056 
>>10968
Положительно.
Аноним 14/02/17 Втр 22:19:05 #265 №11057 
>>11052
>Δx/Δx=1
0/0=1/1=1
Аноним 14/02/17 Втр 22:19:59 #266 №11058 
>>11054
Просто у тебя не хватает фантазии, чтобы ворочать по-настоящему бесконечно большими и видеть бесконечно малые величины.
Аноним 14/02/17 Втр 22:20:23 #267 №11059 
>>11027
Доказательство теорем с помощью калькулятора. 2ch.hk/math.
Аноним 14/02/17 Втр 22:21:53 #268 №11060 
148356056139.png
/math/ - здесь троллят тупостью.
Аноним 14/02/17 Втр 22:23:40 #269 №11061 
>>11057
Ты опять начинаешь, ебучий шакал. Когда до твоей, побоюсь этого слова, головы дойдёт, что бесконечно малая величина Δx->0 это не 0.
Аноним 14/02/17 Втр 22:24:10 #270 №11062 
cached-thumb-img.14160.0.405595690239604.jpg
>>11060
/math/ - здесь тупят даже тролли.
Аноним 14/02/17 Втр 22:24:18 #271 №11063 
Вы что, не умеете округлять?
Аноним 14/02/17 Втр 22:24:59 #272 №11064 
>>11063
Вы ещё не округляете? Тогда мы идём к вам!
Аноним 14/02/17 Втр 22:25:44 #273 №11065 
Если x никогда не достигает x0, тогда Δx никогда не достигает нуля и тогда секущая никогда не станет касательной и тогда мы даже не имеем права приравнять предел тангенса угла секущей к производной!
Аноним 14/02/17 Втр 22:26:47 #274 №11066 
/math
Там, где секущая никогда не станет касательной.
Аноним 14/02/17 Втр 22:27:42 #275 №11067 
Натуральные и отрицательные числа, планиметрия и тригонометрия это все, что нужно. Остальное ненужно.
Аноним 14/02/17 Втр 22:28:19 #276 №11068 
>>11067
Евклид, иди нахуй
Аноним 14/02/17 Втр 22:28:54 #277 №11069 
>>11067
Отрицательные числа - выдумка дьявола. Они тоже ненужны. Отрицательное число - это отсутствие числа. Как у отсутствия может быть количество?
Аноним 14/02/17 Втр 22:29:26 #278 №11070 
>>11069
Катющик?
Аноним 14/02/17 Втр 22:29:42 #279 №11071 
>>11067
>Натуральные и отрицательные числа, планиметрия и тригонометрия это все, что я осилил. Остальное неосилил.
Аноним 14/02/17 Втр 22:29:43 #280 №11072 
>>11061
Так ты мне в реале покажи твою "бесконечно малую" конечную точку.
А то ты может бесконечность на бесконечность делишь.
Бесконечно малая не равна нулю, она его не достигает.
А обязана достигнуть, ведь Δx=0 в производной, иначе секущая не касательная и наш предел производной не равен.
Аноним 14/02/17 Втр 22:30:36 #281 №11073 
>>11069
Да, нуль от дьявола.
Аноним 14/02/17 Втр 22:31:06 #282 №11074 
>>11071
Как он тригонометрию осилит без дробей-то?
Аноним 14/02/17 Втр 22:31:57 #283 №11075 
>>11072
Пойдем выйдем, покажу тебе бесконечно малую. Завтра на Петровско-Разумовской, я буду в футболке с портретом Дедекинда.
Аноним 14/02/17 Втр 22:33:45 #284 №11077 
>>11072
Бесконечность деленная на бесконечность это Единица. Но Бог любит Тройцу. Бесконечность происк сатаны. Бог Един.
Аноним 14/02/17 Втр 22:36:05 #285 №11079 
Как стать шизофреником изучая математику?
Аноним 14/02/17 Втр 22:36:10 #286 №11080 
Я когда дрючил бесконечно малую функцию в точке, где она строго равна нулю, я всё это уже объяснял, что в пределе нельзя ставить знак "=".
Ведь точка x0 не достигнута, а мы ставим =y0, что это за фуфло?

И потом начинаются споры, что функция даже существовать не обязана в точке предела, а как она не обязана там существовать, если мы написали f(x)=y0, совсем поехавшие что ли?
Аноним 14/02/17 Втр 22:36:22 #287 №11081 
>>11072
>Так ты мне в реале покажи твою "бесконечно малую" конечную точку.
В трусы себе загляни иди представляй расстояние между осью абцисс и веткой гиперболы в 1ой четверти. Там она долго пытается коснуться до оси
Аноним 14/02/17 Втр 22:37:21 #288 №11082 
>>11080
>Ведь точка x0 не достигнута, а мы ставим =y0, что это за фуфло?
Вот тут у Фихтенгольца всё чётко, почему эквивалентные выражения дают точные значения. Но тебе же плевать, и читать ты его не хочешь.
Аноним 14/02/17 Втр 22:37:21 #289 №11083 
>>11077
А если это две разные бесконечности? Всё сразу облом.
Аноним 14/02/17 Втр 22:37:56 #290 №11084 
>>11083
У дьявола много обличий.
Аноним 14/02/17 Втр 22:38:11 #291 №11085 
>>11081
Потому что так сказал "Великий" математик-классик?
Аноним 14/02/17 Втр 22:39:25 #292 №11086 
>>11070
А ты гусь или пуля?
Аноним 14/02/17 Втр 22:40:18 #293 №11087 
>>11085
Да, потому что в действительных числах операция деления на 0 не разрешена, т.к. приводит к противоречиям.
Аноним 14/02/17 Втр 22:40:39 #294 №11088 
14856932423260.png
Аноним 14/02/17 Втр 22:41:01 #295 №11089 
/math - карательная педагогика.
Аноним 14/02/17 Втр 22:42:00 #296 №11090 
Ну так что, всё облом?
Секущая никогда не становится касательной и мы не приравниваем наш предел к производной.
Ведь, если приравняем там нас ждёт 0/0.
Аноним 14/02/17 Втр 22:42:35 #297 №11091 
>>11087
А в реальности ты на ноль поделить просто не сможешь, а на бесконечность не успеешь :)
Аноним 14/02/17 Втр 22:44:02 #298 №11092 
Пределы, производная и интеграл - это математический сюрреализм.
Аноним 14/02/17 Втр 22:44:13 #299 №11093 
>>11090
Запости в тред определение предела, пожалуйста.
Аноним 14/02/17 Втр 22:44:30 #300 №11094 
>>11091
>А в реальности ты на ноль поделить просто не сможешь, а на бесконечность не успеешь :)
Но в пределе-то я могу увидеть, к чему всё стремится. Ахиллес и черепаха, увы.
Аноним 14/02/17 Втр 22:44:50 #301 №11095 
>>11092
А гамалогии тогда что, профессор?
Аноним 14/02/17 Втр 22:45:36 #302 №11096 
>>11094
фуфло ваши пределы, сами не видите?
Аноним 14/02/17 Втр 22:47:53 #303 №11097 
>>11096
Мы сейчас начнём тебя кормить шутками про твою фуфлыжную мамку. Если ты не понимаешь разницы между 0 и бесконечно малой величиной, то это твои проблемы а здесь могла быть шутка про твою мамку.
Аноним 14/02/17 Втр 22:48:01 #304 №11098 
Как я уже говорил, в производной: "Мосты не сводятся".
Аноним 14/02/17 Втр 22:48:18 #305 №11099 
>>11096
Пределы используются при расчете маневров ракет Спейс Икс. Это что, получается Илон Маск нас всех наебал? Илон Маск - фуфло?
Аноним 14/02/17 Втр 22:52:49 #306 №11100 
А тот, кто написал то, что 0,(9)=1, не понимает, где реальный мир, а где его маленькая гипотетическая модель, которая существует только у него в голове.
Аноним 14/02/17 Втр 22:53:13 #307 №11101 
Сука, лолирую с треда.
Веселит даже не дурачок с пределами, а тем что ему пытаются по кругу под бурбаки пояснить где он не прав.
Аноним 14/02/17 Втр 22:53:26 #308 №11102 
>>11099
А я всё гадал, почему там столько падений и неудач.
Аноним 14/02/17 Втр 22:53:58 #309 №11103 
>>11101
Так он про производную говорит и там он прав.
Аноним 14/02/17 Втр 22:54:41 #310 №11104 
>>11060
>>11062
А вот тут просто перешел на беззвучный смех. потому что воздух в легких кончился, а приступ гомерического хохота нет.
Аноним 14/02/17 Втр 22:54:45 #311 №11105 
>>11101
/math - конструктивные дискуссии.
Аноним 14/02/17 Втр 22:55:16 #312 №11106 
>>11103
Семен семенович, чему равен наклон функции у=х?
Аноним 14/02/17 Втр 22:55:58 #313 №11107 
>>11102
Но ты же говоришь, что предел вообще фуфло. То есть у них ни одна ракета не должна была взлететь с такими фуфлыжными пределами. А взлетела таки, а не одна. Как так? Совпадение?
Аноним 14/02/17 Втр 22:56:38 #314 №11108 
>>11105
Прекрати, я щас тянучку свою разбужу и придется ей пояснять что такое предел, касательная. А есля я начну ваш тред у себя в квартире делать я просто кончюсь от смеха.
Аноним 14/02/17 Втр 22:57:04 #315 №11109 
DETAILPICTURE588787.jpg
>>11106
Он равен наклону твоей мамки, когда я к ней в гости прихожу.
Аноним 14/02/17 Втр 22:58:31 #316 №11110 
задорнов американы псаки.webm
>>11109
Аноним 14/02/17 Втр 22:59:49 #317 №11111 
>>11108
А твоя тянучка бесконечно малая или бесконечно большая? Приращение, гхм, у нее линейное или не очень? Ей больше по душе касательные или секущие?
Аноним 14/02/17 Втр 23:00:16 #318 №11112 
>>10952
>>10998
Ну что, поможете разобраться? Там y=-u/vx+v,
точка (a,b), b=-u/v
a+v, v=bu/(u-a)
В итоге вот такое выражение получается:
u^n+(bu/(u-a))^n
Минимум там равен u+v, кроме случая u,v<1, тогда 0.
Дальше я лютой хернёй страдал, подумал, что нужно найти минимальное значение вот этой функции.
u^n+(bu/(u-a))^n,
Взял производную, приравнял к 0, получил
(u-a)^(n+1)=b^n
u=b^(n/(n+1))+a
Ну и в итоге вышло
(b^(n/(n+1))+a)^n+(b+ab^(1/(n+1) ))^n
Уравнение прямой
x/(b^(n/(n+1))+a)+y/(b+ab^(1/(n+1) ))=1
На тестах по всякому прогонял, хорошо проходит, при изменении u+-0.1 значение увеличивается, вроде как минимум, но этот ответ неверный. Подозреваю, что , а хз если честно, не знаю что не так, может где-то ошибка в рассуждениях?
Аноним 14/02/17 Втр 23:01:26 #319 №11113 
>>11111
Бесконечно поджарая. И приращение у меня от нее очень линейное. И ей больше по душе мои слегка секущие руки.
Аноним 14/02/17 Втр 23:01:59 #320 №11114 
>>11112
Трудно было бы найти более неподходящее время для вопроса по теме треда.
Аноним 14/02/17 Втр 23:02:21 #321 №11115 
>>11113
Фу.
Аноним 14/02/17 Втр 23:03:11 #322 №11116 
opredelenieproizvodnoismyslproizvodnoiclipimage081.jpg
>>11106
Если секущая стала касательной, тогда там Δx/Δx=0/0.
Если Δx бесконечно мала, но не 0, тогда, может быть, Δx/Δx=1. Но тогда мы нашли не произвдоную!
Ведь если Δx бесконечно мала, но не 0, тогда точка L не дошла до точки K, секущая не стала производной и мы нашли, даже не что-то близкое к производной, а неведомую хуйню. Мы нашли аналог производной у секущей.
У одной секущей из бесконечного количества.

Я это уже в хрен знает какой раз всё повторяю.
У вас логика хотя бы на уровне школы работает?
Аноним 14/02/17 Втр 23:03:25 #323 №11117 
>>11114
Ну я тут уже давно сижу, просто подумал, что вы оживились так, вдруг кому-то не внапряг вникнуть.
Аноним 14/02/17 Втр 23:03:29 #324 №11118 
>>11112
>Пусть n — натуральное число, а a и b — положительные числа. Через точку с координатами (a,b) проведена прямая, пересекающая оси абсцисс и ординат в точках u>0 и v>0 соответственно. Найдите уравнение прямой, для которого выражение u^n+v^n принимает наименьшее значение. Приведите это уравнение к виду αx+βy=1 и в ответе укажите выражение αx+βy.

Слишком много переменных.
Может тебе нужно найти инструкцию как найти эти самые наименьшие когда все числа выбраны наугад?
Аноним 14/02/17 Втр 23:03:45 #325 №11119 
>>11107
Опытным путём.
Аноним 14/02/17 Втр 23:04:41 #326 №11120 
>>11116
PREKRATI я же реально могу подавиться слюнями.
Аноним 14/02/17 Втр 23:05:45 #327 №11121 
>>11120
ну тогда открой книжку свою на геометрическом смысле производной и сам подумай
Аноним 14/02/17 Втр 23:05:49 #328 №11122 
>>11119
Ну то есть сидели, в случайном порядке лепили друг к другу листы титана и алюминия, и вдруг оно как-то один раз взяло и полетело, да?
Аноним 14/02/17 Втр 23:06:37 #329 №11123 
/math - здесь открывают книжку.
Аноним 14/02/17 Втр 23:07:25 #330 №11124 
>>11121

УМИРАББ БЛЯДБ, СУКА, ПОЙДУ СПАТЬ ЛУЧШЕ
Аноним 14/02/17 Втр 23:07:36 #331 №11125 
>>11118
>когда все числа выбраны наугад?
Там не совсем наугад, наугад только a и b, u и v точки пересечения с осями, можно уравнение прямой отсюда достать:
y=-(v/u)*x+v, туда подставить a,b и выразить v через u. Но это я так думаю,а там хз, уже третий день парюсь.
Аноним 14/02/17 Втр 23:08:51 #332 №11126 
мама даша исследовать.jpg
>>11123
СУКАААААААААА
Аноним 14/02/17 Втр 23:15:26 #333 №11127 
Пределы.

Вот смотрите:
Высыпали плотно набитую яблоками коробку.
10 яблок - предел количества яблок в коробке.
lim{x->10}f(x)
Предел говорит, что x не должен равняться 10.
Значит мы должны написать x->11.
Но максимальный x=10, у нас больше нет яблок и x->10.
И получается так, что при x->10, мы только 9 яблок обратно положим.
Но мы шлём предел на хуй и складываем обратно все 10 яблок.
Аноним 14/02/17 Втр 23:15:48 #334 №11128 
>>11112
Сначала реши, при каких значениях u, v твоя u^n + v^n минимальна.
Аноним 14/02/17 Втр 23:17:27 #335 №11129 
>>11125
Ну и да, по такой логике вроде бы как действительно наименьшее значение получается при разных а, б и n. В плане скриптом тестил, долго не мог понять, что при а,b<1 вообще минимум должен быть, случайно в очередной раз на тестах заметил, потом для других а и б нашёл, ну а потом уже аналитически выразил у подставил и сошлось. Причём при малом изменении ощутимо меняется, правдоподобно вроде. Я вот пока писал подумал, может быть там n тоже задано? Нужно для конкретных a,b и n, а не только а и б?
Аноним 14/02/17 Втр 23:18:33 #336 №11130 
>>11128
u=b^(n/(n+1))+a
v=bu/(u-a)=b+ab^(1/(n+1) )
Аноним 14/02/17 Втр 23:21:11 #337 №11131 
lim.png
>>11116
>Если Δx бесконечно мала, но не 0, тогда, может быть, Δx/Δx=1. Но тогда мы нашли не произвдоную!
>Производная это предел
>Мам, но мы тогда не нашли производную
>Ведь если Δx бесконечно мала, но не 0, тогда точка L не дошла до точки K
Но бесконечно приблизилась
>секущая не стала производной
Наркоман
>и мы нашли, даже не что-то близкое к производной, а неведомую хуйню.
Наркоман, который ссыт против ветра и не хочет усвоить определение производной
>Мы нашли аналог производной у секущей
Дабл наркоман
>У одной секущей из бесконечного количества.
Трипл наркоман
https://www.youtube.com/watch?v=MwxjYFqP35A
Аноним 14/02/17 Втр 23:22:55 #338 №11132 
чел понимает, что дх стремится к нулю, но до этого берёт и [...].gif
Ещё раз возьмём этот пример.

Если Δx не равен 0.
Мы делим бесконечый процесс на бесконечный процесс, а не два конечных количества и якобы получаем 1. Ладно, проехали.

Перед последним "равно" он выбрасывает Δx, выходит, что дальше он должен ставить "примерно равно", а у него стоит "равно".

Здесь производная найдена в любом случае не верно.
Аноним 14/02/17 Втр 23:23:25 #339 №11133 
>>11127
>10 яблок - предел количества яблок в коробке.
>Натуралист не знает, что множество целых чисел не обладает непрерывностью
Аноним 14/02/17 Втр 23:24:16 #340 №11134 
>>11129
То есть я понял как есть множество функций u^n+v^n, где u,v зависят от a,b. Среди всех таких функций выбрать ту, чьё значение минимально. А если n тоже задан, то хз как делать опять таки, ведь тогда зависит от u,v>1 или меньше. Но вряд ли, тогда тупо u+v ответ либо u^n+b^n.
Аноним 14/02/17 Втр 23:31:04 #341 №11135 
>>11131
Нет, она не приблизилась, она приближается даже сейчас и конца и края не видно.

Секущая не стала касательной, я уже напрочь уебался спорить и очень устал.

Производная = тангенсу угла, к которому бесконечно стемится тангенс другого угла, но никогда его не достигает.
Что-то не так разве?
Ну тогда мы не можем и знак "=" поставить.

"Аналог производной у секущей" - в геометрическом смысле Δy/Δx, не равные 0/0.

Между секущей и касательной бесконечное количество секущих.
Аноним 14/02/17 Втр 23:31:36 #342 №11136 
>>11133
Давай без блабла, предел есть предел и он должен работать, а он не сработал.
Аноним 14/02/17 Втр 23:33:08 #343 №11137 
>>11133
А если мы расчитываем зубчатые передачи с конечным количеством зубьев, которое принадлежит к натуральным числам.
Аноним 14/02/17 Втр 23:37:04 #344 №11138 
>>11125
Ну смотри
>Пусть n — натуральное число, а a и b — положительные числа. Через точку с координатами (a,b) проведена прямая, пересекающая оси абсцисс и ординат в точках u>0 и v>0 соответственно. Найдите уравнение прямой, для которого выражение u^n+v^n принимает наименьшее значение. Приведите это уравнение к виду αx+βy=1 и в ответе укажите выражение αx+βy.
тот факт что
>пересекающая оси абсцисс и ординат в точках u>0 и v>0 соответственно
говорит о том что наклон прямой выражается отрицательным числом, кривая растет справа налево, иначе она бы проходила по одному отрицательному числу.
Уже имеем какую то ясность что у=-(наклон)х+с, плюс то что точки (а,б) находятся исключительно в первой четверти координатной оси.
Дальше
>Найдите уравнение прямой, для которого выражение u^n+v^n принимает наименьшее значение.
Наименьшее значение возможно только если прямая проходит на расстоянии меньше единицы к оригину/началу координат любое число меньше нуля возведенное в квадрат даст еще более меньшее число в идеале бесконечно/неопределимо близко к этому самому оригину/началу координат.
У нас же заданна конкретная точка (а,б)
Вспоминаем уравнение прямой проходящей через заданную точку, y-б = -(наклон)(x-а) в нашем случае.

Дальше нужно искать
>оси абсцисс и ординат в точках u>0 и v>0 соответственно
Как очевидно всем кто читал калькулус томаса и стюарта три раза, два раза дифференциальное и интегральное исчисление лузина, вышмат зельдовича, анализ и знакомство с вышкой понтрягина, тригонометрию и алгебру гельфанда-шеня-львовского-тоома, планиметрию и стереометрию понарина, точки пересечения
>
>
>
>
>
>
Аноним 14/02/17 Втр 23:39:40 #345 №11139 
>>11130
Ты меня, кажется, не понял. Забей пока на эн, пусть будет равно единице. У тебя есть точка, через которую тебе нужно провести под каким-то углом прямую так, чтобы у получившегося треугольничка сумма двух сторон на осях была минимальна, так? Составь функцию, которая выражает сумму этих двух сторон. То есть у тебя есть какая-то общая переменная, от которой зависят обе стороны, и тебе нужна их сумма: у = первая(т) + вторая(т).
Аноним 14/02/17 Втр 23:40:20 #346 №11140 
>>11138
БЛЯЯЯЯЯТЬ
Поумничать не выйдет уже.
Ищешь координаты пересечения прямой с осями, у тебя будут две точки (а+дельта,0) и (0,б+дельта), и уже находишь уравнение прямой проходящей через эти две точки.
Потом смотришь на формулу нахождения этих двух точек и на формулу прямой через эти две точки.
Аноним 14/02/17 Втр 23:44:46 #347 №11141 
>>11140
Короче, прозреваю что лучший результат будет когда наклон прямой будет -1, расстояния от точек пересечения до начала координат будет равной, а финальное уравнение будет в виде (пересечение с осью Х)х+(пересечение с осью Х)у=1
Аноним 14/02/17 Втр 23:47:57 #348 №11142 
>>11141
Почему? На примере, расстояние от точки по иксам 99 по игрекам 1. 99квадрат равен 9801+1=9802. Если расстояние по игреку и иксам 50 то 50квадрат + 50квадрат=5000 что меньше 9802.
Можешь проверить сам дальше.
Аноним 14/02/17 Втр 23:49:41 #349 №11143 
>>11140
>уравнение прямой проходящей через эти две точки.
Не понял, про какие дельты ты говоришь(u-a,v-b)??, но вот уравнение
y=(-v/u)*x+v
Насчёт суммы двух сторон, то есть мы не рассматриваем случай с u,v>1? Ну да, есть точка a,b и u,v могут быть любыми по идее. Или нет? Ведь u<a, что если a будет больше 1?
Аноним 14/02/17 Втр 23:52:06 #350 №11144 
>>11141
>финальное уравнение будет в виде (пересечение с осью Х)х+(пересечение с осью Х)у=1
У меня получилось x/пересечениеx+y/пересечениеy, ну и пересечения выражены через а,б, может быть тут ошибка?
Аноним 14/02/17 Втр 23:53:09 #351 №11145 
>>11143
>Ведь u<a
Вернее a<u, u пересечение с Х, a<u.
Аноним 14/02/17 Втр 23:54:02 #352 №11146 
equ.png
>>11135
>Нет, она не приблизилась, она приближается даже сейчас и конца и края не видно.
И никогда не приблизиться, спасибо >>11053
>Секущая не стала касательной, я уже напрочь уебался спорить и очень устал.
Но она будет отличаться от неё на пренебрежимо малую величину, порядок малости которой больше Δx->0
>Производная = тангенсу угла, к которому бесконечно стемится тангенс другого угла, но никогда его не достигает.
Тоже самое
>Ну тогда мы не можем и знак "=" поставить.
Именно, поэтому говорят про эквивалентность
>"Аналог производной у секущей" - в геометрическом смысле Δy/Δx, не равные 0/0.
Наркоманская хуйня, я тебя опять не понял.
>Между секущей и касательной бесконечное количество секущих.
Абсолютно верно. Просто с какого момента, когда Δx->0 стал бесконечно мал, эти секущие отличаются на бесконечно малые величины высшего порядка, которыми можно пренебречь, и более того, в результате получить точное значение? Магия? Ага. Только тебе никто не обещал, что анализ будет простым, если ты хочешь почувствовать, а не надрочить себя в быстром вычислении производным и взятии интегралов.
>>11136
>Давай без блабла, предел есть предел и он должен работать, а он не сработал.
Предел lim{x->10}f(x) что это такая величина, что перечитай >>11051 >>11048 и пик >>11034 Не всосал? Это твои проблемы
>>11137
И какие ты там производные считать собрался? Уж на то пошло, у зубчатых передач есть такой параметр как "коэффициент перекрытия", характеризующий плавность непрерывность, неразрывность передачи. Если он меньше 1, то одна пара зубьев выходит из зацепления, когда следующая ещё не успела войти. Да, зубчатые передачи в этом отношении говно, против той же ременной.
Аноним 14/02/17 Втр 23:55:21 #353 №11147 
>>11143
Дельта это приращение. Очевидно что если точка лежит в первой четверти и через нее проходит прямая то точки пересечения лежат хоть немного в сторону плюс бесконечности от проекций самих точек на оси.
Как это не рассматриваем? У нас рандомные координаты точки, там могут быть мильены с мульярдами, а могут бесконечно малые.
Плюс я тебе уже написал готовый конечный ответ, найди просто алгоритм в буквах по которому можно выразить координаты пересечения прямой с осями.
Аноним 14/02/17 Втр 23:55:47 #354 №11148 
>>11146
>спасибо >>11053
спасибо >>11048
fix
Аноним 14/02/17 Втр 23:56:21 #355 №11149 
>>11144
Деление это умножение на 1/число, если ты не знал.
Давай уже завтра а то сейчас спать уже пора.
Аноним 15/02/17 Срд 00:01:50 #356 №11150 
>>11139
>сумма двух сторон на осях была минимальна
min b*u/(u-a)+u =sqrt(b)+a
Вот для n=1, но это же и есть решение уравнения, только там в общем случае, если подставить 1, то получим
u=b^(n/(n+1))+a, то что выше писал
Аноним 15/02/17 Срд 00:03:22 #357 №11151 
>>11147
>Как это не рассматриваем?
>аименьшее значение возможно только если прямая проходит на расстоянии меньше единицы к оригину/началу координат
То есть если u,v<1, тогда a и b не могут быть больше 1
Аноним 15/02/17 Срд 00:05:23 #358 №11152 
>>11150
>если подставить 1, то получим
u=b^(1/(1+1))+a=sqrt(b)+a
Так ход мысли верный выходит? Деление, обратное к умножению в каком смысле, в каноническом виде там делить или умножить?
Аноним 15/02/17 Срд 00:06:12 #359 №11153 
>>11147
> написал готовый конечный ответ
Да вроде бы так же всё сделал, но не подходит.
Аноним 15/02/17 Срд 00:07:34 #360 №11154 
>>11149
Эх, ну ладно, спасибо, что запарился. Никак не могу понять, в чём же проблема, вроде бы всё верно, а нихуя не выходит.
Аноним 15/02/17 Срд 01:15:18 #361 №11156 
>>11149
Ебать, анон, огромное спасибо! Пересчитал, всё подошло,коварная ошибка была, на многих значениях один и тот же результат был, всю голову сломал, думал в логике ошибка. Всё таки очень важно знать, что твой ход мыслей правильный, уже не первый раз такое, когда напишешь и удостоверишься, что ошибка не в логике спустя минут 10 ошибку находишь. Спасибо ещё раз.
Аноним 15/02/17 Срд 01:49:15 #362 №11157 
>>11156
> один и тот же результат
Вот изначально было u=b^(n/(n+1))+a, а так верно
u=b^(n/(n+1))*a^(1/(n+1))+a, разница в a^(1/(n+1)), на больших н незаметно, на малых по идее должно было быть, но мб шаг маленький слишком был, особенно подло, что часто a=1 брал.
sage[mailto:sage] Аноним 15/02/17 Срд 08:50:11 #363 №11160 
В шёпот с вас, говнососы
в /d трэд создать что-ли?
Аноним 15/02/17 Срд 09:24:22 #364 №11161 
Что такое неопределенный интеграл
Аноним 15/02/17 Срд 10:22:26 #365 №11162 
>>11029
Блять, объясняю же, производная не определяется как тангенс угла касательной, почему ты игнорируешь мои посты?
Аноним 15/02/17 Срд 11:13:47 #366 №11163 
>>11161
Это интеграл ВООБЩЕ для какой то функции.
Есть функция икс квадрат. Интеграл от нее икс куб делить на три. Однако найти интеграл мы сейчас не можем, так как нам не сказали откуда и до куда считать его.
Аноним 15/02/17 Срд 12:52:48 #367 №11167 
>>11163
Почему икс квадрат стал икс куб, да еще делить на три?

мимоимбецил
Аноним 15/02/17 Срд 13:31:53 #368 №11171 
>>11162
Ты хочешь сказать, нельзя поставить "=":
f' (x)= tg ф
ф - угол наклона касательной
О чём с тобой тогда и говорить.
Перечитай тему.

Я пытался сделать неправильный предел правильным вчера весь день, но мне это не удалось в итоге последняя попытка исправить ситуацию провалилась (а я их дохрена перебрал) и я вернулся к своему первому утверждению, что мы не можем поставить знак "=" между пределом и производной.

Что значит:
"производная не определяется как тангенс угла касательной"?
Аноним 15/02/17 Срд 13:34:51 #369 №11173 
>>11163
какая разница, ведь эта площаль лишь примерна на нелинейных.
Аноним 15/02/17 Срд 13:41:52 #370 №11176 
Производные.
Почему мы пишем dx, если он всегда равен Δx и мы уже имеем Δx, зачем нам лишнее обозначение? Для красоты?

Пределы.
Нужен математический термин для обозначения материнской функции относительно урезанной условиями функции из предела, которая никогда не достигнет некоторого значения, хотя эта же функция, но без условий предела (x-x0) легко той точки (x0) достигает.
Аноним 15/02/17 Срд 13:51:34 #371 №11179 
>>11162
Википедия с тобой не согласна.
https://ru.wikipedia.org/wiki/%D0%9F%D1%80%D0%BE%D0%B8%D0%B7%D0%B2%D0%BE%D0%B4%D0%BD%D0%B0%D1%8F_%D1%84%D1%83%D0%BD%D0%BA%D1%86%D0%B8%D0%B8#.D0.93.D0.B5.D0.BE.D0.BC.D0.B5.D1.82.D1.80.D0.B8.D1.87.D0.B5.D1.81.D0.BA.D0.B8.D0.B9_.D0.B8_.D1.84.D0.B8.D0.B7.D0.B8.D1.87.D0.B5.D1.81.D0.BA.D0.B8.D0.B9_.D1.81.D0.BC.D1.8B.D1.81.D0.BB_.D0.BF.D1.80.D0.BE.D0.B8.D0.B7.D0.B2.D0.BE.D0.B4.D0.BD.D0.BE.D0.B9
Аноним 15/02/17 Срд 14:00:40 #372 №11181 
есть книжки типа математика для умственно отсталых?
15/02/17 Срд 14:01:55 #373 №11182 
>>11181
Бурбаки.
Аноним 15/02/17 Срд 14:06:27 #374 №11184 
>>11182
я открыл какую-то случайную их книгу а там уже на первых страницах непонятные значки
Аноним 15/02/17 Срд 14:10:13 #375 №11185 
Кстати говоря:
a->ф
tg a -> tg ф
Δy/Δx -> dy/Δx
Δy -> dy
Об этом говорит и рисунок.
Аноним 15/02/17 Срд 14:12:30 #376 №11187 
>>11184
Умственно отсталые могут понимать только знаки, словами им не понятно.
Аноним 15/02/17 Срд 14:15:15 #377 №11188 
>>11187
а слова это не знаки
Аноним 15/02/17 Срд 14:16:40 #378 №11189 
>>11188
У умственно отсталых свои собственные знаки, понятные только им, ну как у слепых алфавит с пупырками. Главные дцпшник Гротендик, у него была микроцефалия почти с летальным исходом.
Аноним 15/02/17 Срд 14:25:42 #379 №11190 
изображение.png
опасный поворот лол
Аноним 15/02/17 Срд 14:45:36 #380 №11194 
>>11185
lim{Δx->0}Δy -> dy = f'(x)Δx
lim{Δx->0}f(x+Δx)-f(x) -> dy = f'(x)Δx
Аноним 15/02/17 Срд 16:22:44 #381 №11203 
бурбаки-определение 1 же.png
>>11190
В википедии написана ложь. На самом деле определение - пикрелейтед.
Аноним 15/02/17 Срд 16:42:01 #382 №11204 
Поясните за теорию категорий и зачем она нужна.
15/02/17 Срд 16:44:36 #383 №11206 
>>11204
Говно без задач. Не нужна.
Аноним 15/02/17 Срд 16:51:09 #384 №11207 
>>11206
А чего так?
Аноним 15/02/17 Срд 16:56:59 #385 №11209 
>>11167
Потому что площадь это ширина на длину.
Ширина у нас x^2 а длинна x. Умножение дает x^3. Но так как у нас график занимает только одну треть этой всей площади, то нужно поделить на три для правильного ответа.

>>11173
Чего чего?
Аноним 15/02/17 Срд 16:57:18 #386 №11210 
Почему математики создают такие определения сложные к восприятию на слух?
Аноним 15/02/17 Срд 16:58:18 #387 №11212 
>>11209
А зачем размещать график на площади?
Аноним 15/02/17 Срд 16:59:19 #388 №11213 
>>11212
Секущую к касательной уже приблизил?
Аноним 15/02/17 Срд 17:00:00 #389 №11214 
Я понял, наш тренд посетил РЫБНИКОВ или один из его адептов.
Аноним 15/02/17 Срд 17:01:14 #390 №11215 
>>11214
Моя фамилия Карпов.
Аноним 15/02/17 Срд 17:02:09 #391 №11216 
>>11213
Я разве хотел?
Аноним 15/02/17 Срд 17:05:47 #392 №11217 
>>11214
Это что-то вроде МОРСКОЙ УЛИТОЧКИ! Можно отдельный мем запилить.
Аноним 15/02/17 Срд 17:20:56 #393 №11218 
>>11217
http://vserod.com/
Он уже и так мемный дед.
Мне иногда кажется что он так тролирует. Ну или бабло рубит с тупых приРОДных гоев
Аноним 15/02/17 Срд 18:07:46 #394 №11226 
>>11209
Да площади примерны по интегралам и в линейных примерны.
Аноним 15/02/17 Срд 18:11:18 #395 №11228 
>>11171
>>11179
Именно, касательная определяется через производную, а не наоборот, то что ты говоришь про прямую, которая якобы получается при дельта x = нулю, это не определение касательной, это хуита.
Аноним 15/02/17 Срд 18:15:36 #396 №11230 
>>11171
> Ты хочешь сказать, нельзя поставить "=":
> f' (x)= tg ф
Не нужно говорить за меня, что я хочу сказать, сам перечитай мои посты ещё раз, внимательно.
Аноним 15/02/17 Срд 18:21:58 #397 №11233 
>>10508 (OP)
Где скачать все тома образовательной манги, без регистрации и смс?
Аноним 15/02/17 Срд 18:22:05 #398 №11234 
>>11228
Как ты найдёшь производную не зная формул, которые даёт геометрический смысл?
Аноним 15/02/17 Срд 18:23:47 #399 №11236 
>>11230
Как ты определяешь производную?
Аноним 15/02/17 Срд 18:25:21 #400 №11237 
https://2ch.pm/sci/res/403016.html#404623

Есть такая идея программируемого обучения. Заинтересован ли кто в выпуске такого учебника для математики для широких масс?
Аноним 15/02/17 Срд 18:34:13 #401 №11238 
>>11237
Кушай шоколадку после каждого взятого интеграла из Демидовича - вот тебе и программируемое обучение.
Аноним 15/02/17 Срд 18:34:28 #402 №11239 
>>11236
Выше есть ссылка на википедию.
Аноним 15/02/17 Срд 18:38:17 #403 №11241 
>>11228
Если существует предел:
lim{Δx->0}Δy/Δx
тогда он называется "производная" функции.

Δx не доходит до 0, об этом нам говорит сам предел.

Такой предел есть, никаких проблем и производная есть.

Но при "Δx не доходит до 0" секущая не стала касательной и мы не можем приравнять
lim{Δx->0}Δy/Δx=tg ф, где ф - угол касательной.

И тогда мы не можем найти дифференциал dy.
Аноним 15/02/17 Срд 18:40:16 #404 №11242 
а посоветуйте учебник по матану где подробно круто поясняется с нуля что такое интеграл. Про производные, бесконечно малые, дифференциалы я все понял, в шараге щас интегралы начали проходить хочу разобраться
Аноним 15/02/17 Срд 18:43:36 #405 №11243 
>>11241

А следовательно:
Процесс нахождения производной - не дифференцирование функции.

Тут и спорить не о чем.
Аноним 15/02/17 Срд 18:43:56 #406 №11244 
>>11241
> Но при "Δx не доходит до 0" секущая не стала касательной
Не стала, и что?
> мы не можем приравнять
> lim{Δx->0}Δy/Δx=tg ф, где ф - угол касательной.
Почему это не можем?
Тебе всё-таки стоит уже почитать, что такое предел.
Аноним 15/02/17 Срд 18:47:36 #407 №11245 
>>11244
тот человек
>>11146
с тобой не согласен

Аноним 15/02/17 Срд 20:53:50 #408 №11252 
SpaceSuperNova.jpg
https://www.youtube.com/watch?v=hUkugkUnZ8k


Пределы.

Давайте я вам объясню главную проблему всех пределов:
lim{x->x0}f(x)=y0
В этой записи присутствует логическая ошибка:
Там нельзя ставить знак "=".
Любой предел должен выглядеть так:
lim{x->x0}f(x)->y0
Ведь пока мы не достигли точки x0, до те пор мы и точки y0 не достигли, тогда о каком "равно" речь?
Но видимо, в головах классиков пошли логические нескладухи в дальнейших размышлениях и они решили написать "=".
Ну типа слева предел и справа предел и поэтому "равно", а там вовсе не равно, а одностороннее стремление, причём бесконечное: мы никогда не достигнем ни x0, ни y0.
Ещё хуже, что теперь мы не можем сказать "предел в точке".
Теперь у нас, как и должно был быть: "предел функции на промежутке до точки y0, не включая точку y0". Саму точку мы никогда не включаем в классических пределах. И мы должны подчеркнуть, что y0 - конечная точка.
"Примерно равно" использовать в пределах просто глупо, в пределах у нас всё стремится - "->".
И глупец тот, кто поставит "равно" там, где его нет, никогда не было и не будет и быть не может вообще.


Бесконечно малые.

Сейчас дела обстоят так:
lim{x->x0}f(x)=0
"Функция бесконечно убывает в точке, где она равна 0".
Я исправил эту проблему:
lim{x->x0}f(x)->0
"Функция бесконечно убывает на промежутке до точки 0, не включая точку 0."
Откуда убывает - зависит от функции, но всегда до 0 в бесконечно малых и обязательно не включая 0.


Достижимый Предел. Аchievable LIMit.

alim{x->x0}f(x)=y0
"y0 - достижимый предел функции в точке x0." Вот теперь именно в точке, а не на промежутке до неё, не включая её саму.
Мне пришлось добавить ещё одну штуку в математику, но это не предел и не вид пределов.
"Предел", он же "Классический Предел" - это совсем другая вещь - это "lim". И alim мы никогда не называем "пределом", это конкретно "достижимый предел".

alim{x->x0}f(x)=y0 - короткий вариант записи, есть и другие варианты, но мы не будем углубляться.

Ну так вот в достижимом пределе x стремится, достигает и становится равен x0, а y стремится, достигает и становится равен y0, чего никогда не произойдёт в классическом пределе.


Достижимый Предел и дифференциал.

Вот этот достижимый предел мы наконец можем приравнять к tg ф, где ф - угол наклона касательной:
af '(x) = alim{Δx->0}tg a = tg ф
af '(x) – уже не классическая производная, но её аналог для alim{Δx->0}tg a ("аналог" - да, мы можем так говорить, тем более в данном случае).
a - угол наклона секущей.
alim{Δx->0}tg a = tg ф – это значит, что они стали равны в тот момент, когда Δx стал равен 0, до того момента, они не равны.

Но "alim{Δx->0}tg a" нам в случае нашего аналога производной, к сожалению, ничего не даёт, ведь здесь alim{Δx->0}tg = alim{Δx->0}Δy/Δx = 0/0 Мы получаем неопределённость.
Δy = 0, когда Δx достигает нуля:
Δy=f(x+Δx)-f(x)=f(x+0)-f(x)=f(x)-f(x)=0

А из неопределённости вида 0/0, нам tg ф не найти.
И так как tg ф = dy/Δx, то и dy (дифференциал функции в точке Δx=0) нам из достижимого предела alim{Δx->0}tg a не найти.
А из классического предела, т.е. из производной мы можем найти лишь примерное значение dy.


Заключение.

И пусть конкретно здесь "Достижимый Предел" не пригодился, но он ещё займёт достойное положение в математике, а может быть и превзойдёт своего логически ошибочного классического предка.
Всем спасибо за внимание.
Аноним 15/02/17 Срд 21:37:27 #409 №11253 
Коробка набита плотно 10 яблоками.
Мы высыпаем их на стол и начинаем складывать обратно.

Мы убрали все яблоки в коробку с помощью классической теоретической модели, как ещё наши деды делали:
lim{x->x0}f(x)=9
Оп, одно яблоко лишнее, ну как обычно :), оно осталось в руке и мы не понимаем, почему это яблоко не влезает обратно в коробку.

Мы убрали все яблоки в коробку реалистичным путём, забыв про дедовский метод:
alim{x->x0}f(x)=10
Все яблоки на месте.

Достижимый предел не имеет дурацких ограничений предела.


Мы производим приблизительные расчёты при помощи предела, нам никто не мешает производить их и при помощи достижимого предела.
Кроме того, в пределе нет конечной точки, а точки y0 мы ни в коем случае никогда не достигаем, там частенько всё чисто приблизительно, а достижимый предел позволит производить нам точные расчёты гораздо чаще.
Аноним 15/02/17 Срд 22:09:58 #410 №11254 
>>11252
>>11253
Как называется эта болезнь?
Аноним 15/02/17 Срд 22:12:30 #411 №11255 
Я так полагаю, достижимый предел полностью вытеснит и заменит собой предел почти везде, ведь в нём всё то же самое, но больше и лучше.
Достижимый предел универсальнее.

Не хочешь 0/0 - не надо, подставляй близкие значения, выкидывай +Δx и получишь приблизительные значения, как и в пределе. И приблизительную dy получишь, никаких проблем.

Самое главное их - не путать, lim - классический предел, присутствует уже в огромном количестве работ, теорий, гипотез, патентов, различных изобретениях и он живёт по своим правилам, а alim живёт по своим.

Можете продолжать писать =A в пределе и говорить, что "A - предел функции в точке", вместо ->A и сложного "A - предел функции на промежутке до точки A, не включая точку A", ведь, как я уже сказал, этот предел именно в таком "классическом" виде и с такими правилами везде упомянут.
Аноним 15/02/17 Срд 22:19:58 #412 №11256 
>>11254
Конструктивизм.
Аноним 15/02/17 Срд 22:20:52 #413 №11257 
>>11254

Я болен способностью думать и понимать то, о чём я думаю.

В пределе (lim) мы не имеем права выбрать какую-либо точку из бесконечного количества, вставить её в предел и написать знак "=", мы так ни одного точного решения не получим (есть исключения).

А в alim я со спокойной совестью выбираю конечную точку, подставляю в предел и ставлю знак "=" и я точно знаю, что это 100%-е абсолютно верное решение, а не примерное, которое даст большие погрешности в определённых случаях.
Аноним 15/02/17 Срд 22:23:39 #414 №11258 
>>11257
Ладно, шутка затянулась, придумай что-нибудь другое уже, на твою толстоту никто не ведётся уже.
Аноним 15/02/17 Срд 22:28:25 #415 №11259 
>>11256
Любая наука начинается с философии.
Но я ненавижу философию потому, что по статистике большинство первейших утверждений оказываются ложными и любая философия в итоге превращается в смесь ошибок, слабоумия и шизофрении.
Аноним 15/02/17 Срд 22:29:37 #416 №11260 
>>11258
Не понимаешь - не читай и тем более не пиши.
15/02/17 Срд 22:29:41 #417 №11261 
>>11259
Поэтому, следует отказаться от конструктивизма.
Аноним 15/02/17 Срд 22:33:50 #418 №11262 
>>11260
Если б только ты следовал этому принципу...
Аноним 15/02/17 Срд 22:34:44 #419 №11263 
Мы сами творим математику.
15/02/17 Срд 22:38:25 #420 №11264 
>>11254
Пучкизм Модульный фашизм в терминальной стадии.
Аноним 15/02/17 Срд 22:46:48 #421 №11265 
Как люди умудряются решать пределы, подставляя конечное значение вместо x?

Т.е. в пределе x->x0, а они берут и запихивают x0 в предел вместо x, ну так решения же не верны получаются, там везде знаки примерно равно после этого нужно ставить, а у вас у всех "=" написано.
Вот как так?
И всех всё устраивает, странно это.
А вот меня не устраивает.

Так мы можем делать только в alim и тогда это будут знаки "=".
Аноним 15/02/17 Срд 22:49:38 #422 №11266 
>>11265
не только после этого, но и перед действием такой подстановки.
Аноним 15/02/17 Срд 22:52:11 #423 №11267 
>>11265
Короче, вы решаете пределы, тупо спекулируя примерностями и в конце ставите знак "=". (и до этого)

Ну тогда вас всех к психиатру за идиотию. :)
15/02/17 Срд 22:58:39 #424 №11268 
>>11267
>(Автор этого поста был забанен. Помянем.)
Найс.
Аноним 15/02/17 Срд 23:09:21 #425 №11269 
>>11267
просто я устал и не то сказал, что хотел.
Аноним 15/02/17 Срд 23:36:21 #426 №11272 
>>11265
>Т.е. в пределе x->x0, а они берут и запихивают x0 в предел вместо x
Непрерывность.
Аноним 16/02/17 Чтв 00:05:21 #427 №11273 
Почему среди математиков так много глубоко верующих людей?
Аноним 16/02/17 Чтв 00:15:06 #428 №11275 
>>11273
Примеры таких?
Аноним 16/02/17 Чтв 00:41:15 #429 №11276 
Здравствуйте, /math/аны. Возник такой вопрос: насколько я понял математика это такое собирательное название для десятков разных учений, в каждом из которых куча подразделов, собственной терминологии/методологии, альтернативных взглядов и трактовок и тд. Так вот, по логике у всего этого дерева должен быть некий базовый корень(в который, как минимум, входит арифметика с тригонометрией) Может ли кто-нибудь набросать примерный список базовых, основных разделов математики и сильно ли он отличается от того, что преподают в школе?
Аноним 16/02/17 Чтв 00:42:28 #430 №11277 
>>11276
Ну и, возможно, есть некая книженция, в которой автор обзорно проходится по всему этому дереву, было бы занятно почитать такое.
Аноним 16/02/17 Чтв 00:43:49 #431 №11278 
>>11273
Не больше, чем в других профессиях.
Аноним 16/02/17 Чтв 02:35:27 #432 №11283 
>>11276
Официальный классификатор математики
http://www.ams.org/mathscinet/msc/pdfs/classifications2010.pdf
Аноним 16/02/17 Чтв 02:47:49 #433 №11284 
>>11283
Спасибо! Именно то, что хотел! И ещё такой вопрос: есть ощущение глубокой связи между теорией хаоса(или динамических систем, вроде как первое подвид второго) и многими вещами, происходящими в природе/социуме/сознании, короче трудно объяснить. Хотелось бы понимать эту область математики. Какие области необходимо освоить в качестве порога вхождения?Ну и совсем в идеале учебников по этой теме годных, хотя у меня скачан торрент с кучей книг по математике, если что оттуда возьму.
Аноним 16/02/17 Чтв 02:48:29 #434 №11285 
>>11284
> по этой теме
этим темам
Аноним 16/02/17 Чтв 02:52:49 #435 №11286 
>>11284
Уровень знаний базовый: анализ, вероятность. Всё это совсем в базовом варианте МухГУ. Ещё немного по алго/графам/комбинаторике/статистике, но там ещё меньше.онлайн-курсы
Аноним 16/02/17 Чтв 04:31:29 #436 №11287 
>>11283
интересно, кто её "официальной" сделал?
Аноним 16/02/17 Чтв 04:44:04 #437 №11289 
А эти Mathematical Reviews и Zentralblatt MATH, случаем, не государством спонсируются?

А то лично я уже по горло сыт "наукой от государства".
Аноним 16/02/17 Чтв 05:13:45 #438 №11290 
1483984318513.jpg
>>10508 (OP)
Ребят просвятите меня пожалуйста уже, заблудшего в свои анальные пещеры безумия, в чём смысл мнимой еденицы?
я так осознал ее положение в комплексных числах, что она лишь показывает угол наклона между числовыми векторами, нахуя тогда придумывать какую-то ебаторию с -1? Когда можно просто взять какую-нить букву и обозначать угол(вектор поворота).
Аноним 16/02/17 Чтв 06:28:04 #439 №11291 
>>11290
https://habrahabr.ru/post/246747/
Аноним 16/02/17 Чтв 06:43:54 #440 №11292 
diff1.png
diff2.png
>>11176
>Почему мы пишем dx, если он всегда равен Δx и мы уже имеем Δx, зачем нам лишнее обозначение? Для красоты?
Вроде того, но нет, пикрилейтед
>Нужен математический термин для обозначения материнской функции относительно урезанной условиями функции из предела, которая никогда не достигнет некоторого значения, хотя эта же функция, но без условий предела (x-x0) легко той точки (x0) достигает.
Нихуя не понятно, материнские функции, вообще охуеть.
Аноним 16/02/17 Чтв 06:54:25 #441 №11293 
>>11161
Неопределенный интеграл - это первообразная функция + константа. Если угодно, отыскание неопределённого интеграла это действие обратное дифференцированию, как деление - действие обратное умножению.
Это говно было нужно Ньютону с его механикой, т.к. дохуя задач, в первую очередь разыскание скорости по известному ускорению, сводится у интегрированию. Константу интегрирования определяют из начальных/граничных условий.
Определенный интеграл - это в первую очередь охуевшая взятая в пределе интегральная сумма. Это говно было нужно в задачах по разысканию квадратур и кубатур площадей и объёмов, тогда очень любили сводить всё к понятиям "равновеликий", т.к. площадь квадрата и объём куба разыскать несложно фигур. Между ними есть связь - ф. Ньютона-Лейбница.
Аноним 16/02/17 Чтв 10:49:22 #442 №11294 
>>11292
Он теперь на http://dobrochan.ru/u/index.xhtml
Аноним 16/02/17 Чтв 12:34:43 #443 №11295 
>>11272
Что даёт непрерывность, если x не может стать x0 по определению предела?


А ещё когда мы работаем с пределом, у нас бесконечное количество точек и мы меряем всё точечным эквивалентом. И меряем и мерямем итд
И когда мы устремим x от нуля не 1, а к 10, для предела ничего не изменится. И так бесконечность и так бесконечность.
А для нас x и y начнут стремиться к другим числам. А в пределе нужно будет пройти всё тот же бесконечный путь.
Аноним 16/02/17 Чтв 13:18:49 #444 №11299 
>>11295
Закиньте ему уже CIH на пека, пусть он учит фихтенгольца на дхду.
Аноним 16/02/17 Чтв 13:26:58 #445 №11300 
>>11292
как вроде того, но нет?
Геометрический смысл производной, рисунок, там dy-ку соответствует Δx. Всё точно.

А вообще штуке dy-ку соответствует некий dx по логике вещей.

И если мы не поставим знак "=" между dx и Δx: dx = Δx, тогда у нас получится 2 разных dy.
Аноним 16/02/17 Чтв 13:29:09 #446 №11301 
>>11293
Да но если, константы точно не было и когда мы в примерах пишем +C (+0), на мой взгляд, это уже глупо выглядит. Мы с таким же успехом можем ещё добавить дробь с делением на 1 и взять всё в скобки и в первую степень.
Аноним 16/02/17 Чтв 13:31:13 #447 №11302 
Как понять геометрию?
Аноним 16/02/17 Чтв 13:31:25 #448 №11303 
>>11301
>на мой взгляд
Значение знаешь?
А то что дробь 2.7 и 2.70 это разные дроби ты знаешь?
Аноним 16/02/17 Чтв 13:37:30 #449 №11304 
>>11302
Сначала освой элементарную логику.

Я вот от природы одарённый и я говорю, что нельзя поставить занк "=", когда там чистое "->", но здешние логики не знают и утверждают обратное.
Аноним 16/02/17 Чтв 13:37:43 #450 №11305 
>>11302
Доказывай теоремы
Аноним 16/02/17 Чтв 13:38:24 #451 №11306 
>>11304
Приведи прмер
Аноним 16/02/17 Чтв 13:39:12 #452 №11307 
>>11303
>А то что дробь 2.7 и 2.70 это разные дроби ты знаешь?
Разные в каком смысле?
Аноним 16/02/17 Чтв 13:43:43 #453 №11308 
Что такое картофан? Как сделать из ребенка гения? Можно ли пичкать ребенка неэвклидовой геометрией и теорией категорий до того, как он освоил натуральные числа? Что из этого будет?
Аноним 16/02/17 Чтв 13:44:23 #454 №11309 
>>11306
Вот пример:
lim{x->x0}f(x)=y0
Все думают, что там знак равно, но в в этой штуке x не должен стать x0, он к нему бесконечо стремится.

А если так, то и функция никогда не станет y0, т.е. левая часть никогда не станет =y0.
И поэтому я ставлю там "->":
lim{x->x0}f(x)->y0

А остальные поливают меня грязью, хотя я им объяснил ситуацию.
Они не понимают, что такое предел и говорят мне: иди учи что такое предел.
Аноним 16/02/17 Чтв 14:36:30 #455 №11311 
Подкиньте, пожалуйста, объяснения декартового произведения множеств на пальцах.
Аноним 16/02/17 Чтв 14:51:45 #456 №11312 
14759122166490.jpg
>>11291
>давайте прикинемся шлангом
удивительные открытия пытливого ума
пиздец блять
если все было действительно так, то я просто в ахуе от этого цырка, охуеть исследование - ткнул пальцем и чот вышло чем дальше интересуюсь всем этим, тем больше возникает ощущение, что современная математика - это костыль на костыле для инвалидов от ума, к строгой научной дисциплине не имеющая никакого отношения.
Аноним 16/02/17 Чтв 14:52:16 #457 №11313 
>>11309
А следовательно, пределы вообще не дают точных решений, они их в принципе не могут дать.

Пределы нужно решать: от и до.
Если там бесконечное множество точек но на конечном промежутке, да даже если и устремится в бесконечность, не имеет значения, мы леко записываем решение промежутком, включая первое значение x, но не включая x0.
Аноним 16/02/17 Чтв 15:00:58 #458 №11314 
>>11313
Абсолютно точное решение предела будет:
[y,y0) - полуоткрытый интервал.
Нет, там не одна конечная точка должна быть, там этих точек бесконечное множество до y0, а к ней самой мы не имеем права приравнять.

А то "примерно равно одно число", которое вы получаете, сейчас, ещё и думая, что там "=", это уже верх слабоумия.
Аноним 16/02/17 Чтв 15:03:45 #459 №11315 
>>11314
А если вместо первой точки бесконечность, тогда:
(inf,y0) - открытый интервал.

Итого, всего два возможных варианта.
Аноним 16/02/17 Чтв 16:04:06 #460 №11323 
>>11284
Начни изучение мира с понимания того, что в этом мире не бывает случайных событий, здесь вообще нет и в принципе не может быть ничего случайного.

Одно событие строго вызывает другое событие или ряд событий, система едина и живёт по одним правилам. Процессы перемешиваются, но всё равно это чёткая и строгая система, такая же как кристаллическая решётка.

Ты думаешь, что загадал случайное число, на самом деле это не так.

Ты случайно подскальзнулся и упал? Тоже нет.

Конченные идиоты только верят в случайные события.

Случайность - это пережиток прошлого, это волшебство.

Единственная проблема заключается лишь в том, что та система, в которой мы живёт - для нас супер сложная система на сегодня. И для нас пока просто не реально расчитать её. Мы пока не знаем со 100% вероятностью что будет на самом деле.
Но мы можем делать урезанные модели и расчитывать их при помощи компьютеров, совсем простые модели, даже одним нашим мозгом могут быть просчитаны.
А твой мозг, между прочим, постоянно обрабатывает простейшие модели, даже не спрашивая тебя. Ты и сам можешь представить что-то относительно простое и расчитать эту модель.
Аноним 16/02/17 Чтв 16:47:05 #461 №11328 
>>11323
Это всё так, по крайней мере моё мироощущение очень сходно с такой моделью, именно потому я так выпал, узнав про эту область математики, к тому же постоянно натыкаюсь на её отголоски повсюду: то сознание, оказывается, аттрактор, то пришла в голову мысль о том, что самые стабильные системы имеют два центра(двупартийность, два полушария и тд), стал гуглить и опять попал на
>. Теорема Пуанкаре — Бендиксона утверждает, что двумерное дифференциальное уравнение имеет очень стабильное поведение
то, оказывается, волновая функция из того же семейства да и в целом, потому и хочу на другом уровне, не интуитивном, более конкретном что ли, трудно объяснить, посмотреть на это. Всё же это вроде как серьёзная область математики, сомневаюсь, что вот так, без фундамента, получится правильно понимать написанное. Я бы в тематический тред пошёл, но в поиске что-то ничего не выдаёт.
Аноним 16/02/17 Чтв 17:01:18 #462 №11330 
>>11323
Только я скорее думал про это не как про детерминированность, а как о, ну вот, как раз слово уравнение хорошо подходит, некий набор законов, ну например гравитация та же, в присутствии которого среда деформируется во времени, полностью предсказать значение каждой точки невозможно, зато можно предсказывать/менять режим работы всей системы, в общем такой макроуровень. Но самое весёлое, что система настроена вроде домино, точечное изменение выводит её из равновесия. Короче, жутко интересно, лол.
16/02/17 Чтв 18:27:02 #463 №11332 
Стоит ли изучать общую топологию?
Аноним 16/02/17 Чтв 18:36:10 #464 №11334 
>>11332
Стоит ли изучать то, что не приносит тебе денег и не помогает выживать, жить дольше и лучше и наслаждаться жизнью?
16/02/17 Чтв 18:37:18 #465 №11335 
>>11334
Да.
Аноним 16/02/17 Чтв 18:42:46 #466 №11336 
>>11334
Когда поймёшь, что "НЕТ", будет уже слишком поздно.
Аноним 16/02/17 Чтв 19:16:04 #467 №11340 
>>11328
Вся человеческая математика - чистые модели.

Точки в реалности не существует.

Мельчайшее событие в галактике на одном конце Вселенной как повлияет на подобное событие в другой галактике на другом конце Вселенной? И когда?
Аноним 16/02/17 Чтв 20:07:26 #468 №11351 
>>11340
>как повлияет на подобное событие в другой галактике на другом конце Вселенной? И когда?
Одному Богу известно. Великий Настройщик, все константы подобраны, все значения рассчитаны, на одну миллионную влево/вправо и через миллионы лет схлопнется игрушка,этого нельзя допустить, нужно лучше стараться, просчитывать, в сотый раз создавать этот мир, ну в этот раз точно получится продержать его в стабильном состоянии подольше. Хех, красиво.
Аноним 16/02/17 Чтв 23:30:05 #469 №11362 
Посоветуйте хорошие русскоязычные учебники (с англоязычными я знаком) по
1) ОДУ
2) Нелинейные ОДУ
3) ДУ в частных производных
Аноним 16/02/17 Чтв 23:34:13 #470 №11363 
Снимок экрана от 2017-02-16 23-35-38.png
>>11351
>Великий Настройщик
Вот, вспомнил, откуда это.
Аноним 17/02/17 Птн 02:07:06 #471 №11367 
20170217020000-1.jpg
Помогите найти ответ на тупой вопрос
Аноним 17/02/17 Птн 02:12:08 #472 №11368 
>>11367
Отбой. Это очевидно.
Аноним 17/02/17 Птн 02:29:30 #473 №11369 
>>11363
Полная хуйня то, что там написано.
Аноним 17/02/17 Птн 03:21:56 #474 №11370 
>>11369
Поподробнее? Вроде Пенроуз, уважаемый чувак.
Аноним 17/02/17 Птн 07:38:15 #475 №11375 
>>10992
Бамп
Аноним 17/02/17 Птн 08:48:03 #476 №11377 
Где посмотреть вейвлет-метод на примере колебаний струны?
sage[mailto:sage] Аноним 17/02/17 Птн 11:05:16 #477 №11380 
>>11375
алфутова устинов
Аноним 17/02/17 Птн 13:50:29 #478 №11381 
>>10508 (OP)
Я пропустил весь школьный курс математики, все предлагаемые вами книги слишком сложны для меня. Что мне делать? Неужели я в 21 уже все проебал и поздно учить науки.
Аноним 17/02/17 Птн 14:02:37 #479 №11382 
>>11381
Khanacademy, школьные учебники
Аноним 17/02/17 Птн 14:43:57 #480 №11383 
>>11381
http://lesswrong.ru/w/Цуёку_наритаи_Я_хочу_стать_сильнее
Аноним 17/02/17 Птн 17:11:54 #481 №11384 
https://youtu.be/uWwUpEY4c8o - как можно доказать это утверждение, не прибегая к ординалам? Какте есть аналогичные структуры из других частей математики?
Аноним 18/02/17 Суб 07:57:57 #482 №11398 
>>10508 (OP)
Если не ответите, то создам тРеЭд. Я школьник, здравствуйте. Как стать матшкольником? Цель - НМУ.
Аноним 18/02/17 Суб 11:23:20 #483 №11399 
>>11398
Прорешай "Теорему Абеля в задачах и решениях". Всю.
Даже если что-то знаешь, без умения доказывать каждое утверждение ты в НМУ пососешь, я гарантирую это.
Аноним 18/02/17 Суб 13:04:31 #484 №11400 
>>11399
Допустим я это осилил, что дальше? У меня есть целый год. И я не хочу сосать в НМУ.
Аноним 18/02/17 Суб 13:10:52 #485 №11401 
типа-книги.jpg
>>11400
Прочитай пикрелейтед.
Аноним 18/02/17 Суб 13:30:32 #486 №11402 
>>11401
В каком порядке?
Аноним 18/02/17 Суб 14:36:35 #487 №11404 
>>11401
Неоднозначный ответ :c то одну книгу, то кучу книг, то пососешь. Непонятно! С чего начать?
Аноним 18/02/17 Суб 14:52:52 #488 №11405 
>>11384
Видео не смотрел, выключил после слова "ordinals". Задача чисто комбинаторная, для 5 класса, никакие ординалы тут не нужны.
Докажем по индукции.

Для высоты h = 1 очевидно.

Докажем для h = 2. Пусть u - корень.

Лемма 1.
Докажем индукцией по n, что любое дерево вида
c_1, c_2, ..., c_n
|
v
|
u
можно превратить в дерево вида
v_1, v_2, ..., v_m
|
u
для конечного m.
При n = 1 это очевидно. При n > 1, если мы удалим c_1, мы получим 2 поддерева:
c_2, c_3, ..., c_n
| c_2, c_3, ..., c_n
| |
v v
| /
u
Дальше 2 раза используем предположение индукции. Лемма доказана.

Теперь посмотрим на наше дерево
... ...
| |
v_1 v_2 ...
| / /
u
По лемме 1 за конечное количество шагов каждое поддерево
...
|
v_i
|
u
можно свести к поддереву высоты 1, таким образом, все дерево сводится к дереву высоты 1, а по индукции и к дереву высоты 0. Случай h = 2 доказан.

Теперь рассмотрим случай h > 2. Рассмотрим любой узел u на высоте h - 2. По индукции поддерево с корнем в u сводится к поддереву высоты 0. Поступая так со всеми узлами на высоте h - 2, получим из исходного дерева дерево высоты h - 1. По индукции это дерево можно свести к дереву высоты 0.

Может, это и не очень строго, но для комбинаторики это нормальный уровень строгости.
Аноним 18/02/17 Суб 15:04:54 #489 №11406 
>>11401
Почему на книжке по топлогии нарисован кастет?
Аноним 18/02/17 Суб 15:06:28 #490 №11407 
>>11402
Их нужно читать параллельно. Зорича и Рудина - ближе к началу, Спивака и Милнора - ближе к концу.
Аноним 18/02/17 Суб 15:34:23 #491 №11408 
>>11405
> для 5 класса
https://en.m.wikipedia.org/wiki/Goodstein%27s_theorem
Аноним 18/02/17 Суб 15:35:28 #492 №11409 
Хорошо, наверное, быть умным и понимать математику.
Аноним 18/02/17 Суб 16:14:17 #493 №11410 
>>11408
Ну может, если прям формально доказывать, то это сложно и действительно можно назвать теоремой, но ты же видишь, что у меня доказательство правильное и что это прям пиздец очевидно.
Аноним 18/02/17 Суб 16:18:01 #494 №11411 
>>11405
Смотри, в последнем (точнее предпоследнем абзаце) у тебя, кажется, ошибка. В индукции у тебя было доказано для деревьев, выходящих из корня, а здесь ты это обобщаешь на промежуточные узлы - что неверно, потому что новые головы не растут только для ветвей, выходящих из корня. Правильно?
Аноним 18/02/17 Суб 16:26:42 #495 №11412 
>>11407
Их вполне можно изучать самостоятельно, если есть в знаниях полный школьный курс. Верно?
Аноним 18/02/17 Суб 16:27:09 #496 №11413 
>>11411
> В индукции у тебя было доказано для деревьев, выходящих из корня
Что значит "деревьев, выходящих из корня"? Каждое дерево "выходит" из своего корня.

> а здесь ты это обобщаешь на промежуточные узлы - что неверно, потому что новые головы не растут только для ветвей, выходящих из корня
Дальше вообще не понял. Что такое промежуточные узлы?
Аноним 18/02/17 Суб 16:33:57 #497 №11414 
>>11413
Прочитай формулировку задачи. Под корнем я имел в виду тело гидры, то есть корень всего дерева. Новые ветви не добавляются, только если ты убираешь прилегающую к телу ветвь. Ты говоришь: возьмем узел на высоте аш минус два. Этот узел будет промежуточным (ну, то есть не будет телом гидры, не будет корнем всего нашего целого дерева). А предположение индукции по высоте у тебя справедливо только для деревьев, корнем которых является тело медузы. Третье предложение предпоследнего абзаца ("По индукции поддерево с корнем...") - ошибка. Так?
Аноним 18/02/17 Суб 16:41:35 #498 №11415 
ziq2igeRdNY.jpg
>>11414
> А предположение индукции по высоте у тебя справедливо только для деревьев, корнем которых является тело медузы.
Нет, естественно, для любых.
Аноним 18/02/17 Суб 17:05:31 #499 №11417 
Сап, матанач. У меня к вам вопрос. Я вот тут скачал себе книгу по начертательной геометрии, но моя цель — научиться находить пересечения фигур. Правильное направление я выбрал?
Аноним 18/02/17 Суб 17:13:16 #500 №11418 
>>11415
Ты меня, кажется, не понял. Ну или я сам забыл условия. У тебя написано: по индукции любое поддерево с корнем в ю сводится к поддереву высоты ноль. Но это не так, потому что при отрубании любой головы, не растущей непосредственно из тела, вырастает новая голова. Ты и в случае для аш равному единице, и в случае для аш равному двойке можешь сказать, что высота обращается в ноль только тогда, когда ю - это тело. Если ю - е тело, то одновременно с обрубанием рассматриваемого поддерева рядом появляется другое, которое ты не учитываешь и которое у тебя как бы неявно обрубается потом по индукции.
Аноним 18/02/17 Суб 17:53:32 #501 №11419 
Как вы начали понимать математику? Когда и как произошел сей переломный момент?
Аноним 18/02/17 Суб 23:56:15 #502 №11424 
Как правильно искать корневые подпространства?
Вот есть матрица A. Я считаю размерность ядра A, A2, A3 и пока она не перестанет увеличиваться. Но схуяли если она перестанет увеличиваться на каком-то этапе, то это навсегда?
И вообще, правильный ли это алгоритм?
Аноним 19/02/17 Вск 00:06:04 #503 №11425 
>>11424
Точнее не A, а A - \lambda E, где \lambda - собственное значение.
Аноним 19/02/17 Вск 00:30:30 #504 №11428 
>>11417
>моя цель — научиться находить пересечения фигур
Всё что я видел в учебниках - это куча приёмов для построения. Можно прекрасно задрочить и получить навыки, чтобы просто работало уровень ремесленника. Хочешь по-серьёзному - выходи в сторону линейной алгебры и аналитической геометрии.
Попробуй почитать 3 главу "Роджерс Д., Адамс Дж. - Математические основы машинной графики, 2001".
Аноним 19/02/17 Вск 00:40:46 #505 №11429 
>>11428
Благодарю, анончик.
Аноним 19/02/17 Вск 01:14:33 #506 №11430 
Есть ли книга(ги) inb4:школьные учебники по которой можно полностью изучить школьный курс математики? Хочется до конца разобраться в основах.
Аноним 19/02/17 Вск 02:57:48 #507 №11431 
>>11430
А нахрена? Иди сразу на Khan Academy или Mathprofi, вполне зайдет заместо школьного курса, там все как для даунов объясняют. И по сравнению с учебником меньше вероятность, что ты дропнешь это дело от скуки. Еще можешь почитать "Алгебру" (Гельфанд, Шень), она достаточно интересная.
Аноним 19/02/17 Вск 12:05:50 #508 №11436 
Пишут, что для невырожденной матрицы существует только одна обратная матрица. Я через метод Гаусса жордана из одной матрицы вывожу две обратные. Перемножение обеих с исходной даёт единичную матрицу. Метод не универсален? где ошибка?
Аноним 19/02/17 Вск 12:28:13 #509 №11437 
>>11431
Мне важна полнота знаний, чтобы пробелов не осталось. На mathprofi такое возможно?
Аноним 19/02/17 Вск 12:33:09 #510 №11438 
>>11431
Я вот тут заглянул в список литературы оп-поста. "Элементарная математика" от Сканави покрывает школьный курс?
Аноним 19/02/17 Вск 13:17:39 #511 №11440 
>>11437
>чтобы пробелов не осталось
Чем дальше в лес, тем толще партизаны. Чем лучше разбираешься в одних вопросах, тем больше возникает других.
Аноним 19/02/17 Вск 17:14:19 #512 №11441 
>>11430
Очевидные школьные учебники. Или математика туманова и прекалькулус стюарта.
Тем более что вся школьная математика это ебаные преобразования суммы квадратов в квадрат суммы. А с другой стороны, вся математика это преобразования
Аноним 19/02/17 Вск 17:47:55 #513 №11442 
в чем различия между версиями демидовича (я только про те, которые есть в сети, в электронном виде) и если есть такой, в котором меньше всего ошибок, то дайте линк, пожалуйста
Аноним 20/02/17 Пнд 05:57:41 #514 №11474 
НАДЕЮСЬ на вас
Аноним 20/02/17 Пнд 05:58:11 #515 №11475 
IMG2750.JPG
IMG2751.JPG
НАДЕЮСЬ на вас
Аноним 20/02/17 Пнд 07:50:50 #516 №11476 
>>11475
Да ты чё, охуел? Ты ещё домашку за 5 класс на двачи закидывай.
Аноним 20/02/17 Пнд 08:06:42 #517 №11477 
>>11476
Да бля кр за 11 же
Помог бы если шаришь
Аноним 20/02/17 Пнд 11:33:51 #518 №11479 
>>11475
На каком шаге возникает проблема?
Аноним 20/02/17 Пнд 11:47:45 #519 №11481 
>>11475
короче, ставишь Wolfram Math и он тебе делает всё хорошо, это чтобы время на х. не тратить. А чтобы понимать, что делаешь, учи теорию.
Аноним 20/02/17 Пнд 15:44:27 #520 №11527 
Поясните за слова "первокультурный", "второкультурный", которые тут используются. Это что-то наполненное смыслом или болтовня очередного Тифарета?
20/02/17 Пнд 15:55:04 #521 №11531 
>>11527
Наполненная смыслом болтовня очередного тифарета.
Аноним 20/02/17 Пнд 17:06:44 #522 №11566 
>>11527
https://www.dpmms.cam.ac.uk/~wtg10/2cultures.pdf
http://mathcenter.spb.ru/nikaan/misc/Two_cultures.pdf
Аноним 20/02/17 Пнд 17:11:23 #523 №11567 
>>11566
Спасибо.
Аноним 21/02/17 Втр 09:37:25 #524 №11594 
6438b4e169a047c149077f543332155f.png
Когда математику, в голове отсутствуют мысли, становится спокойно, легко. Причем она начинает занимать все больше и больше места в моей жизни, т.е. я не просто учусь в вузике, но еще и помимо него обмазываюсь. Будто все проблемы в жизни не существуют. Боюсь поехать и стать ноулайфером с 5-сантиметровыми ногтями. Х.з. че делать.
Я подсел на абстракную иглу, анон?
Аноним 21/02/17 Втр 12:29:26 #525 №11600 
>>11594
Норм всё, от маетматики не едут, это мифы. От бытовухи едут.
Аноним 21/02/17 Втр 15:42:39 #526 №11618 
>>11566
что это за философский мусор?
потеря времени
Аноним 21/02/17 Втр 19:35:59 #527 №11625 
pic1.png
pic2.png
СпаситИ, у меня по уравнению окружность выходит как на первом пике, а мне надо как на втором, типа симметрично отраженная от Oy. Хочу хуй нарисовать из функций просто.
Аноним 21/02/17 Втр 19:41:19 #528 №11626 
1486064388-95b4f10ba462391ea904d54a8c158cf5.jpeg
Смотрю как устроенно образование в сшп и у нас и это пиздец какая разница. У нас математика существует в сферическом вакууме, в виде абстрактной самодостаточной ебалы. В сшп это инструмент, там есть нормальные книги с нормальными справочными материалами, там блять обязательно уметь пользоваться граф калькулятором. Открываешь наш учебник куча форму, каких-то ебаных лемм и теорем который нахуй не нужын нормальному человеку, но написано явно фанатом на ему ведомом языке и манере. Открываешь СШП учебник там написано вот так и так, вот такая хуйня сюда а эта сюда. У нас же трипиздец на бумажке с карандашами. Ебаная африка, ебаные белые негры. У них готовят специалистов, у нас хуй пойми кого которые потом так же учат и так же готовят хуй пойми кого. Как же горит от рашкниского худшего в мире образования.
Аноним 21/02/17 Втр 19:57:22 #529 №11627 
>>11626
Щас тебя тут говном закидают, зря поднял эту тему.
перечитываю тотомаса пятый раз
Аноним 21/02/17 Втр 20:14:42 #530 №11628 
>>11627
Российская наука это печальное наследие совка. Когда иваны и васяны копавшие землю тысячу лет до этого внезапно начитались переводов харди и прочих умных господ, нихуя не поняли и по инерции учат как попугаи поколения дефективных специалистов. Есть конечно отдельные талантливые люди, и были, но в большинстве своем это помойка состоящая из классовых крестьян с образованием слабо понимающих что они вообще делают, а чтобы не сдохнуть от голоду они преподают то что не понимают и книги пишут. И пишут их как всякие трактаты магические по буддизмам - максимально непонятно и уклончиво, перегружая ненужной и бесполезной информацией 80% из которой потом в жизни нииикогда не пригодится, а остальные 20% преподаются в устарелом виде на манер африки где пишут куском угля на камне. Зачем писать софт, учить им пользоваться? Зачем создавать эвм и полезную йобу, многие кстати в сшп бугуртят что тексас инструмент имется госконтракт для школ и прочих учебных заведений, а думаю это правильно, ведь это современно и задел на будущее. У нас же до сих пор ручка и листок, ебашьте дохуилиард формул, поймите их, умудритесь незабыть и потом через Н лет внезапно верно использовать. Ладно сейчас это как-то начало исправляться, и то за счет интернета и доступа к зарубежным материалам, наработкам интрузивов, железок типа смартфонов могущих в функционал гк.
Аноним 21/02/17 Втр 20:39:56 #531 №11633 
>>11626
>Смотрю как устроенно образование в сшп и у нас и это пиздец какая разница.
Ну начнем с того, что сами системы образования разные это раз.
>В сшп это инструмент, там есть нормальные книги с нормальными справочными материалами
В РФ тоже выпускают справочники по высшей математике.
> Открываешь наш учебник
Какой из? Учебников много, каждый под свою аудиторию.
> Открываешь СШП учебник
Открою тайну, в святых штатах тоже есть разные учебники: для даунов с картинками и что-куда вставлять, до серьезных учебников типа Рудина e.t.c
> У них готовят специалистов
У нас тоже, просто ты тупой еблан, который не разобрался в вопросе и разводит демагогию на ровном месте, грустно быть тобой.
Аноним 21/02/17 Втр 20:41:51 #532 №11634 
>>11628
> У нас же до сих пор ручка и листок
Дык больше ничего и не нужно. Ну maple можно подвезти, если влом алгебраическими преобразованиями заниматься.
Аноним 21/02/17 Втр 21:15:03 #533 №11654 
Слушайте, как это решить?
999!-1000!+1001! ⋮ 3n
n - натуральное
Надо найти максимальное значение n
Аноним 21/02/17 Втр 21:17:31 #534 №11655 
>>11626
> Открываешь наш учебник куча форму, каких-то ебаных лемм и теорем который нахуй не нужын нормальному человеку
Что ты можешь знать о нормальных людях? Ты же не человек даже, а животное ебаное.
Аноним 21/02/17 Втр 21:24:42 #535 №11659 
>>11654
Скорее всего брать логарифм и вести к 1/e. факториал на 1 /1!-1/2!+1/3!.... я бы в этом направлении начал думать. Ну а там хз.
Аноним 21/02/17 Втр 21:37:13 #536 №11664 
>>11659
Я думал для начала общий множитель m!и раскрыть скобки, а вот дальше не знаю
Аноним 21/02/17 Втр 21:37:41 #537 №11665 
>>11664
что-то какая-то м всратая получилась
Аноним 21/02/17 Втр 21:37:45 #538 №11666 
>>11654
a!=999!
a!(a+2)^2=3^n, дальше минимум фции от н найти, слева у тебя константа.
Аноним 21/02/17 Втр 21:38:13 #539 №11668 
>>11664
Не, это я погнал чёта. Проще всё гораздо.
>>11666
Аноним 21/02/17 Втр 21:39:10 #540 №11670 
>>11665
Забыл в посте привести к виду
m! − (m + 1)! + (m + 2)!
Аноним 21/02/17 Втр 21:41:55 #541 №11672 
>>11654
999! - 1000! + 1001! = 999!(1 - 1000 + 1000 * 1001).
Чтобы посчитать, сколько раз 3 входит в 999! факториал, нужно посчитать, сколько в произведении чисел, которые делятся на 3, 32, 33, ...
Аноним 21/02/17 Втр 21:42:11 #542 №11673 
>>11664
https://ru.wikipedia.org/wiki/%D0%A4%D0%BE%D1%80%D0%BC%D1%83%D0%BB%D0%B0_%D0%A1%D1%82%D0%B8%D1%80%D0%BB%D0%B8%D0%BD%D0%B3%D0%B0
Мб вот это тебе понадобится.
Аноним 21/02/17 Втр 21:43:19 #543 №11674 
>>11673
Хот яне, у тебя там константа, вообще похуй, обзовие её как нибудь типа a! и сразу всё решишь.
Аноним 21/02/17 Втр 21:50:07 #544 №11675 
>>11664
x =((999ln999-999)ln2002^2)/ln3, магазин закроется без сижек останусь.
Аноним 21/02/17 Втр 21:54:48 #545 №11676 
>>11675
какой х
Аноним 21/02/17 Втр 22:10:00 #546 №11677 
>>11676
x=310394.8
Аноним 21/02/17 Втр 22:10:47 #547 №11678 
>>11677
я не просил находить х что за х вообще
Аноним 21/02/17 Втр 22:10:56 #548 №11679 
>>11676
А, лол, ты про это. 310394 по идее, раз натуральное, то вниз округли.
Аноним 21/02/17 Втр 22:11:16 #549 №11680 
>>11678
Ну Н, какая разница.
Аноним 21/02/17 Втр 22:12:15 #550 №11681 
>>11680
Что большое такое? Ты уверен?
Оно вряд ли превышает трехзначное число
я решил уже
Аноним 21/02/17 Втр 22:16:42 #551 №11682 
>>11681
Может наебался где-то.
>врядли превышает трёхзначноё
Не, лол, ты чё.
300!<3^999
Это уже неверно например, попробуй сам, а утебя там 999!
Аноним 21/02/17 Втр 22:18:07 #552 №11683 
>>11682
А, лол, это кратность вообще. Спать пойду.
Аноним 21/02/17 Втр 22:18:51 #553 №11684 
>>11683
Сладких снов, няш
Аноним 22/02/17 Срд 02:55:51 #554 №11690 
>>11628
>Когда иваны и васяны копавшие землю тысячу лет до этого
>Российская академия наук
>Попов
>Менделеев
>Ломоносов
Ну толсто же! Не верю, что можно быть ну настолько промытым дауном.
Аноним 22/02/17 Срд 05:06:13 #555 №11697 
firefox2017-02-2205-07-18.png
Ребята, помогите решить хотя бы парочку заданий отсюда (не считая 1, 2, 6 и 8, конечно). Или хотя бы объясните, как двигаться.
Аноним 22/02/17 Срд 07:59:30 #556 №11700 
>>11628
>2017
>интернет в селах
>личный пк, доступный крестьянину
этот крестьянин бугуртит, наверное, от того что не может осилить работу с интернет-источниками и самообучение
Аноним 22/02/17 Срд 09:37:15 #557 №11701 
скапздц.jpg
>>11697
Аноним 22/02/17 Срд 17:01:18 #558 №11714 
>>11690
Не забудь, что в период жизни Ломоносова число грамотных людей в стране не превышало 10% от всего населения. СССР за 70 лет выпустил в десять раз больше кандидатов и докторов наук, чем РИ за все время.
Аноним 22/02/17 Срд 17:03:43 #559 №11715 
>>11700
В 2017, кстати, доступ в интернет имеют около 48% людей, если что. В рашке побольше, конечно, скажем 80.
Аноним 22/02/17 Срд 17:48:17 #560 №11721 
Короче, анон, я не знаю че делать, но после математики мне все остальное кажется говном. Будто чсв взлетает, хотя не стал пока таким уебком. Боюсь что поеду на шизе, даже думаю дропать изучение...
Аноним 22/02/17 Срд 18:37:54 #561 №11741 
Подскажите пожалуйста, как это решить?
"Пусть 1, 2, … – последовательность чисел, генерируемая случайным образом на отрезке [0, 1]. Этот процесс продолжается до тех пор, пока последовательность строго возрастает или строго убывает. Какова ожидаемая длина этой последовательности?"
Даже написал программу, моделирующую этот процесс, лол. Получилось что-то в районе 1,437.
Аноним 22/02/17 Срд 18:38:47 #562 №11742 
>>11741
"Пусть x1, x2, … – последовательность чисел"
фикс
Аноним 22/02/17 Срд 19:00:11 #563 №11747 
>>11741
Никак.
Аноним 22/02/17 Срд 21:22:04 #564 №11783 
Почему такой дрочь на "элитные" и "неэлитные" учебники?
Я по матану читаю Тер-Крикорова, а по алгебре Винберга просто потому, что привык и мне норм.
Один хуй ищешь параллельно что-то, у меня еще куча схем, справочников и википедия открыта постоянно
Аноним 22/02/17 Срд 21:23:30 #565 №11784 
>>11783
как будто сраный матан не перекладывают и не мусолят по двести раз, а каждый пидор придумывает СВОЙ СОБСТВЕННЫЙ ПОДХОД
Аноним 22/02/17 Срд 22:45:20 #566 №11786 
>>11684
подрочим друг другу?
Аноним 22/02/17 Срд 22:48:04 #567 №11787 
Где полный список всей математики с начала времён по 2017 включительно?
Аноним 22/02/17 Срд 22:49:01 #568 №11788 
>>11787
не нужен.
весь матаппарат переписан сто лет назад старыми пердунами.
Аноним 22/02/17 Срд 23:10:56 #569 №11791 
>>11788
а вот с этого места подробнее
Аноним 22/02/17 Срд 23:15:36 #570 №11793 
>>11684
поможешь мне понять что такое пределы, няш
Аноним 22/02/17 Срд 23:53:31 #571 №11794 
Снимок экрана от 2017-02-22 23:53:53.png
Что здесь значит средняя линия и как от неё к площади перейти?
Аноним 23/02/17 Чтв 01:54:37 #572 №11796 
>>11701
Благодарю.
Аноним 23/02/17 Чтв 01:56:05 #573 №11797 
>>11787
>>11791
https://en.wikipedia.org/wiki/Timeline_of_category_theory_and_related_mathematics
Аноним 23/02/17 Чтв 01:59:14 #574 №11798 
>>11797
всё не то
какие-то авторы каких-то теорий и всё
Аноним 23/02/17 Чтв 02:48:47 #575 №11800 
>>11798
Это и есть математика. Другой математики нет. Это все, что тебе нужно знать о математике.
Аноним 23/02/17 Чтв 03:31:08 #576 №11804 
>>11800
А я то думал, что математика гораздо старее трёх веков.
Аноним 23/02/17 Чтв 09:36:17 #577 №11808 
>>11800
А мне кажется, что математика - скорее набор различных методов и приёмов, каждый из которых создан для рещения вполне конкретных задач.

Не всё категорифицировано. А что категорифицировано - часто сложно для конкретной задачи. Выбрать только один подход и пытаться притягивать только к нему изучаемый материал - неэффективная стратегия изучения.

Аноним 23/02/17 Чтв 10:03:29 #578 №11809 
>>11791
тебе математика для чего нужна?
разделов много так-то.
т.е. как врач может быть грубо говоря "хирургом" и "психиатром".
везде придется вкатываться и учиться заново
Аноним 23/02/17 Чтв 10:04:34 #579 №11810 
>>11808
в том-то и дело что нужно изучать все отрасли, все разделы. постоянно. периодически. иначе стагнаци, гроб, кладбище
Аноним 23/02/17 Чтв 10:05:22 #580 №11811 
>>11798
твоя первая ошибка: неумение ухватывать суть и искать дальше.
и это не только в математике. это в любой области
Аноним 23/02/17 Чтв 17:47:46 #581 №11826 
Ыыы, как прокачать пространственное мышление и всякую там геометрию с тригонометрией? Я, вообще, весьма плохо двигаюсь в пространстве (во времени еще нормально) и у меня проблемы с мозжечком.
Аноним 23/02/17 Чтв 18:06:15 #582 №11827 
>>11826
>как прокачать пространственное мышление и всякую там геометрию с тригонометрией? Я, вообще, весьма плохо двигаюсь в пространстве (во времени еще нормально) и у меня проблемы с мозжечком.
Научиться быстро моделить твердотелки в solidworks/компас.
Аноним 23/02/17 Чтв 18:07:01 #583 №11828 
>>11826
Пространственное мышление тебе все равно не поможет с размерностями больше трех, так что забей на эту ерунду и займись математикой.
Аноним 23/02/17 Чтв 19:23:47 #584 №11830 
>>11828
Если я могу глядя на двумерный рисунок трехмерного объекта представить этот самый трехмерный объект, что мне мешает по трехмерному представлению 4-х мерного объекта восстановить образ последнего? Правильно, ничего.
Аноним 23/02/17 Чтв 19:53:30 #585 №11833 
>>11830
Теперь смотри сюда, мразь.
Одно измерение это когда две точки находятся на одинаковом расстоянии друг от друга.
Два когда три точки находятся на одинаковом расстоянии друг от друга.
Три когда четыре точки находятся на одинаковом расстоянии друг от друга.
Четыре когда пять точек находятся на одинаковом расстоянии друг от друга.

Нарисуй мне теперь четырехмерный рисунок в триде, выродок.
Аноним 23/02/17 Чтв 20:04:52 #586 №11837 
>>11830
Эволюционные ограничения мозга, который заточен под выживание в трехмерном пространстве.

Твой КО.
Аноним 23/02/17 Чтв 20:35:16 #587 №11839 
>>11833
Я могу нарисовать параллелепипед (трехмерное дело) мелом на двумерной доске. Глядя на рисунок, у меня, как и у любого нормального человека не будет двух мнени по поводу того, что именно там изображено. Все мы обладаем достаточной интуицией, чтобы мысленно добавить еще одно измерение и восстановить полное изображение по его проекции.
Не смотря на то, что на двумерной доске невозможно нарисовать три взаимноперпендикулярных прямые, на ней можно нарисовать абсолютно любой трехмерный объект, и он будет узнаваем всеми трехмерными существами. Понял аналогию, мразь?
23/02/17 Чтв 20:41:36 #588 №11840 
Безымянный.png
>>11839
>Не смотря на то, что на двумерной доске невозможно нарисовать три взаимноперпендикулярных прямые, на ней можно нарисовать абсолютно любой трехмерный объект, и он будет узнаваем всеми трехмерными существами.
Какая на этой картинке трёхмерная фигура?
Аноним 23/02/17 Чтв 20:51:33 #589 №11842 
>>11840
И еще больше скажу: есть такая вщь как живопись и кинематограф. Если подумать, трехмерные существа очень много времени проводят наблюдая двумерные объекты. При этом всегда сохраняя трехмерную интуицию.
Очевидно, что четырехмерный человек часто ррибегает к третьему измерению как средству линеаризации, допустим. Мы же используем чертежи всякие. Это просто упрощенная форма для трехмерных объектов. Упрощенная, но узноваемая.
Если четырехмерный человек идет на выставку, в кино или еще куда и видит трехмерные полотна, на которых запечатлены четырехмерные объекты в виде упрощенных проекций, и может узнавать их, почему ты не можешь? Головой подумай, олигофрен.
Аноним 23/02/17 Чтв 20:51:38 #590 №11843 
>>11826
Говорят пазлы помогают. Маленьким детям. Но даже я их не умею. Или, скорее, даже их я не умею.
Аноним 23/02/17 Чтв 20:59:35 #591 №11844 
Бампану свою задачу:
"Пусть x1, x2, … – последовательность чисел, генерируемая случайным образом на отрезке [0, 1]. Этот процесс продолжается до тех пор, пока последовательность строго возрастает или строго убывает. Какова ожидаемая длина этой последовательности?"
Серьезно, никто не подскажет хотя бы с чего начать? Какие темы и понятия надо почитать? Через мат ожидание не получается.
Аноним 23/02/17 Чтв 21:23:25 #592 №11845 
>>11741
Нихуя не понял, когда процесс заканчивается то? Перечитай свою задачу.
Аноним 23/02/17 Чтв 21:23:57 #593 №11846 
Поясните, пожалуйста, гуманитарию за материальную импликацию (⇒) в логике.

A ⇒ B верно, только когда либо A ложно, либо B истинно.

Я ничего не понял с такого объяснения.
Аноним 23/02/17 Чтв 21:24:50 #594 №11847 
А, нет, я сам не дочитал.
>>11845
Аноним 23/02/17 Чтв 21:28:07 #595 №11849 
>>11846
Хотя подождите. ⇒ и → это одно и тоже? Чем они отличаются?
Аноним 23/02/17 Чтв 21:32:35 #596 №11850 
>>11846
Из А следует В, т.е. если А истинно, то и В истинно, т.е. если А=1, то и В=1. Иными словами "неверно, что А=1, а В=0"
Аноним 23/02/17 Чтв 21:33:26 #597 №11851 
>>11849
Да, это одно и то же
Аноним 23/02/17 Чтв 22:18:36 #598 №11852 
>>10508 (OP)
Какую матсистему ставить на пекарню чтобы решать ящики, находить касательные к кривым и строить графики в четырех измерениях?
Аноним 23/02/17 Чтв 22:20:22 #599 №11853 
>>11839
Ты дебил, всё что ты видишь на доске - это проекции 3D пространства. Так как интуитивно ты давно аучился в этом тридэ плавать, проблем у тебя с его ощущением нет. Для того, чтобы ты убедился, какой ты долбоёб, посмотри на ютубе хотя простой гиперкуб - тессеракт. Если ты его раскусил - понял форму, его свойства, то поздравляю - ты шизик либо долбоёб-пиздобол, и это наверняка наш случай
https://www.youtube.com/watch?v=-x4P65EKjt0
Аноним 23/02/17 Чтв 22:42:38 #600 №11854 
>>11852
Мапл. ООООО КЭНЭДААА-АААААААААА!
Аноним 23/02/17 Чтв 22:43:43 #601 №11855 
>>11809
хочу заработать
Аноним 23/02/17 Чтв 22:48:11 #602 №11856 
>>11830
Пускаешь четвёртую ось по одной из трёх да и всё, я так постоянно делаю.
Аноним 23/02/17 Чтв 22:48:53 #603 №11857 
>>11837
Я это первый сказал не так давно.
Аноним 23/02/17 Чтв 22:53:18 #604 №11858 
>>11855
В покер играть что ли? Тогда надо играть вживую, интернет казино шулерят больше обычных.
И да, ты соснешь. Нужно играть с командой в которой все знают теорию игр, которая нихуя не про игры.
Аноним 23/02/17 Чтв 23:03:10 #605 №11859 
>>11856
>Пускаешь четвёртую ось по одной из трёх да и всё, я так постоянно делаю.
Или хмурого по одной из вен да и всё, я так постоянно делаю.
Аноним 23/02/17 Чтв 23:06:23 #606 №11860 
>>11859
Да, зависимости никакой, Перельман жив.
Аноним 23/02/17 Чтв 23:59:19 #607 №11862 
Посоветуйте пожалуйста что-то интересное по PDEпочитать или посмотреть, кроме того что в ОП-посте. Лучше на английском. Они такие классные, я теперь просто не могу остановиться и начать учить другие разделы математики.
Аноним 23/02/17 Чтв 23:59:22 #608 №11863 
>>11794
Бамп
Аноним 24/02/17 Птн 01:03:56 #609 №11864 
>>11862
The Analysis of Linear Partial Differential Operators
by Lars Hörmander
Все 4 тома
Аноним 24/02/17 Птн 01:57:48 #610 №11865 
>>11864
дифференциальные операторы, хуё-моё, зачем это всё надо стандартному математику общей практики?
Аноним 24/02/17 Птн 01:58:12 #611 №11866 
>>11859
кретин
Аноним 24/02/17 Птн 02:15:30 #612 №11867 
>>11865
А что, совсем не надо такое? Я вообще на биолога учусь так то.
Аноним 24/02/17 Птн 02:53:13 #613 №11868 
>>11853
Это ты дебил, я и написал выше про интуицию:
>>11842
хули ты мне мои же слова пересказываешь? Алсо, твой аргумент про тессеракт на ютубе инвалид: это уже двумерное представление четырехмерного объекта. Попробуй спроецировать куб на прямую, дохуя у тебя получится? Но если повертеть в руках трехмерную модель тессеракта, можно ясно представить, что это такое.
Аноним 24/02/17 Птн 07:06:20 #614 №11870 
>>11868
У тебя на мониторе в любом случаи двумерные проекции - либо перспективные, либо аксонометрические. Играя в твой любимый нфс и дотан на экране ты видишь всё то же плоское 2D. Но тут мы возвращаемся к интуиции, которая отрабатывает на ура в случаи 3D, а уже на 4D-кубе у тебя ломаются аналогии и представление, основывающееся на многолетнем опыте, в том числе сравнений с примитивами и ты начинаешь маняврировать с аргументами в стиле "двумерное представление четырехмерного объекта", что, кстати, само по себе верно
Аноним 24/02/17 Птн 10:57:07 #615 №11873 
>>10508 (OP)
ТОПОЛОГИЯ
При каких условиях на топологию дополнение до открытого множества M содержит все свои предельные точки?

Полнота? Сепарабельность? Как доказать?
Аноним 24/02/17 Птн 11:21:16 #616 №11874 
>>11873
Никаких дополнительных условий.

Определение. Точка x называется предельной точкой множества M, если всякая проколотая окрестность x имеет непустое пересечение с M.

Определение. Множество называется замкнутым, если оно является дополнением открытого множества.

Теорема. Множество M открыто тогда и только тогда, когда вместе с каждой точкой содержит некоторую её окрестность.
Доказательство. Пусть множество M открыто. Тогда оно является окрестностью каждой точки.
Обратно, пусть множество M вместе с каждой точкой содержит её окрестность. Тогда M является объединением открытых множеств и открыто по определению топологического пространства.

Теорема. Множество M замкнуто тогда и только тогда, когда оно содержит все свои предельные точки.
Доказательство. Пусть множество M замкнуто. Пусть U - дополнение M. Понятно, что U и M не пересекаются. Пусть x - предельная точка M. Предположим, что x не принадлежит M. Тогда x принадлежит U. Тогда U\{x} - проколотая окрестность x. Она должна пересекаться с M, что абсурдно. Значит, x принадлежит M.

Обратно, пусть все предельные точки M принадлежат M. Пусть U - дополнение M. Тогда любая точка U не является предельной для M. Это значит, что у любой точки из U существует окрестность, не пересекающаяся с M, - т.е. являющаяся подмножеством U. То есть всякая точка множества U входит в U вместе с некоторой окрестностью. Поэтому U открыто. Поэтому M замкнуто.
Аноним 24/02/17 Птн 11:50:49 #617 №11875 
>>11870
>в любом случаи двумерные проекциии
А я этого не знаю? У четырехмерных людей трехмерные мониторы, картины, чертежи в тетрадках, и тд. И они скорее всего так же живут большую часть времени в 3д, как мы в 2д. Вполне возможно что ты и вся твоя жизнь это тоже изображение на таком мониторе из четырехмерного мира. Так что 4д объекты не просто можно представить, тени от них повсюду.
Аноним 24/02/17 Птн 12:10:46 #618 №11876 
>>11874
Ух, как же ты охрененно ты написал, спасибо!!! Жалко, что сейчас занят, спрошу вечером, если что-то не пойму, разбираясь.
Аноним 24/02/17 Птн 12:14:41 #619 №11877 
Может кто-нибудь для дебила объяснить, как строить сплетения групп? У Каргаполова это какой-то треш.
Аноним 24/02/17 Птн 12:25:02 #620 №11879 
>>11875
>Так что 4д объекты не просто можно представить, тени от них повсюду.
Тени да, а вот интуитивного представления у простых трёхмерных людей пока нет. Хотя тебе разрещаю эволюционировать до n-мерного человека, размерностью n, какой пожелаешь.
Аноним 24/02/17 Птн 12:44:12 #621 №11880 
>>11877
Везде всё очень написано даже без единого примера, неужели нельзя было вставить хоть один.
Аноним 24/02/17 Птн 12:57:59 #622 №11881 
Я вот что подумал, что если начать ходить репетиторствовать по обучению вышмату для всяких студиозов и каникуляров? Будет кто то интересоваться или интригалы с дифиринциалами никому не нужны?
читаю тотомаса 8 раз
Аноним 24/02/17 Птн 15:33:43 #623 №11882 
>>11794
Помогите
Аноним 24/02/17 Птн 16:15:34 #624 №11883 
>>11882
ТЫ ЧТО, ДУРАК ЧТО ЛИ БЛЯДЬ?
https://ru.wikipedia.org/wiki/%D0%A1%D1%80%D0%B5%D0%B4%D0%BD%D1%8F%D1%8F_%D0%BB%D0%B8%D0%BD%D0%B8%D1%8F
Аноним 24/02/17 Птн 16:18:42 #625 №11884 
>>11883
у 4угольника 3 варианта средней линии
ни с одним из них я не могу решить задачу
плюс неизвестно выпуклый о или вогнутый
Аноним 24/02/17 Птн 16:29:15 #626 №11886 
>>11884
Средняя линий между диагоналями, очевидно же по тексту задачи.
Аноним 24/02/17 Птн 16:46:27 #627 №11888 
>>11886
И что здесь нужно применить?
Аноним 24/02/17 Птн 18:14:59 #628 №11891 
>>11794
можешь считать, что у нас квадрат
Аноним 24/02/17 Птн 18:16:23 #629 №11892 
>>11891
Нет. Длина диагоналей разная.
Аноним 24/02/17 Птн 18:33:43 #630 №11894 
>>11892
но если достроить, квадрат
Аноним 24/02/17 Птн 18:35:09 #631 №11895 
cколько в 200мм2 см2 ??? Подскажите, как это решить.
Аноним 24/02/17 Птн 18:35:51 #632 №11896 
>>11894
Всё равно дайт примерный ответ.
Аноним 24/02/17 Птн 18:36:26 #633 №11897 
>>11895
>см2
В /sci, быдло.
Аноним 24/02/17 Птн 18:39:13 #634 №11898 
>>11897
>В /sci, быдло.
Это задачка с найобом? из учебника 5 класса
Аноним 24/02/17 Птн 21:45:27 #635 №11901 
>>11895
Никак. Вот тебе приём уровня мартышки, чтобы такое щёлкать на раз.
В твоём случаи: 10 мм = 1 см
Делай раз: переводишь из большего в меньшее - умножаешь, переводишь из меньшего в большее - делишь.
Делай два: коэффициент умножения деления/умножения равен: 10^n, где n - число в размерностях. n=1, если линейные размеры, n=2 - площади, n=3 - объёмы.
Делай три: умножаешь на количество переводимых единиц (в твоём случаи 200)
Если "дохуя" умный, приводишь в к стандартному виду числа, выделяя мантиссы, и ловко работая с экспонентами.
Аноним 24/02/17 Птн 23:01:10 #636 №11902 
ну ты и даун.PNG
>>11895
Аноним 24/02/17 Птн 23:31:27 #637 №11904 
CodeCogsEqn.gif
>>11895
Все очень просто. Ну а лучше просто воспользоваться встроенным конвертеров в гугле, как тебе выше уже показали.
Аноним 25/02/17 Суб 02:34:18 #638 №11907 
>>11902
Google скоро умнее людей станет.
Аноним 25/02/17 Суб 12:15:24 #639 №11910 
>>11907
Уже, разве нет?
Аноним 25/02/17 Суб 12:18:31 #640 №11912 
Screenshot from 2017-02-25 14-13-12.png
HELP
Аноним 25/02/17 Суб 15:52:04 #641 №11927 
>>11894
Нет. Это скорее равнобедренная трапеция т.к. у квадрата с такими диагоналями ни одна средняя линия не может быть примерно 15. И это средняя линия соединяет не диагонали, а противоположные стороны т.к. если бы она соединяла диагонали, то можно было бы построить несколько четырёхугольников например с площадью 320 или 340. В автокаде проверял Поэтому считать надо по формуле sqrt(2727-1515)*15 и ответ 336, а не 364, который даёт формула для квадрата. Всем спасибо. хотя я сам ответ подобрал
Аноним 25/02/17 Суб 23:16:54 #642 №12008 
>>11907
Дата центры гугла уже сейчас содержат суммарно больше информации, чем все люди когда либо жившие. Но гугл неспособен "обработать" эту информацию, преобразовывая и внедряя новые концепции. Гугл сам по себе не способен плодить сущности.
Аноним 25/02/17 Суб 23:33:58 #643 №12009 
>>12008
Способен, конечно.
Аноним 26/02/17 Вск 01:17:06 #644 №12012 
Господа, поясните плз за аксиому выбора в самом простом ее варианте, у зорича как-то мутно написано

У зорича: "для любого семейства непустых множеств. существует множество С такое, что каково бы ни было множество Х данного семейства, множество "Х пересечение С" состоит из одного элемента."

Но вот семейство: {{1,2},{3,4},{1,2,3,4}}

и как тут она работает?

Аноним 26/02/17 Вск 01:53:02 #645 №12014 
>>12012
В таком варианте нужно требовать, чтобы множества семейства не пересекались. Если этого не требовать, то нужно говорить не о множестве C, а об отображении f из семейства в объединение его членов, таком, что f(X) принадлежит X
Аноним 26/02/17 Вск 06:33:18 #646 №12017 
>>12012
При безальтернативном выборе из двух противоположных сущностей обе будут являть собой исключительную хуйню.
Аноним 26/02/17 Вск 10:30:04 #647 №12018 
>>12012
{1,3}
Аноним 26/02/17 Вск 12:15:28 #648 №12021 
Selection008.png
Selection009.png
Selection010.png
>>11828
Аноним 26/02/17 Вск 13:19:21 #649 №12023 
>>12018
А ничего, что это множество даст в пересечении с {1,2,3,4} две точки, а не одну?
Аноним 26/02/17 Вск 13:58:29 #650 №12047 
>>12021
Фи, костыль и не тру, геометрической интуиции толком не дает. Лучше бы заскринил что-нибудь из трудов настоящих 4д-визуализаторов.
Аноним 26/02/17 Вск 16:27:16 #651 №12066 
TPh1Ji.png
Всем привет. Есть невзвешенный граф, нужно найти кратчайший путь от одной вершины до всех остальных. Каким алгоритмом воспользоваться? n-1 раз использовать BFS? Минимальная временная сложность в приоритете.
Аноним 26/02/17 Вск 16:28:11 #652 №12067 
>>12066
кратчайшие пути
фикс
Аноним 26/02/17 Вск 17:05:23 #653 №12076 
>>12066
>n-1 раз использовать BFS
Одного раза достаточно.
Аноним 26/02/17 Вск 18:30:56 #654 №12095 
>>12066
алгоритм беллмана-форда гугли, это классика, это знать надо
Аноним 26/02/17 Вск 19:07:56 #655 №12098 
>>12066
> n-1 раз использовать BFS?
Нет, одного раза достаточно. Первое, что приходит в голову: для каждой вершины сохранять, откуда мы в нее пришли, и когда достаешь вершину v из очереди делаешь dist[v] = dist[parent[v]] + 1.
Аноним 26/02/17 Вск 19:27:01 #656 №12100 
6b8a7d7f60d49af6377b1026c4fb58e1.jpg
Заканчиваю прикладную математику и у меня все больше чувство, что нихуя я не найду нормальную работу и от этого впадаю в уныние. Столько лет и стараний вложил...
Еще и все молодыми бизнесменами стали после школы на бмвX5, вообще охуеть
Аноним 26/02/17 Вск 21:02:12 #657 №12112 
Безымянный.png
помо гите срочно на за ночь решит
Аноним 26/02/17 Вск 23:28:37 #658 №12123 
>>12112
Ну и что тебе непонятно? Ты квадратное уравнение решить не можешь? Или ромбик нарисовать?
Аноним 27/02/17 Пнд 14:10:41 #659 №12149 
>>12100
>Еще и все молодыми бизнесменами стали после школы на бмвX5, вообще охуеть
Если хочешь так же, то дорога тебе в профессиональную математику
Аноним 27/02/17 Пнд 14:34:02 #660 №12150 
>>12149
А они там до какого знака после запятой округляют?
Аноним 27/02/17 Пнд 15:30:37 #661 №12151 
>>12149
это какую?
тензоры что-ли?
Аноним 27/02/17 Пнд 15:37:45 #662 №12152 
>>10508 (OP)
Почему основание показательной функции положительно?
Аноним 27/02/17 Пнд 16:03:10 #663 №12155 
>>12152
Чего?
Аноним 27/02/17 Пнд 17:17:41 #664 №12161 
Подкиньте, пожалуйста, красивое видео с объяснением теории множеств для маленьких детей. Вроде все и понятно, но не хватает мультяшности.
27/02/17 Пнд 17:26:52 #665 №12162 
>>12161
Лучше читни книгу Рассказы о множеств. Там совсем для детей.
Аноним 27/02/17 Пнд 17:40:45 #666 №12163 
Двач, как научиться доказывать утверждения? Хожу в НМУ (с весеннего семестра), лекции понимаю, пытаюсь делать листочки и каждый раз спотыкаюсь об мысль "а не чушь ли я мелю". Я не понимаю, достаточно ли моих рассуждений, чтобы утверждать, что изначальное утверждение доказано или нужно делать еще какие-то допущения или же спотыкаюсь об наивность собственных рассуждений, мне кажется, что это частный пример, а не общее. Разумеется, я ни разу ничего не пытался сдавать, потому что боюсь, что в меня начнут тыкать пальцем и смеяться "ха-ха, вот дебил, что за пургу он несет".
Аноним 27/02/17 Пнд 17:43:02 #667 №12164 
>>12155
y=a^(x) ("a" положительно и не равно 1). Почему положительно?
Аноним 27/02/17 Пнд 17:45:27 #668 №12165 
>>12163
Лучше создай отдельный тред про нму, в этом в основном школьники, потеряется твой пост.
Аноним 27/02/17 Пнд 17:46:14 #669 №12166 
>>12163
И вообще, странные у тебя вопросы, конечно. Ты в нму после школы пошел что ли?
Аноним 27/02/17 Пнд 18:22:55 #670 №12167 
>>12164
Потому что попробуй дать определение с отрицательным основанием. Не получится.
Аноним 27/02/17 Пнд 18:29:57 #671 №12168 
>>12163
Приведи примеры. Если твои доказательства в порядке, и ты просто себя накручиваешь, то мы тебе об этом честно скажем. Если нет - покажем, где именно ты сфейлился.
Аноним 27/02/17 Пнд 18:38:05 #672 №12169 
>>12167
Он имеет ввиду "почему нельзя взять отрицательное число, если степени с отрицательными числами возможны."
Аноним 27/02/17 Пнд 19:02:11 #673 №12170 
>>12163
Лучше решай полностью, доказывай все, вплоть до тривиальности.
Я был на твоем месте. Нарешал дома несколько листков в сентябре. Доказывал вплоть до того места, пока мне не становилось очевидно. Ну в общем пока ждал проверяющего уже примерно понял, о чем речь: у местных задротов даже для элементарной (по моему мнению) задачи было по 3-4 страницы писанины с таблицами и чертежами. Мое предчувствие подтвердилось, проверяющий был готов принять только одну, и то, если я её додумаю.
Аноним 27/02/17 Пнд 19:34:56 #674 №12172 
>>12166
Не после школы, у меня просто самооценка очень низкая и связанное с ней психическое расстройство.
>>12170
Чем история кончилась, ты сдал хоть один листок целиком? Мне кажется, мне уже поздно в этом году пытаться что-то сдавать, зато с сентября заживу!.. (аутотренинг который год)
Аноним 27/02/17 Пнд 21:57:34 #675 №12178 
>>12169
Ты только ещё больше всё запутал.
Аноним 28/02/17 Втр 03:49:20 #676 №12181 
Где "Высшая Математика Для Детей" почитать? Полный курс, пожалуйста. И чтобы вообще вся математика и розжёвано, как для полных идиотов.
Аноним 28/02/17 Втр 04:43:48 #677 №12182 
Есть уравнение кривой, заданное суммой двух интегралов по dt, первый от 1 до Х, второй от 0 до У. Сумма равна некоторой константе С Есть точка с координатами, через неё проходит касательная к кривой. Нужно найти угол с осью абсцисс. Легко решаются интегралы, находится константа, составляется уравнение, берётся производная, арктангенс и ответ. Казалось бы. Проблема в том, что интегралы эллиптические(первый вида sqrt(1+x^n), второй корень из синуса). Можете подсказать как решать такое? Ну или ткнуть в задачник с похожими примерами, потому что эллиптические интегралы мы не разбирали ещё, препод специально их туда запилил, потому нужно как-то без вычислений всё сделать похоже.
Аноним 28/02/17 Втр 05:08:14 #678 №12183 
>>12182
Отбой, разобрался. Нужно в ряд разложить.
Аноним 28/02/17 Втр 08:08:45 #679 №12184 
>>12163
>Разумеется, я ни разу ничего не пытался сдавать, потому что боюсь, что в меня начнут тыкать пальцем и смеяться "ха-ха, вот дебил, что за пургу он несет".
Сдавай задачки - это как раз и нужно, чтобы научиться тому, что такое правильное доказательство и как его понятно излагать. Стеснятся ошибиться здесь совершенно лишнее, ведь получить фидбэк от квалифицированного человека это самый простой метод узнать о своих ошибках.
Аноним 28/02/17 Втр 14:29:37 #680 №12188 
>>12181
На английском только.

https://www.gutenberg.org/files/33283/33283-pdf.pdf
Аноним 28/02/17 Втр 14:36:08 #681 №12190 
>>10508 (OP)
Здравствуйте, аноны-математики. Помогите, пожалуйста, с одной задачей. Взял из обычного учебника, решал разными способами и всегда получал один и тот же ответ, который не совпадал с ответами в конце учебника. Задача такая:

Два брата идут домой из школы с одной и той же скоростью. Однажды через 15 минут первый брат побежал обратно в школу, а когда добежал, то побежал догонять второго. Второй. оставшись один, продолжал идти в два раза медленнее. Когда первый брат догнал второго, они стали идти с прежней скоростью и пришли домой на 6 минут позже обычного. Нужно написать во сколько раз скорость бега первого брата больше их обычной скорости.

Я решал двумя способами сложным и простым:
Взял в качестве x-все расстояние, а в качестве t - обычное время. Тогда x/t - обычная скорость братьев, x/2t - замедленная в два раза скорость второго брата. В качестве ускорения при беге ввел коэффициент a. Скорость первого брата при беге ax/t . После этого получается простая задача на движение, которая сводится к поиску коэффициента а. Я это вычислил опять же двумя способами и получил значение 5,5.

Простой подход исходил из того, что раз они опоздали на 6 минут, то только из-за того, что второй брат шел в два раза медленнее, а значит все его время пути будет равно 12 минутам. Значит, если взять за х - обычную скорость, то можно вычислить все расстояние, что проделал первый брат за это время
(1/4)x+(1/4)x+(1/5)(x/2).
Тогда, если разделить это выражение на скорость первого брата с ускорением а, то можно найти время, которое он бегал. А потом приравнять полученное значение к 12 минутам и найти итоговое значение коэффициента а.
Какое из этих двух решений правильнее. Спасибо за помощь.
Аноним 28/02/17 Втр 15:14:35 #682 №12192 
В задаче нет ни слова при ускорение.
Аноним 28/02/17 Втр 15:50:18 #683 №12194 
>>12190
Идейно оба решения правильны. Как именно ты получил 5,5?
Аноним 28/02/17 Втр 17:18:11 #684 №12196 
Корень четвертой степени из -81, комплексные числа, как я понимаю ответ 3i - абсолютно ошибочнвй, направь на путь истинный, как такое осиливать?
Аноним 28/02/17 Втр 17:33:58 #685 №12197 
>>12196
1)i=корень(-1)
2)i^2=-1
3)i^3=-i
4)i^4=1
5)i^5=go to 1)

3^4=81
x^4=-1
x=-i
Корень четвертой степени из -81=-3i
Аноним 28/02/17 Втр 17:36:46 #686 №12198 
>>12197
Эх, облажался с минусом, надеюсь что-то получится исправить позже.
Аноним 28/02/17 Втр 17:59:29 #687 №12199 
>>12196
Гугли тригонометрическую форму комплексных чисел. В ней корни любой степени легко извлекаются, причём все n штук.
Аноним 28/02/17 Втр 18:40:26 #688 №12200 
>>12181
mathprofi
Аноним 28/02/17 Втр 18:57:46 #689 №12201 
14869140480260.jpg
Сколько необходимо времени дабы окончить второй уровень и перейти на шестой? Мне 14 лет на данный момент.
Аноним 28/02/17 Втр 19:16:13 #690 №12202 
>>12199
А потом пусть погуглит формулу Эйлера и будет работать с экспонентами, это же намного проще.
Аноним 28/02/17 Втр 19:21:13 #691 №12203 
>>12202
Я, тащемто, это и имел в виду. С моей точки зрения, нет разницы между показательной и тригонометрической формами.
Аноним 28/02/17 Втр 19:23:20 #692 №12204 
>>12201
От двух лет до бесконечности.
Аноним 28/02/17 Втр 19:37:39 #693 №12205 
>Геометрия в 2 томах - Кушнир
Есть что-то похожее или у кого может ссылка есть? Не могу найти почему-то.
Аноним 28/02/17 Втр 19:46:19 #694 №12206 
А есть такая книга от арифметики к алгебре, которая показывает связь оных с геометрией и как решать оные с помощью геометрии?
Аноним 28/02/17 Втр 20:25:44 #695 №12207 
>>12201
https://www.lektorium.tv/course/22939 вот здесь рассматривается большинство из того, что на шестом уровне. алсо, глянь трейлер этого курса: https://www.youtube.com/watch?v=mqAf5lOJZew
Аноним 28/02/17 Втр 20:26:16 #696 №12208 
>>12207
тащемто это 666 уровень даже
Аноним 28/02/17 Втр 20:45:45 #697 №12209 
>>12207
там нихера ничего не рассматривается, там просто лектор бессвязно пиздит о том, какой он охуенный. ляпнет что-то типа математическое, из ничего, ни начала, ни конца, группу какую-то напишет - и снова новую байку травит

Может быть, надо упороться чем-то, чтобы что-то полезное из лекций этих извлечь, но думаю, и это не поможет
Аноним 28/02/17 Втр 20:47:38 #698 №12211 
>>12209
в 14 лет то что надо
Аноним 28/02/17 Втр 21:16:21 #699 №12214 
photoNormal.jpg
Приветствую вас, обитатели /math. Вкатился 18лвл-кун, студент провинциального медицинского университета. Несмотря на то, что в перспективе я - эскулап, что ни разу не связано с математикой в большинстве случаев, хочу изучить высшую математику. Ещё в школе метался между медициной и образованием, напрямую связанным с математикой. Не буду пояснять, почему в итоге поступил в мед, а, собственно, сразу перейду к тому, зачем пришёл: распишите план самостоятельного изучения высшей математики? Со всеми аспектами типа "чьи учебники читать", "лекции какого вуза просматривать" и т.д. Пытался на досугелекциях по экономике, социологии и прочему побочному говну читать Винберга - заходит туго, так что пришёл к вам за смазкой. Выручайте, товарищи!
Аноним 28/02/17 Втр 21:39:27 #700 №12215 
>>12214
http://imperium.lenin.ru/~verbit/MATH/programma.html
Аноним 28/02/17 Втр 21:48:48 #701 №12216 
>>12214
Список литературы есть в шапке. Начинай с самого нижнего уровня. Только если почувствуешь, что всё слишком легко и скучно, переходи на более высокий. Лекции обычно ничем не лучше книг, посколько по ним и построены. Главное преимущество лекций (очных) - обратная связь с преподавателем, который может тебе персонально разжевать непонятный момент. Этот тред, в принципе, для этого и нужен.

От себя порекомендовать могу книжку Аносова - "Отображения окружности, векторные поля и их применения". Она довольно короткая и доступна для понимания десятикласснику, но весьма глубока в идейном плане. Прочитав её, ты поимеешь представление о том, какими методами работает современная математика (во всяком случае, один из самых богатых и обширных её разделов). Самое то для ньюфага.
Аноним 28/02/17 Втр 22:02:21 #702 №12217 
>>12216
140 страниц ради единственного результата, который в "Наглядной топологии" Болтянского оставлен как упражнение. Just saying.
Аноним 28/02/17 Втр 22:36:32 #703 №12218 
>>12217
Учитывая, что в эти 140 страниц входит вся базовая инфа про комплексные числа, степенные ряды и прочие необходимые энтрилевельные штуки, не вижу ничего плохого.
Аноним 28/02/17 Втр 23:38:25 #704 №12220 
>>12218
Теперь сравни с "Теоремой Абеля в задачах и решениях" и "Алгеброй в современном изложении", где на том же количестве страниц поля, кольца, групповые алгебры, римановы поверхности и гомотопические группы.
Аноним 01/03/17 Срд 00:30:00 #705 №12221 
>>12220
Сжатость - не преимущество для начинающего. Это всё равно что Рудина советовать.
Аноним 01/03/17 Срд 00:52:37 #706 №12222 
>>12214
Есть несколько вариантов.
1. Читать книги из шапки. Но скорее всего будет так же как с Винбергом.
2. Иди на mathprofi (если не нравится аналитическая геометрия, то начинай тогда с предельчиков) и KhanAcademy. Если зайдет, то потом уже и будешь подбирать книжки по заинтересовавшим тебя разделам. Еще потом можешь попробовать на MIT OCW глянуть видео лекций по курсам Multivariable Calculus, ODE, Linear Algebra, они там очень доступные и интересные (по крайней мере для меня были). Но и про практику решения всего самому тоже не забывай, обязательно делай кучу заданий. А теорию и потом успеешь подтянуть, если захочешь. Но скорее всего ты ее просто дропнешь от скуки, по крайней мере, если начнешь именно с нее. Я вот не мог заставить себя прочитать Рудина до тех пор, пока мне не понадобилась дифференциальная геометрия. А как припекло, так даже с интересом его читал, хотя до этого дропал после нескольких страниц. В общем, в математике есть куча разделов, просто найди интересные для себя. А если сильно скучно, то не издевайся над собой и пробуй другие разделы/книги/что угодно.
Аноним 01/03/17 Срд 02:36:02 #707 №12223 
>>12209
Так это и не учебник же, он там рассказывает свой взгляд на всё это, свои результаты, видимо расчёт на обратную связь и тд, вроде как диалога должно было быть по идее, но не получилось, не фартануло, он же ещё в начале говорил, типа, вы тут все разного уровня подготовки, если чё непонятно останавливайте, буду объяснять, по итогу всё равно погнал и толкового мало получилось.
Аноним 01/03/17 Срд 03:09:03 #708 №12224 
>>12223
Это вроде лекции, не? Да и нужен тебе диалог - ну, открой семинар, собери, с кем ты можешь разговаривать, ты же деятельный чувак, Пахома вон пригласил.

Я в нём ничего не понимаю и потому наверно ошибаюсь, но, по-моему, он и не пытается особо что-то содержательное рассказать.

Какой бы там уровень слушателей не был, логика какая-то в рассказе должна быть? А он на первой лекции с первых минут сразу спектралку какую-то выписывает - что, зачем, непонятно - и тут же телегу загоняет про сны и ниндзя.

Хипстер какой-то кришнаитский, блядь.
Аноним 01/03/17 Срд 06:57:29 #709 №12226 
>>12224
>Планируется разбор и обсуждение некоторых открытых проблем теории групп и маломерной теории гомотопий: проблемы асферичности Уайтхеда, D(2)-гипотезы Уолла, проблемы дыр соотношений, проблемы делителей нуля в групповых кольцах. Скорее это не курс, а беседы о теории групп и теории гомотопий, с описанием различных примеров, трюков и методов.
Разбор был, трюки были, беседы были. Нет, это не курс лекций в строгом смысле, формат не тот, о чём и было указано в аннотации. Вообще это как бы запись факультатива, что-то вроде спецкружка.
Аноним 01/03/17 Срд 07:04:06 #710 №12227 
>>12224
И вообще, блядь, проблемы открытые, он обзор охуенный сделал, дал кучу ссылок на литературу. По открытым проблемам особо ничего и не скажешь конкретного, иначе бы они и не были открытыми. Максимум можно о подходах рассказать, о перспективных методах, которые могут сработать, о явно хуёвых методах. Вот это он и делает, приправляя всякими байками, по итогу получилось нескучно. Короче, вы не правы, всего доброго.
Аноним 01/03/17 Срд 07:29:14 #711 №12228 
>>12224
>Хипстер какой-то
Решил доказать обратное, стал гуглить его работы, но хз, если честно его цитирует только он сам судя по всему. Ну ещё книжка есть на амазоне, которую никто не покупает. Может ты и прав, лол.
Аноним 01/03/17 Срд 10:45:33 #712 №12233 
>>12228
Двач. Здесь приходят к консенсусу.
Аноним 01/03/17 Срд 13:52:57 #713 №12238 
Сколько стоят услуги репетиторства профессионалов отсюда?
Аноним 01/03/17 Срд 15:55:21 #714 №12240 
18128348356863828.jpg
>>12238
>профессионалов
sage[mailto:sage] Аноним 01/03/17 Срд 18:04:11 #715 №12250 
1351880065774-b-e845.jpg
Такая тема, начал читать учебник по топологии от Вербита и тут же не въехал в данное суждение:
> Действительно, предположим, что, исходя из аксиом Пеано, нельзя ни доказать, ни опровергнуть утверждение Q "полиномиальное урав-
нение P(t1, t2, ...tn) = 0 не имеет целочисленных решений t1, t2, ...tn". В этой ситуации уравнение P(t1, t2, ...tn) = 0 таки не имеет решений, ибо, если бы такое решение было, мы бы могли его подставить в уравнение, и получить теорему "Q ложно".

С какого хрена, если нельзя ни доказать, ни опровергнуть утверждение о том, что решений не существует, то их не существует?
Аноним 01/03/17 Срд 18:28:20 #716 №12264 
>>12250
Действительно, не очень понятно. Может быть, имеется в виду следующее: рассмотрим множество решений данного уравнения - это множество либо пустое, либо нет, и зависеть от системы аксиом (Пеано + дополнительные) это не может.

Но как-то тоже сомнительно. Может быть, его полиномиальность как-то меняет дело
sage[mailto:sage] Аноним 01/03/17 Срд 18:44:16 #717 №12268 
Screenshot20170301-194405~2.png
>>12264

На всякий случай прикреплю скрин той страницы. Может так понятнее будет?
Аноним 01/03/17 Срд 18:47:42 #718 №12269 
>>12250
Это содержательная точка зрения. Если бы решение уравнения существовало, его можно было бы подставить в уравнение и убедиться, что оно действительно является решением. Это и будет доказательством его существования. Поэтому если доказательства не может быть в принципе, значит, решений нет.
Аноним 01/03/17 Срд 19:01:46 #719 №12271 
>>12240
Всяко знаете больше, чем я.
Аноним 01/03/17 Срд 19:07:12 #720 №12272 
>>12269
Не понял. Там нет никакого "если бы", там ведь четко оговаривается, что в случае если нельзя ни доказать, ни опровергнуть Q, то Q истинно. Но это же бредятина.
Аноним 01/03/17 Срд 19:14:47 #721 №12273 
blob
Здравствуйте. Учусь на программиста и имею дело только со второкультурной математикой, так что заранее извиняюсь.
Начали нам рассказывать про дифференциальные уравнения. И тут, значит, лектор начинает умножать обе части уравнения, скажем, на dx (что позволяет сокращать dx). Я, как нормальный белый человек, озадачился этим сокращением дифференциалов, ведь там нет дроби, зато есть предел.
Не буду кидать конспект, так что скину скрин с mathprofi, где такая же операция совершается
Короче, поясните, почему я/лектор долбаеб и, если считаете нужным, можете дать почитать об это в каком-нибудь серьёзном источнике
Аноним 01/03/17 Срд 19:23:02 #722 №12274 
Как обучают математике слепых?

https://www.youtube.com/watch?v=qRfOuOkm9hk
Аноним 01/03/17 Срд 19:24:27 #723 №12275 
>>12272
Речь о том, что есть, грубо говоря, две математики. Содержательная, основанная на смысле вещей, и формальная, оперирующая синтаксисом цепочек символов. Когда говорят "нельзя ни доказать, ни опровергнуть", имеется в виду именно вторая математика. Т.е. не существует цепочки символов, удовлетворяющей формальным синтаксическим правилам доказательства этого утверждения. Из этого ещё не следует, что мы не можем доказать его содержательно. Просто это содержательное доказательство никак нельзя выразить на формальном языке. Вербит говорит о том, что математику нельзя свести к формализму.
Аноним 01/03/17 Срд 19:44:15 #724 №12277 
>>12250
Ключевые слова - "исходя из аксиом Пеано". Исходить-то можно и из более мощных аксиом. Если бы существовал набор аксиом, не противоречащий аксиомам Пеано, но позволяющий получить решение x уравнения P, то мы могли бы доказать, что аксиомы Пеано противоречивы. Но они непротиворечивы, как доказано Генценом.
Аноним 01/03/17 Срд 19:46:28 #725 №12278 
>>12273
Существует такая вещь как дифференциал. dy/dx можно рассматривать не как цельный символ, а как реальную, всамделишную дробь из двух дифференциалов.
Аноним 01/03/17 Срд 19:47:20 #726 №12279 
>>12273
У Арнольда в его книге по ОДУ, есть пассаж в котором поясняется про это. Более глубоко, смотреть по словам дифференциальные формы.
Аноним 01/03/17 Срд 20:00:38 #727 №12281 
>>12279
Арнольд В.И. "Обыкновенные дифференциальные уравнения"?
Аноним 01/03/17 Срд 20:07:01 #728 №12283 
>>12281
Угу, там где-то в первые главах есть пояснение.
Аноним 01/03/17 Срд 20:08:07 #729 №12285 
>>12281
21 страница примерно
Аноним 01/03/17 Срд 20:10:00 #730 №12286 
>>12278
> можно рассматривать
Она и так есть реальная, всамделишная дробь.
Деление одной бесконечно малой, которая на самом деле некое число/функция которая просто умножается на бесконечно малую, на другую бесконечно малую.
Аноним 01/03/17 Срд 20:10:23 #731 №12287 
>>12283
>>12285
Да, спасибо, уже читаю и понимаю свою неправоту)
Аноним 01/03/17 Срд 20:10:29 #732 №12288 
>>12275
Это странно. А возможно же дополнить формальную систему таким образом, чтобы осуществить выражение данных суждений формальным методом?
Аноним 01/03/17 Срд 20:10:36 #733 №12289 
>>12286
>бесконечно малой
Как там в 18 веке?
Аноним 01/03/17 Срд 20:18:14 #734 №12290 
>>12288
Нет. Теорема Гёделя как раз об этом. Какую бы формальную систему (достаточно богатую) ты ни взял, найдётся утверждение, которое будет доказуемым с содержательной, но не с формальной точки зрения. Поэтому нет смысла дрочить на формализм - он заведомо ущербен. Его роль - в упрощении верификации доказательств, и не более того.
Аноним 01/03/17 Срд 20:39:43 #735 №12291 
>>12290
Это очень интересно. Спасибо за объяснение. Ноо... Еще хотелось бы познакомиться с самой теоремой Гёделя. Можете порекомендовать литературу для возможности осмыслить её?
Аноним 01/03/17 Срд 20:55:20 #736 №12292 
Анон, нужна помощь.
Пропустил одну лекцию по актуарной математике и упустил важный момент: как найти для функции дожития Lx=1-sin(n/200)x точное и округленное значение средней остаточной продолжительности жизни в возрасте 35.
В интернетах нахожу или не совсем то или ЯННП.
Аноним 01/03/17 Срд 21:04:11 #737 №12293 
>>12291
http://www.mathnet.ru/php/presentation.phtml?option_lang=rus&presentid=122
Аноним 01/03/17 Срд 21:07:02 #738 №12294 
>>12289
>буквы времен кирила и мефодия
00101101010101011 10101011001010 10101010101110101010011001 00010101100101010?
Аноним 01/03/17 Срд 21:31:58 #739 №12295 
14442196093090.jpg
>>12294
Аноним 01/03/17 Срд 22:09:50 #740 №12299 
У меня бывают проблемы с преобразованием выражений, типа выразить что-то, перекинуть из числителя в знаменатель и т.д. и т.п.
Где потренироваться в этом? Задания какие-нибудь.
Аноним 01/03/17 Срд 22:11:59 #741 №12300 
>>12299
Какого рода проблемы? Ты не знаешь, как это делать, или просто ошибки лепишь? Если второе, то просто делай по одному действию за раз. Помогает.
Аноним 01/03/17 Срд 22:15:50 #742 №12301 
>>12300
Ошибки. Ну и бывало не знал, что когда например
в уравнении (дробь = дробь) можно из знаменателя слева закинуть в числитель в правой части.
Аноним 01/03/17 Срд 22:25:48 #743 №12302 
Дайджест хороших, годных постов:

>>12214
>>12216
>>12222


>>12250
>>12269
>>12275
Аноним 02/03/17 Чтв 00:41:33 #744 №12305 
>>12295
Пернул чет с тебя.
Работающая конструкция с которой вышка резко сворачивается с двухтомника фихтенгольца до ожднотомника авторства хуева-кукуева говно только потому что старая.
Аноним 02/03/17 Чтв 01:02:40 #745 №12307 
NG6kZus3hx0.jpg
Есть один граф. Необходимо соединить подмножество вершин графа деревом, минимальным по весу - задача Штейнера. Подскажите алгоритм, с помощью которого можно решить задачу. Есть алгоритмы Краскала и Прима, но они не подходят, т.к. находят дерево, соединяющее ВСЕ вершины графа.
Аноним 02/03/17 Чтв 01:50:30 #746 №12310 
>>12273
Это одна из моих самых больших болей в институте, я разобрался только после окончания. Суть в том, что dx и dy -- они оба суть элементы одного и того же одномерного векторного пространства. Векторы делить друг на друга нельзя, но если пространство одномерное, они отличаются друг от друга умножением на скаляр. Так и здесь. dy/dx -- это скаляр, на умножение на который отличаются векторы dx и dy. Вот что это за векторное пространство, которому они принадлежат, - это уже вопрос более глубокий
Аноним 02/03/17 Чтв 03:04:04 #747 №12313 
Суп, как взять этот интеграл? Делал замену потом по частям, но что-то не работает.
Аноним 02/03/17 Чтв 03:08:47 #748 №12315 
IMG0165.JPG
>>12313
У меня двач сломался
Аноним 02/03/17 Чтв 03:15:35 #749 №12316 
>>12310
Поехавший?
Аноним 02/03/17 Чтв 03:41:16 #750 №12317 
>>12315
Никак. Он не берётся с элементарных функциях. Максимум что можно сделать - выразить его через функцию ошибок.
Аноним 02/03/17 Чтв 03:43:00 #751 №12318 
>>12315
Не уверен, что он берется в элементарных. попробуй сделать замену t^2 =9-x, а дальше уже по частям (да, это упорото) и через функцию ошибок экспоненту долбанную захерачить.
Аноним 02/03/17 Чтв 03:50:15 #752 №12319 
>>12316
Всё так. Дифференциальные формы - это в первую очередь тензоры.
Аноним 02/03/17 Чтв 04:24:21 #753 №12320 
>>12292
Спроси у одногрупников.
Аноним 02/03/17 Чтв 10:47:53 #754 №12323 
>>12319
Приехали.
Ты ПОНИмаешь что выражаешь довольно простое понятие очень сложными терминами? Те же бурбаки единицу выражали через хуеву тучу множест с подмножествами. ПРосто потому что могут.
Ты так же.
Аноним 02/03/17 Чтв 11:34:47 #755 №12325 
>>12323
>довольно простое понятие
Просто ты идиот. Оно только идиотам кажется простым. Им картинку показали, "линейная часть приращения" сказали, они всё типа поняли. А нормальные няши охуевают.
Уравнения и их системы Сереженька 02/03/17 Чтв 13:19:35 #756 №12326 
IMG20170302201427.jpg
Анон, выручай. Из меня математик-пират с хуем вместо шпаги. Реши 3 и 5 номер. Заранее спасибо. С меня фотки сёстры.
Аноним 02/03/17 Чтв 14:08:00 #757 №12327 
>>12326
Решать за тебя не буду, просто подскажу идеи. В третьей задаче нужно несколько раз воспользоваться теоремой Виета в обратную сторону. В пятой - зарабатывать ноль в правой части, после чего раскладывать на множители.
Аноним 02/03/17 Чтв 16:23:28 #758 №12330 
tG0vKLMOqY.jpg
Парни, помогите плиз
Нужно указать вид треугольника SCO и SOB
Найти AC, SO, SB, SA
Аноним 02/03/17 Чтв 16:39:49 #759 №12332 
Почему говорят, что квадратное уравнение в комплексных числах имеет ровно два корня? Ведь при D=0 будет только один корень.
Аноним 02/03/17 Чтв 16:58:18 #760 №12334 
Сука, что за скоординированный набег даунов? Все сессии уже кончились вроде.
Шлюха дымится из за отсутствия посещаемости модулей над кольцами?
Аноним 02/03/17 Чтв 17:02:32 #761 №12335 
>>12332
Два. Совпадающих, так получилось.
Аноним 02/03/17 Чтв 17:07:43 #762 №12336 
С точки В - центра правильного треугольника АВС - проведен перпендикуляр SO плоскости треугольника SO=4 см. Найти расстояние от точки S до сторон треугольника если сторона равна 6 корень из 3
Аноним 02/03/17 Чтв 17:16:52 #763 №12337 
>>12335
Так если они совпадают, то это один корень, не?
Аноним 02/03/17 Чтв 17:19:37 #764 №12338 
>>12334
А к егэ готовятся.
Аноним 02/03/17 Чтв 17:42:09 #765 №12340 
>>12336
>точки В - центра правильного треугольника АВС
Аноним 02/03/17 Чтв 18:19:25 #766 №12341 
portraitsofthemindp1772480x3508.jpg
>>10508 (OP)
Я биолог. Мне тяжело читать про мат аппарат, который нужен в работе (недавно встретил дельта-функцию Дирака, например). Тратить много лет на то, чтобы понять фундаментально математику и уметь доказывать теоремы что-то не хочется. Хочется просто быстро войти в курс дела и научиться использовать мат аппарат, интуитивно понимать его с точки зрения биофизики, с как можно меньшими временными затратами.
Мб кто-нибудь знает что-то подходящее? Чтобы мат аппарат объясняли на очень понятном языке как для дебилов, чтобы не было акцента на решение задач (компьютер сам всё посчитает), а именно на понимание, чтобы я был способен сформулировать задачу и/или понять то, что уже написано.
Аноним 02/03/17 Чтв 19:45:42 #767 №12349 
>>12341
Должен тебя разочаровать, но того, что ты хочешь, просто не существует
Аноним 02/03/17 Чтв 19:55:21 #768 №12351 
Кто-то может пояснить за гиперграф просто и понятно?
Аноним 02/03/17 Чтв 20:21:56 #769 №12352 
>>12351
На вики всё просто и понятно. Даже картинка красивая нарисована.
Аноним 02/03/17 Чтв 20:24:54 #770 №12353 
>>12349
Совсем? Даже на английскому? Немецком, японском?
Немного подумав, я смог сформулировал задачу более точно - математическое моделирование. Желательно с уклоном в электродинамику, совсем было бы круто, если бы нашлось что-то по моделированию нейронных сетей (не искусственных, а настоящих).
Аноним 02/03/17 Чтв 20:36:11 #771 №12355 
>>12349
если так, то почему не существует? Спрос на такое ведь есть.
Аноним 02/03/17 Чтв 20:59:13 #772 №12356 
>>12334
Тут нет капчи.
Аноним 02/03/17 Чтв 21:02:18 #773 №12359 
>>12341
https://www.ozon.ru/context/detail/id/2483767/
Полсекунды в гугле. Слабовато.
Аноним 02/03/17 Чтв 21:45:32 #774 №12360 
Странные вы тут какие-то люди. Не, я понимаю заниматься математикой (ну формулы поучить) если заставляют в вузе/школе, но чтобы дома, самому, в свободное время (когда можно выучиться делать что-нибудь действительно интересное и скилловое, типа научиться рисовать или писать музыку) вы сидите и находите интегралы или еще какой чухней занимаетесь. Не, я просто к тому, что когда вам лет 40-50 будет, вы оглянетесь, и явно подумаете мол на какую-же абстрактную чухню я потратил свою жизнь, лучше бы выучил нормальный скилл какой. Разве нет?
Аноним 02/03/17 Чтв 21:48:57 #775 №12361 
>>12360
> когда вам лет 40-50 будет, вы оглянетесь, и явно подумаете мол на какую-же мещанскую чухню я потратил свою жизнь, лучше бы гомологии считал. Разве нет?
Аноним 02/03/17 Чтв 21:51:00 #776 №12362 
>>12360
>научиться рисовать или писать музыку
>считать интегралы
Будто бы это не подобные вещи.
Аноним 02/03/17 Чтв 22:03:14 #777 №12363 
>>12353
> если бы нашлось что-то по моделированию нейронных сетей

Ну тут вообще хардкорная фундаментальная математика. Ты слишком многого хочешь. А так да, совсем.

>>12355
Потому что математика очень большая. И для освоения даже небольшого раздела этой науки, нужно будет много прочитать и во многом разобраться.
Аноним 02/03/17 Чтв 22:04:14 #778 №12364 
>>12360
>заниматься математикой
>формулы поучить
Ясно. Проходите, не задерживайтесь.
Аноним 02/03/17 Чтв 22:13:51 #779 №12365 
>>12360
Такое же развлечение/хобби, как и многие другие занятия. К слову, в школе на изо, музыку и физкультуру тоже заставляют ходить, у тебя же не возникает таких вопросов к любителям рисовать, петь или пробежаться по утрам.
Аноним 02/03/17 Чтв 22:18:17 #780 №12366 
>>12365
Ну дык на школьном изо заставляют рисовать какую-то чухню типо цветов, вмето йоба-монстров и мехов.
Аноним 02/03/17 Чтв 22:20:57 #781 №12367 
Как научиться решать задачки типа "Бригада из 57 работников построила Диснейлэнд за 42 часа. За сколько часов Диснейлэнд построит бригада из 126 работников?" и прочая из школьных учебников?

Сейчас я такие задачи решаю тупо перебором возможных действий и чисел что есть, при чем не самым оптимальным перебором.
Аноним 02/03/17 Чтв 22:20:58 #782 №12368 
mecha.jpg
>>12365
Вот смотри, разве ты не хотел бы научиться рисовать так же, как на пике? А ты мог бы, если бы год-полтора вместо задрачивания математики рисовал. В итоге ты может и понимаешь калкулус или еще чего, но такого охуенного робота не нарисуешь.
Аноним 02/03/17 Чтв 22:23:39 #783 №12369 
>>12353
Ну, я могу тебе подсказать, что надо гуглить "импульсные нейронные сети", "spiking london neural network"
Аноним 02/03/17 Чтв 22:24:44 #784 №12370 
>>12363
Нейроночки это же весьма прикладная математика.
Аноним 02/03/17 Чтв 22:25:24 #785 №12371 
>>12368
Какая хрупкая конструкция.
Аноним 02/03/17 Чтв 22:28:22 #786 №12372 
>>12368
А зачем мне рисовать робота?
Аноним 02/03/17 Чтв 22:30:00 #787 №12373 
>>12372
Чтобы выразить себя. Вот ты поиграешь в игру красивую какую, тебя разве не потянет порисовать локации к этой игре? К фильму?
Аноним 02/03/17 Чтв 22:31:45 #788 №12374 
maxresdefault (2).jpg
>>12368
Вот смотри, разве ты не хотел бы научиться считать гомологии так же, как на пике? А ты мог бы, если бы год-полтора вместо задрачивания рисования изучал математику. В итоге ты может и можешь нарисовать робота или еще чего, но такие охуенные гомологии никогда не посчитаешь.
Аноним 02/03/17 Чтв 22:33:10 #789 №12375 
>>12366
Ну дык потому и бесит многих. Та же история с математикой/историей/биологией/вставитьтекст.
Аноним 02/03/17 Чтв 22:33:33 #790 №12376 
А вы вот смотрите все эти символы... О чем вы думаете? Тупо читаете "для всех x где a блаблабла" или/и представляете что-то особое и интересное?
Аноним 02/03/17 Чтв 22:34:05 #791 №12377 
>>12370
>Нейроночки
Уёбывай в /зк, пидор. Он про
>что-то по моделированию нейронных сетей (не искусственных, а настоящих).
Аноним 02/03/17 Чтв 22:34:16 #792 №12378 
>>12375
Да историю с биологией вообще можно на ходу сочинять.
Аноним 02/03/17 Чтв 22:34:44 #793 №12379 
>>12378
Вы не правы, всего доброго.
Аноним 02/03/17 Чтв 22:34:45 #794 №12380 
>>12377
Ну, импульсные нейроночки, чаго там
Аноним 02/03/17 Чтв 22:36:05 #795 №12381 
hqdefault.jpg
>>12380
>нейроночки
>нейроночки
>нейроночки
Аноним 02/03/17 Чтв 22:37:46 #796 №12382 
>>12373
Это всё прикольно, пока не прокачаешь мозг математикой - и тогда оно всё становится неинтересным.

Хотя я, например, очень редко играю в старые игры времён детства. Всё равно приятно, но задрачиваться с созданием локаций я никогда не буду, потому что это не абстрактное занятие.

Ещё я фильмы смотрю, тоже жвачка для мозга. ТОже иногда такую хрень бездарную увижу - сразу хочется бежать задачи решать, вместо разбазаривания времени.

Видимо мой мозг устаёт от ОЧЕНЬ абстрактных задач?
Жалко, некоторые могут дольше, а я преждевременно заканчиваю.

Где-то от 0 до-10 маленьких теорем в день.
Ну ещё здоровье накладывает отпечаток - так бы не отрывая фигачил бы вплоть до потери здоровья.
Аноним 02/03/17 Чтв 22:40:27 #797 №12383 
>>12368
>>12374
Кстати, он же тебе и ответит.
https://www.youtube.com/watch?v=B2VGmNeNGKo
Аноним 02/03/17 Чтв 22:42:29 #798 №12384 
>>12360
Толсто, надеюсь, /math не до такой степени деграднул, чтобы на это повестисть.
Аноним 02/03/17 Чтв 22:45:14 #799 №12385 
>>12368
Нет, поскольку в школе я увлекался рисованием и понимаю, сколько времени ты угробишь на абсолютно тупую деятельность по вырисовыванию пропорций без какого-либо задействования межушного ганглия.
А вот матан и мозг поддерживает в тонусе (и это не общие слова, а нейрофизиология), и ЧСВ повышает. Чтобы нарисовать что-то, нужно тупо задротствовать, а вот чтобы в матан мочь - это реальный труд, который не каждому по зубам.
Аноним 02/03/17 Чтв 22:45:51 #800 №12386 
>>12385
> и ЧСВ повышает
Вот с этим не поспоришь, аж фонит.
Аноним 02/03/17 Чтв 22:53:42 #801 №12387 
>>12374
О там PHP написано
мимо программист
Аноним 02/03/17 Чтв 22:53:43 #802 №12388 
А что вообще в голове у математиков, когда видят разные символы? Есть, конечно и сухой текст к этим символам "для всех a... блаблабла", но может вы видите там цветных пони которые обнимаются и скрываете это от людей?
Аноним 02/03/17 Чтв 22:54:55 #803 №12389 
>>12387
PHP=2P, пэхапэ равно тупэ, для тупых значит.
Аноним 02/03/17 Чтв 22:55:31 #804 №12390 
>>12359
Эта книга у меня есть, я её читал и там ни слова нет про моделирование популяций нейронных сетей. Не пидорских искусственных сетей, а настоящих, с потенциалами действия, с калиевыми/натриевыми каналами, с порогами деполяризации, с дендритными шипиками, с ёмкостью и пр биофизикой. Вот начал я читать учебник по этому на англе, наткнулся на дельта функцию Дирака, ничего не понял. В книге сказали посмотреть аппендикс. Посмотрел, прочитал формальное определение, но как применять эту функцию и в чём заключена её суть(тм) так и не понял (то есть я не смогу свободно применять этот мат аппарат, когда он мне понадобиться где-то в другом месте, а хотелось бы), пошёл в википедию, понял, что определений много, заодно узнал, что функция каким-то образом описывает распределение величины в пространстве, в том числе зарядов (заодно понял, что это круто и очень полезно для того, чем занимаюсь). Но так чтобы понял суть - до сих пор нет этого.

>>12369
Вот за это спасибо,не знал, буду разбираться.
Аноним 02/03/17 Чтв 22:56:51 #805 №12391 
>>12387
>PHP
>программист
Спешите видеть, иакака возомнила себя человеком
Аноним 02/03/17 Чтв 22:56:58 #806 №12392 
>>12389
Ну правильно

Аноним 02/03/17 Чтв 22:57:19 #807 №12393 
>>12391
Я не пхпшник
Аноним 02/03/17 Чтв 22:57:31 #808 №12394 
>>12390
Ну так это полсекунды в гугле, наверняка если побольше потратить, то можно найти нормальную книгу от биолога для биолога. В твоей области же есть спецы-биологи, которым уже пришлось обмазаться математикой, часть из них наверняка написала книги.
Аноним 02/03/17 Чтв 22:57:36 #809 №12395 
14862381604440.jpg
Аноним 02/03/17 Чтв 22:58:13 #810 №12396 
>>12391
Так последние новости нейробиологии как-раз в том, что макаки получили самосознание и проходят зеркальный тест.
Аноним 02/03/17 Чтв 22:58:43 #811 №12398 
>>12374
Ну а какой кайф в этом? Неужели от этого можно получить такое же удовольствие как от рисования? Когда человек рисует ему блядь миры воображаемые подвластны, он может придумать свою культуру, свое население им созданной планеты, окружение этой планеты нарисовать. А вычисляя гомологии такой кайф можно получить?
Аноним 02/03/17 Чтв 22:59:08 #812 №12399 
>>12383
Бля, этоо правда, что он тут говорит про научную карьеру?
https://youtu.be/bbqYl54Uetw?t=241
Аноним 02/03/17 Чтв 23:00:05 #813 №12401 
>>12398
>ему блядь миры воображаемые подвластны
А математике подвластен реальный мир. Прикинь какой кайф?
Аноним 02/03/17 Чтв 23:00:43 #814 №12402 
>>12398
Можно гораздо больший кайф. Твои планеты нах никому не всрались. А твою математику, если она годная, обязательно прочитают.
Аноним 02/03/17 Чтв 23:00:52 #815 №12403 
>>12398
Конечно. Математики вместо гомологий на самом деле видят бесконечномерных цветных пони и прочие удивительные вещи, а для непосвященных это лишь кучка символов.
Аноним 02/03/17 Чтв 23:02:25 #816 №12405 
>>12398
https://www.youtube.com/watch?v=044VwC_uptU
Попали.
Аноним 02/03/17 Чтв 23:04:51 #817 №12406 
А почему гуманитарии не дружат с математикой? Ведь, по сути, и там язык, и там язык.
Аноним 02/03/17 Чтв 23:05:17 #818 №12407 
>>12402
>Твои планеты нах никому не всрались
Концепт-художники создают все эти охуенные виды планет и космических кораблей в современных сай-фай фильмах. Да и во всех фильмах они принимают активное участие, а так же в играх. Фильмы/игры смотрят/играют миллионы блядь людей МИЛЛИОНЫ НАХУЙ!
Аноним 02/03/17 Чтв 23:05:31 #819 №12408 
>>12398
https://www.youtube.com/watch?v=l8tGfLcn3i0
А вот тут он загоняет, что сначала появились уравнения и потом лет через 40 появились физические объекты, описываемые ими. Типа может статься, что математики создают этот мир, как художники создают свои миры и тд
Аноним 02/03/17 Чтв 23:10:30 #820 №12409 
>>12407
"Концепт-художником" станут лишь единицы из миллионов. Поэтому сравнивать их нужно не с обычными математиками, а с топовыми звёздами уровня Ньютона или Тьюринга, буквально с создателями нашей цивилизации.
Аноним 02/03/17 Чтв 23:10:57 #821 №12410 
>>12394
Биологи, особенно в РФ, занимаются либо выращиванием говна на дрожжах, либо структуру протеин считают, в лучшем случае сигнальные каскады. Нейросцаенс в РФ находится в полумёртвом состоянии, поэтому даже спросить особо не у кого. Даже мои преподаватели занимаются не этим, а обмазываются статистикой на матлабе и больше занимаются с анализом ЭКГ или в лучшем случае спайковой активности отдельных клеток. Так, чтобы они пытались их моделировать - нет. Поэтому приходится самому инфу по крупицам собирать. Я тупил, потому что не знал как запрос сформулировать.
Аноним 02/03/17 Чтв 23:14:12 #822 №12411 
>>12410
> а обмазываются статистикой на матлабе
Стак мо лэерс?
Аноним 02/03/17 Чтв 23:17:00 #823 №12412 
>>12407
О чём ты пытаешься спорить? Есть жизнь, есть тысячи занятий, есть личность, чьё сознание является суперпозицией всего, происходящего с ней до этого, есть вкусы этой личности и есть выбор одного и тысяч занятий, основанный на вкусах этой личности, основанных на мне лень писать. Ты хочешь поспорить о вкусах или что?
Аноним 02/03/17 Чтв 23:20:20 #824 №12414 
>>12399
Да, правда. Впрочем, это верно почти везде, где есть социальные мартышки.

мимосцаентист.
Аноним 02/03/17 Чтв 23:21:59 #825 №12416 
14784235474340.jpg
Вот смотрите, говорят, что комплексные числа можно представить в виде вектора на плоскости, и с ними можно выполнять действия так же, как и с вектором. Но как понимать умножение через это? Если я представлю эти числа как векторы и перемножу их, то я получу по идее вектор ортогональный плоскости. Но при их перемножении получается вектор в той же плоскости. И в чем смысл тогда? Даже скалярное умножение не работает, модули перемножаются без косинуса. В чем же тогда смысл представления их в виде векторов? Может все-таки есть нормальное истолкование этого?
Аноним 02/03/17 Чтв 23:27:51 #826 №12418 
>>12388
Ну бывает, например, геометрическая интуиция: это когда представляешь картинки. Тут все понятно. Бывает, когда за одними символами представляешь другие более привычные для тебя. Скажем, когда изучаешь евклидовы кольца, можешь думать про кольцо целых чисел и кольцо многочленов. Когда привыкаешь к евклидовым кольцам, так делать перестаешь. Например, детсадовец, когда думает о натуральных числах, может представлять счетные палочки или яблоки, а взрослый человек так уже не делает, потому что к натуральным числам привык если это не N-петух.
Аноним 02/03/17 Чтв 23:30:15 #827 №12419 
>>12418
>а взрослый человек так уже не делает, потому что к натуральным числам привык если это не N-петух.
Или если он не ко-ко-консруктивист. А ведь он как раз про палочки какие-то говорил.
Аноним 02/03/17 Чтв 23:30:41 #828 №12420 
>>12416
Умножение не стоит так интерпретировать.
Аноним 02/03/17 Чтв 23:39:31 #829 №12421 
>>12416
Модули перемножаются, аргументы складываются, что тут ещё толковать. Скалярное произведение, кстати, вполне выражается на языке алгебры комплексных чисел. Первый вектор умножаешь на сопряжённый ко второму, и берёшь вещественную часть. А если возьмёшь мнимую часть, то вместо этого получишь площадь параллелограмма, натянутого на эти два вектора. Некоторые теоремы с использованием комплексных чисел доказываются очень легко и элегантно.
Аноним 02/03/17 Чтв 23:42:53 #830 №12422 
xIZauvU584.jpg
Суп, поехавшие.
Посоветуйте годный калькулятор с большими возможностями, желательно на андроид. Но можно и пк.
Заранее благодарю
Аноним 02/03/17 Чтв 23:43:47 #831 №12423 
>>12416
>>12420
Умножение на re геометрически можно проинтерпретировать так: модуль вектора умножается на r и вектор поворачивается на угол φ.
Аноним 03/03/17 Птн 00:05:14 #832 №12424 
>>12341
Я вот тоже биолог правда эволюционной биологией занимаюсь, но, как тебе уже сказали, ты слишком много сразу хочешь, учить придется прилично, если по-хорошему. Я потратил года два, чтобы научиться самому делать модельки в своей области, правда они тут одни из самых сложных в биологии вообще или может я просто не слишком умный.
Вот мой пост >>12222 выше был, я именно так и начинал в первые пол года изучения. На том же MIT OpenCourseWare в курсе по ODE есть, например, и про дельта-функцию, и про то, как решать уравнения с ней. Более того, там почти нет формализма, а больше упор на интуитивное объяснение. Попробуй, мне очень понравилось и помогло, но изучение математики реально занимает достаточно много времени.
Если все же хочешь начать ее учить и у тебя есть вопросы, то задавай.
Аноним 03/03/17 Птн 00:06:41 #833 №12425 
>>12416
https://betterexplained.com/articles/a-visual-intuitive-guide-to-imaginary-numbers/
Аноним 03/03/17 Птн 13:40:00 #834 №12433 
>>12424
Спасибо, про матпрофи знал, а курс от MIT наверну.
Аноним 03/03/17 Птн 15:54:20 #835 №12436 
Как посчитать сумму прогрессии S=4/(1,2^1)+4/(1,2^2)+...+4/(1,2^n)? 4 и 1,2 просто для примера.
Аноним 03/03/17 Птн 15:59:44 #836 №12437 
Зачем нужна производная? В каких расчетах она используется ирл?
Аноним 03/03/17 Птн 16:02:27 #837 №12438 
для себя себя кое-что решаю. есть 4 числа - на углах квадрата, высоты. центральная вычисляется, понятно, ц=(a+b+c+d)/4, лежащие на сторонах по прямой от центра, например, (ц*2+a+b)/4 - так можно определить все точки, но как произвольную? и вообще, верно ли это, наверно я какую-то хуйню себе в голове накрутил.
Аноним 03/03/17 Птн 16:11:41 #838 №12439 
а не, не обращайте внимание. сам не понял что я хочу. как называется это хрень, когда вычисляешь что-то в точках, в разных пространствах, например? может почитал бы что.
Аноним 03/03/17 Птн 16:48:08 #839 №12440 
>>12436
Прогрессия называется геометрической. Формула её суммы гуглится.
>>12437
Во всём, что имеет отношение к физике. Все физические законы записываются в терминах производных.
Аноним 03/03/17 Птн 17:02:41 #840 №12441 
>>12440
А почему в школке на физике не используем производные?
Аноним 03/03/17 Птн 17:10:51 #841 №12442 
>>12441
Даже в быдло школах их используют, не ври.
Аноним 03/03/17 Птн 17:17:21 #842 №12443 
>>12442
Ну на уроках матеши, не на физике же.
Аноним 03/03/17 Птн 17:19:48 #843 №12444 
>>12441
Потому что физика начинается с 7-го класса, а производные с интегралами проходят в 11-м, и то без теории, на уровне магических пассов. Это ничем не лучше, чем просто дать готовые формулы для частных случаев.
Аноним 03/03/17 Птн 17:34:28 #844 №12446 
>>12443
Нет, на физике.
Аноним 03/03/17 Птн 18:38:48 #845 №12459 
>>12440
Геометрическую я нагуглил. Но там знаменатель 1-q, а у меня степень меняется. Возможно, для моего примера нужно просто нулевой член прогрессии вычесть?
Аноним 03/03/17 Птн 18:59:34 #846 №12461 
>>12459
Где же он меняется, лол? Он равен 1,2.
Аноним 03/03/17 Птн 19:26:43 #847 №12462 
>>12461
У меня знаменатель 1,2^i то есть возрастает на 1 в каждом следующем члене. В проблеме своей я, вроде, разобрался. Там нужно вычесть из суммы нулевой член, т. к. у меня в примере его нету.
Аноним 03/03/17 Птн 19:28:58 #848 №12463 
>>12462
Внимательно прочитай, что такое геометрическая прогрессия, и что такое q в ней.
Аноним 03/03/17 Птн 19:41:51 #849 №12466 
Треды по математике убрали из /sci. Получается матеша - не наука?
Аноним 03/03/17 Птн 20:09:58 #850 №12468 
>>12466
Получается так.
Аноним 03/03/17 Птн 20:25:10 #851 №12470 
>>12466
>>51
посоны помогайте  вопросы для умных 03/03/17 Птн 23:16:28 #852 №12475 
Один известный математик, публикуя очередной объемный труд, обозначил первые главы в книге цветами. А с какого-то места продолжил нумеровать их числами, как это делается обычно. О чем была книга? И почему он так поступил?
распишите поподробнее
Аноним 03/03/17 Птн 23:27:37 #853 №12476 
>>12475
У меня есть ответ. Но откуда это?
Аноним 03/03/17 Птн 23:30:29 #854 №12477 
>>12476
задачку задал хитрый дядька, сказал кто отгадает тот молодец.
так что источник-некий хитрый дядька
Аноним 03/03/17 Птн 23:33:12 #855 №12478 
>>12477
То есть отгадаю я, а молодец будешь ты?
Аноним 03/03/17 Птн 23:34:55 #856 №12479 
>>12478
я честно сказал как есть. еще и спасибо скажу, могу еще помолиться если надо.
Аноним 03/03/17 Птн 23:40:50 #857 №12480 
>>12479
Подобные загоны были у Э. Ландау. Он написал учебник по теории чисел. В этом учебнике ему приходилось использовать нумерацию теорем числами. Однако достаточное количество теорем, чтобы определить число 1, у него набиралось далеко не сразу, а нумерация была нужна уже с первой же теоремы. Поэтому Ландау оправдывался, что номера, которые он использовал, не являются числами, но являются лишь ярлычками, и что он мог бы именовать теоремы вместо "теорема 1", например, "темно-синяя теорема" или "серо-зеленая теорема". So, ответ. Возможно, труд был по основаниям, а автор использовал для нумерации глав цвета потому, что определял числа сильно не в первой главе и не хотел пользоваться тем, что ещё не определено.
Аноним 03/03/17 Птн 23:43:51 #858 №12481 
>>12480
большое спасибо, мил человек. завтра отвечу правильно или нет. свечку ставить?
Аноним 03/03/17 Птн 23:46:15 #859 №12482 
>>12481
Ставь. Но лучше детали запили, завтра. Мне же интересно теперь.
Аноним 04/03/17 Суб 00:12:50 #860 №12483 
Айо, математики. Узнал о дифференциале функции, но не понял что это за штука такая. Так немного почитал интернет и встретил одну мысль: "дифференциал функции - это кароч когда график кривой приводим к маленьким маленьким прямым и все такое". А разве производная это не тоже самое?
Аноним 04/03/17 Суб 00:39:02 #861 №12484 
>>12483
1. Не всякая функция является линейной. Но некоторые функции могут быть локально приближены линейными. Они называются дифференцируемыми. Дифференциал функции f в точке x - это линейный оператор, значения которого близки к f в окрестности x. 2. В пространстве R^n имеется канонический базис (единичный). При выборе базиса всякий линейный оператор биективно соответствует некоторой матрице. Дифференциал - линейный оператор, поэтому он тоже соответствует матрице. В каноническом базисе эта матрица называется производная функции f в точке x. В случае R^1 эта матрица вырождается в одно-единственное число.
Аноним 04/03/17 Суб 01:27:11 #862 №12485 
>>12484
Нахуя человеку, только что узнавшему о дифференциале, твои линейные операторы?
Аноним 04/03/17 Суб 01:28:33 #863 №12486 
Miy2L3vJUQ.jpg
>>12485
Аноним 04/03/17 Суб 03:12:22 #864 №12487 
Безымянный.png
У меня пригорело. Не могу решить.
Аноним 04/03/17 Суб 04:37:33 #865 №12488 
>>12483
Допустим, у тебя есть некая функция. Ты выбираешь точку и проводишь в ней касательную к графику. В небольшой окрестности точки эта касательная хорошо приближает функцию. Так вот, ты интересуешься вопросом, насколько сильно поменяется функция, если из выбранной точки куда-нибудь чуть-чуть сдвинуться. Ты можешь буквально взять разность значений функции в разных точках (это называется "приращением функции"), но это может быть сложно посчитать и ещё сложнее проанализировать, если функция хитрая. Но у тебя есть касательная. Она представляет собой линейную функцию, и с ней работать всяко проще. А в небольшой окрестности исходной точки, как уже сказано, она хорошо приближает функцию, так что сильно ты не наврёшь, если не будешь уходить далеко. Так вот, если ты вычисляешь приращение не по самой функции, а по касательной - это и есть дифференциал. А производная - это угловой коэффициент касательной. Т.е. да, эти вещи очень сильно связаны, но всё-таки не одно и то же.
Аноним 04/03/17 Суб 04:39:32 #866 №12489 
CodeCogsEqn.gif
>>12487
Возможно уже поздно и я творю херню, но вышло так. Дальше просто составляешь функцию Лагранжа для оптимизации f при условии фи, а там уже на стадии нахождения частных производных Лагранжиана получаешь, что косинусы двух других углов равны. Следовательно, треугольник равнобедренный.
Аноним 04/03/17 Суб 04:47:13 #867 №12490 
>>12489
Что-то мне стыдно, что я сразу о теореме синусов не подумал. Уже чуть было не полез координаты вводить.
мимо
Аноним 04/03/17 Суб 04:54:17 #868 №12491 
>>12490
Я просто кроме теоремы синусов и теоремы Пифагора больше ничего не помню толком из школьной геометрии.
Аноним 04/03/17 Суб 05:51:26 #869 №12492 
>>12486
Соседей разбудил.
Аноним 04/03/17 Суб 10:35:28 #870 №12493 
>>12486
Это не ответ на вопрос.
Ты простыми словами можешь объяснить или нет?
Аноним 04/03/17 Суб 10:45:25 #871 №12494 
>>12483
>"дифференциал функции - это кароч когда график кривой приводим к маленьким маленьким прямым и все такое"
Нет, это определение дифференцируемости функции в точке. На очень больших масштабах большинство функций можно с разной степенью грубости заменить прямыми линиями вместо кривых/дуг.
В пределе мы заменяем ВСЮ функцию в выбранной области/участке/там где нам нужно, на такие мелкие отрезки. Можно даже сказать что после этой всей замены на отрезки, мы выбираем самый понравившийся нам отрезок который превращаем в прямую и которая прямая в смысле будет в дальнейшем называться касательной.

А дифференциал это уже линейное приближение потому то рассчитывается по линии, которая вычисляется по производной и называется касательной функции как написал вот этот >>12488 гротендрик.
Аноним 04/03/17 Суб 10:47:44 #872 №12495 
1488610175245.jpg
Каждый раз когда вижу этот знак, до смерти пугаюсь. Есть какие-нибудь ссылки, где ВСЕ про суммы расписано?
Аноним 04/03/17 Суб 10:50:46 #873 №12497 
>>12495
Что значит все? Ты не знаешь как выразить 1+1 или что? Тем более что в большинстве случаев считают только первые пару штук/десятков максимум членов и уже получают нужный результат.
Аноним 04/03/17 Суб 10:58:30 #874 №12498 
>>12497
2
Е1
1
так что ли? Действительно не знаю
Аноним 04/03/17 Суб 11:26:00 #875 №12499 
>>12493
Я объясню за него. Алгебра приблизительно в 20 раз проще/легче/понятнее анализа. Однако, в силу ряда причин, некоторые люди, включая тебя, немного знают анализ, но абсолютно не знают алгебры. Это калеки, убогие, прокажённые, несчастные люди.
Плохо здесь не это, а то, что ты пытаешься навязать своё убожество другим, заставляя их учить анализ до/вместо алгебры.
Иди нахуй, говно.
Аноним 04/03/17 Суб 11:27:04 #876 №12500 
Есть некоторое количество чисел типа: 1057, 45, 12, 4579, 51, 477 и т.д. (или любые другие).
Как математически грамотно записать, что 12 (или другое наименьшее) меньше всех остальных?
04/03/17 Суб 12:40:53 #877 №12502 
>>12500
12=min{1057, 45, 12, 4579, 51, 477}
Аноним 04/03/17 Суб 13:10:20 #878 №12504 
>>12502
А если не указывать конкретных чисел, для общего случая, так как числа могут быть разными?
04/03/17 Суб 13:15:24 #879 №12505 
>>12504
A - множесnво чисел
s=minA
Аноним 04/03/17 Суб 13:26:13 #880 №12506 
>>12505
сенкс
Аноним 04/03/17 Суб 13:56:21 #881 №12510 
>>12499
>Иди нахуй, говно.
Какая мощная аргументация. Даже и не знаю как с таким поспорить.
Аноним 04/03/17 Суб 15:00:29 #882 №12511 
>>12510
А чего ты ожидал от алгебраического сектанта, уёбки свою санину всюду суют, забывая о том, что у большинства математиков она вызывает отторжение.
Аноним 04/03/17 Суб 15:10:38 #883 №12512 
>>12510
Попробуй прочитать остальной пост. Почему ты только оскорбления услышал, а остальное проигнорировал? Ты не мазохист случаем?
Аноним 04/03/17 Суб 15:35:24 #884 №12513 
>>12512
Ну давай разберем по частям, написанное.
Складывается впечатление что писал реально контуженный , обиженный жизнью имбецил

>локально приближены
Что такое локально приближена?
>линейный оператор
Что такое оператор и почему он линейный?
>канонический базис (единичный)
Что такое базис и почему он единичный?
>биективно соответствует некоторой матрице
Что кому соответствует и почему?

Могу приехать к написавшему и в глаза сказать "Вся та хуйня тобою написанная это простое пиздабольство". Он готов выслушать?
Аноним 04/03/17 Суб 15:55:24 #885 №12515 
>>12513
А я откуда знаю? У него и спрашивай.
Аноним 04/03/17 Суб 16:07:01 #886 №12517 
Это действительно так, логарифмы - самый запутанный из элементов программы школьной математики?
Аноним 04/03/17 Суб 16:08:09 #887 №12518 
>>12517
Самый полезный для дальнейшего обучения, скорее.
Аноним 04/03/17 Суб 16:08:37 #888 №12519 
>>12518
Чем?
Аноним 04/03/17 Суб 16:24:03 #889 №12520 
>>12519
Тем, что очень часто применяется в математике, науке.
Аноним 04/03/17 Суб 16:25:38 #890 №12521 
>>12519
Связью с теорией групп и комплексным анализом, к примеру.
Аноним 04/03/17 Суб 16:28:11 #891 №12522 
>>12517
непривычный просто. Это те же самые степени только записанные задом наперед.
Аноним 04/03/17 Суб 16:39:36 #892 №12523 
>>12522
Это я и так понимаю. Просто там много путаниц в решении всяких сложных примеров. Нужно насквозь понять все взаимосвязи и перевороты логарифмов чтоб не смотреть как осел на очередную непонятку.
Аноним 04/03/17 Суб 16:41:23 #893 №12524 
>>12523
Потому что тебе его преподают как дебилу, без использования комплексных чисел. Сам предмет не при чем, абсолютно.
Аноним 04/03/17 Суб 16:43:41 #894 №12525 
>>12524
Я сейчас серией решебнико-задачников А.Х.Шахмейстера латаю все собственные пробелы. Там логарифмы - 8я книга, а комплексные числа - 14я.
Аноним 04/03/17 Суб 16:46:01 #895 №12526 
>>12525
А надо было решать "тригонометрию" Гельфанда-Львовского и "теорему Абеля" Алексеева.
Аноним 04/03/17 Суб 17:35:07 #896 №12527 
>>12513
>Что такое оператор и почему он линейный?
Ой всё.
Аноним 04/03/17 Суб 18:41:03 #897 №12531 
14839634678113.jpg
Господа, объясните смысл задачи:

"Запиши двузначное число при делении единиц которого на 6 применяется равенство 12:6=2"

какой тут принцип ? Чо им надо от ребенка?
Аноним 04/03/17 Суб 19:15:56 #898 №12533 
>>12531
12
Аноним 04/03/17 Суб 19:57:59 #899 №12534 
пиздец чашки коммутативность.png
>>12531
В душе не ебу. Педагогика - лженаука пополам с шизофренией.
Аноним 04/03/17 Суб 19:58:45 #900 №12535 
>>12525
Не слушай этого >>12526 дауна.
Тебе нужно просто привыкнуть к такой записи степеней и все будет ПУЧКом.
Аноним 04/03/17 Суб 19:59:36 #901 №12536 
>>12534
Учебник от рыбникова?
Аноним 04/03/17 Суб 20:02:38 #902 №12537 
01a275f807ed3edL.jpg.jpg
f7d403e6363302af2ffad9da806f794e.jpg
>>12536
Нет, новая методичка для российских школ. Профессор Анна Витальевна Белошистая отменила в России коммутативность умножения.
Аноним 04/03/17 Суб 20:04:59 #903 №12538 
>>12481
>завтра отвечу правильно или нет.
Ихде.
Аноним 04/03/17 Суб 20:47:36 #904 №12540 
А какую ОС вы используете?
Аноним 04/03/17 Суб 21:08:46 #905 №12541 
>>12535
Порекомендовал книг с нормальным изложением – даун. Откуда лезете только.
>>12537
Не знаю как в россии, но в аксиомы кольца коммутативность начиная с 1920 не входит.
Вообще скатили раздел, совсем, сволочи.
Аноним 04/03/17 Суб 22:26:43 #906 №12545 
>>12537
В общем-то, всё правильно. Коммутативность ещё доказать надо. Детей правильно приучают к тому, что её может не быть, и вообще разные операнды могут иметь разный смысл и вообще разную природу. Первый операнд - объект, второй операнд - его кратность.

Я во втором классе не очень-то верил в коммутативность умножения. Меня даже прямоугольники не убеждали. Потом я придумал для себя доказательство и успокоился.
Аноним 04/03/17 Суб 22:29:35 #907 №12546 
>>12537
Вот только она не отменяла коммутативность умножения. Там же русским языком все написано, почему ты не читаешь. И вообще, уже был срач про это.

>>12531
Так, двузначное число. Единицы, ну то есть правая цифра. На шесть. То есть число вида ху, у/6 = чему-то. Применяется равенство 12/6=2. Что значит применяется? Прменяется при делении единиц. Хм. Может они системы счисления там проходят? И нужно записать любое число, у которого в единицах цифра, равная 12 в десятичной системе? Нет?
Аноним 04/03/17 Суб 22:31:10 #908 №12547 
>>12545
Какое доказательство ты придумал, расскажи?
Аноним 04/03/17 Суб 22:41:24 #909 №12548 
>>12547
Ну тип у нас есть n кучек по m яблок. Мы из каждой кучки берём по одному яблоку и складываем их в новую кучку. Всего мы так можем сделать m раз, и в результате получаем m кучек по n яблок. Я сразу же понял, что по сути это почти то же самое, что нарисовать прямоугольник m на n и перевернуть его, но конкретный алгоритм перекладывания был как-то более убедителен для моего детского здравого смысла
Аноним 04/03/17 Суб 22:52:36 #910 №12549 
>>12548
Дай угадаю: потом ты стал конструктивистом? Вот, даже дети чувствуют, что убедительно, а что подозрительно похоже на черную магию!
Аноним 04/03/17 Суб 22:54:08 #911 №12551 
>>12549
Лолнет. Хотя аксиома выбора мне всё ещё не очень нравится.
Аноним 04/03/17 Суб 22:56:00 #912 №12552 
>>12551
Пусть у тебя есть множество непересекающихся кучек, возможно бесконечное. Из каждой кучки ты можешь взять элемент и сложить из них новую кучку, правда?
Аноним 04/03/17 Суб 22:58:46 #913 №12553 
>>12552
Из конечного могу. Из счётного тоже могу, наверное, если мне дать бесконечно много времени. А вот с несчётными уже не факт.
Аноним 04/03/17 Суб 22:59:52 #914 №12554 
>>12553
Ну почему же не факт?
Аноним 04/03/17 Суб 23:02:51 #915 №12555 
>>12554
Есть кучки, до которых я не доберусь никогда, сколько бы времени ни потратил.
Аноним 04/03/17 Суб 23:22:40 #916 №12556 
>>12534
Имеется ввиду, что при такой записи ребёнок привыкает понимать бэкграунд за числами/формулами, не просто 10 , а 10 кусков сахара. На самом деле годная техника педагогическая, зря ты так на него наехал.
Аноним 04/03/17 Суб 23:27:50 #917 №12557 
>>12499
Комбинаторика, дискретка, числа. Там все тайны.
Аноним 04/03/17 Суб 23:33:03 #918 №12558 
5835c22ce38dc.jpg
Котаны, поясните пожалуйста за пикрелейтед. Как, например, 16 должно превратиться в в 8 в степени четыре третьих, а 27 в 9 три вторых? Не улавливаю сути.
Аноним 04/03/17 Суб 23:39:29 #919 №12559 
gif.latex.gif
>>12558
Аноним 04/03/17 Суб 23:41:27 #920 №12560 
>>12541
>>12545
А ничего что сложение это краткая запись суммирования одинаковых чисел некоторое число раз?
Аноним 04/03/17 Суб 23:42:36 #921 №12561 
>>12556
Ты дурачок или физику прогуливал? Приписываем к каждому числу размерности, куски в стакане умножить на стаканы даст обычные куски.
Аноним 04/03/17 Суб 23:44:22 #922 №12562 
photo2017-02-0919-21-47.jpg
>>12560
>сложение это краткая запись суммирования
А ты хорош.
Аноним 04/03/17 Суб 23:44:59 #923 №12563 
>>12562
Умножение, жы
Аноним 04/03/17 Суб 23:46:45 #924 №12564 
>>12563
И в чём тогда проблема?
Аноним 04/03/17 Суб 23:49:00 #925 №12565 
В шараге скоро дифф. уравнения начнутся. Что почитать?
Аноним 04/03/17 Суб 23:54:46 #926 №12566 
>>12565
КНижки по диффурам, очевидно же.
Аноним 05/03/17 Вск 00:56:26 #927 №12567 
>>12566
>КНижки по диффурам, очевидно же.
Уж не Арнольда ли? Посоветовал бы чего годного тогда уж.
Аноним 05/03/17 Вск 01:42:50 #928 №12568 
>>12559
Кажется, что-то начинает проясняться. А можно еще раз словами?
Аноним 05/03/17 Вск 01:59:56 #929 №12569 
>>12568
Да тут и слов особо не вставишь, обычные операции со степенями. Возвести в степень ab - это всё равно, что возвести сначала в степень a, а потом результат - в степень b. Возвести в степень 1/3 - всё равно что извлечь корень третьей степени. Отсюда всё следует.
Аноним 05/03/17 Вск 02:05:32 #930 №12570 
>>12555
Ты напоминаешь Зенона сейчас, у него тоже континуум непроходим.
Аноним 05/03/17 Вск 02:36:15 #931 №12571 
>>12570
У Зенона не в континууме дело. Там рациональных чисел хватает с избытком.
Аноним 05/03/17 Вск 03:14:51 #932 №12573 
мешок.png
>>12571
Именно в континууме.

Вместо кучек давай рассматривать капсулы c элементами.

Вообрази себе автомат со сквозной трубой, к которому привязан мешок. Когда сквозь трубу такого автомата пролетает капсула с элементами, автомат выдергивает один элемент из капсулы и кладет в мешок.

Затем вообрази единичный отрезок. К каждой точке отрезка специальной ниткой привязана капсула с элементами. Капсул, таким образом, континуум.

Мы начинаем смещать отрезок параллельно самому себе. При этом капсулы начинают пролетать сквозь автомат. Сместив отрезок достаточно далеко, мы можем добиться, чтобы все капсулы пролетели сквозь автомат. При этом в мешке автомата окажется по одному элементу из каждой капсулы.

Интуитивно?
Аноним 05/03/17 Вск 03:22:55 #933 №12575 
>>12573
Тут есть более простой пример. Представим, что каждое множество - лист бумаги. Сложим их стопочкой и проколем иглой. Игла в каждом множестве сделает дырку - т.е. выберет элемент.
Аноним 05/03/17 Вск 03:32:35 #934 №12576 
>>12575
Неявно подразумевается единообразие множеств, чуть ли не биекция между ними. Это не совсем то.
Аноним 05/03/17 Вск 06:10:41 #935 №12578 
>>12561
Я-то нет, а вот детям объяснить, что такое размерность попробуй, да так, чтобы не тупо зазубрили, а понимали почему именно так.
Аноним 05/03/17 Вск 06:20:52 #936 №12579 
>>12561
>куски в стакане умножить на стаканы даст обычные куски
И объяснить, почему не стаканы получатся. Хотя я долбоёб, а ты прав, там чёрным по-белому
>Сколько кусков сахара положили
Так что скорее всего и правда автор имел ввиду, то что имел ввиду, лол. Думал как-то оправдать это можно.
Аноним 05/03/17 Вск 10:13:23 #937 №12584 
>>12546
погуглил. конкретно эту не нашел - но подобные задачи решаются так, что единицы должны делиться на 6 и десятки должны делиться на 6 потом

тоесть ответ в задаче 72

12\6 и потом еще 60\6

ебанутая постановка вопроса.
Аноним 05/03/17 Вск 11:33:36 #938 №12587 
Помогите придумать две перестановки такие, что их композициями можно получить любую перестановку.
Аноним 05/03/17 Вск 11:41:57 #939 №12588 
>>12576
Нет.
Аноним 05/03/17 Вск 13:12:25 #940 №12600 
flutebyguweiz-dar1pk8.jpg
бля хикканы хочу поступить на программиста ( сейчас заканчиваю свое ПТУ тоже учусь на программиста) Матан вообще не знаю. Летом уже заканчиваю и поступаю по вступительным экзаменам. Как вы думаете можно ли выучить матан за это время ? Экзамены по сути как ЕГЭ.
Аноним 05/03/17 Вск 13:42:13 #941 №12604 
>>12588
Ленина Пакет.
Аноним 05/03/17 Вск 13:44:45 #942 №12605 
>>12600
Можно конечно, если заниматься хотя бы по час-два в день.
Аноним 05/03/17 Вск 13:48:31 #943 №12606 
>>12587
Какие перестановки специального вида ты знаешь?
Аноним 05/03/17 Вск 13:53:01 #944 №12607 
>>12606
Все, кроме цикла
Аноним 05/03/17 Вск 13:54:38 #945 №12608 
>>12607
Циклы знаешь, уже хорошо. А что такое чётность перестановки, и как она определяется?
Аноним 05/03/17 Вск 14:34:54 #946 №12613 
>>12608
Цикл как раз не совсем понял.
Четноость по инверсиям
Аноним 05/03/17 Вск 15:36:56 #947 №12614 
>>12613
Что непонятного в цикле?
(12345) =
1 -> 2
2 -> 3
3 -> 4
5 -> 1
Про твой вопрос - это цикл (1234...n) и любая 1 транспозиция.
Докажи:
1) Любую перестановку можно представить как произведение циклов
2) Любой цикл можно представить как произведение транспозиций
3) Любой цикл можно представить как произведение циклов (1234...n) и транспозиции[(12) как пример]
К прмеру (321) = (12)(123)(123)
Аноним 05/03/17 Вск 15:41:47 #948 №12615 
>>12614
>любая 1 транспозиция
>как произведение циклов (1234...n) и транспозиции

Может ты не так поймешь, поэтому допишу, что кол-во транспозиций в композиции тоже может быть сколько угодно, просто она одна единственная.
Аноним 05/03/17 Вск 15:49:25 #949 №12617 
>>12614
>3) Любой цикл можно
Ещё поправочка, не цикл, а перестановку.
Аноним 05/03/17 Вск 17:15:00 #950 №12618 
blob
Дорогие, есть такая шляпа, как разрулить и выйти на ответ?
Аноним 05/03/17 Вск 17:55:54 #951 №12621 
>>12618
t=1, t<0
Аноним 05/03/17 Вск 18:01:03 #952 №12622 
>>12621
при t>0 единственность корня можно доказать?
Аноним 05/03/17 Вск 18:02:43 #953 №12623 
>>12622
Не знаю, я в Wolfram Alpha решал.
Аноним 05/03/17 Вск 18:08:51 #954 №12624 
math.PNG
Аноним 05/03/17 Вск 18:10:45 #955 №12625 
>>12624
Они разочаровываются и уходят учить математику. Выучив что-то возвращаются, но уже чтобы посрать.
Аноним 05/03/17 Вск 18:13:53 #956 №12626 
>>12624
Ну мне вот надо решить нелинейное PDE, но толку тут что-то спрашивать. Вряд ли помогут же.
Аноним 05/03/17 Вск 18:14:11 #957 №12627 
>>12625
Я просто захожу время от времени и читаю последние посты; вечно школо-первокурс задачи, вот про перестановки хоть спросили, я аж обрадовался, ответил, хотя это тоже 1лвл, но всё же хоть какое-то разнообразие в веренице уравнений для 5 класса.
Аноним 05/03/17 Вск 18:32:59 #958 №12628 
Как сделать, дабы не надо было ручки и блокнота, а математика витала прямо возле тебя и ты не видел бы реальный мир?
Аноним 05/03/17 Вск 18:37:34 #959 №12629 
12.png
>>10508 (OP)
Возможно решить? Как вообще можно понять синус бесконечности?
Аноним 05/03/17 Вск 18:42:08 #960 №12630 
>>12622
Продифференцировать.
Аноним 05/03/17 Вск 19:25:49 #961 №12631 
>>12629
Никак его не нужно понимать. Вычислить предел - не значит подставить x = 0. От синуса в числителе тебе нужна только его ограниченность.
Аноним 05/03/17 Вск 19:33:48 #962 №12632 
>>12631
Ну ладно, подставил я ноль, какая неопределенность в конце будет? При поставлении нуля получаю (0 * sin(inf))/0 . Вопрос что будет вместо sin(inf), просто inf или ноль?
Аноним 05/03/17 Вск 19:38:16 #963 №12633 
>>12632
Такого sin(inf) предела, насколько мне известно, не существует. Преобразуй выражение каким-либо образом, чтобы убрать эту нехорошую неопределённую величину.
Аноним 05/03/17 Вск 19:41:53 #964 №12634 
>>12632
Ты читать умеешь? Не надо ничего подставлять. У тебя синус ограничен, это всё что тебе требуется. В знаменателе легко получить первый замечательный, и останется только применить одну из теорем о бесконечно малых.
Аноним 05/03/17 Вск 19:44:07 #965 №12635 
12.png
>>12634
У меня задание доказать, что правило Лопиталя тут неприменимо, а как мне доказать, если я не могу найти неопределенность? Или в этом и есть доказательство?
Аноним 05/03/17 Вск 19:50:27 #966 №12636 
>>12635
А, вот оно что. Нет, с неопределённостью здесь всё в порядке. Числитель стремится к нулю, знаменатель тоже. Правило Лопиталя звучит так - ЕСЛИ СУЩЕСТВУЕТ предел отношения производных, то он равен исходному пределу. Тебе нужно продифференцировать числитель и знаменатель, после чего доказать, что предел не существует.
Аноним 05/03/17 Вск 20:03:35 #967 №12637 
>>12636
Аноним 05/03/17 Вск 20:05:22 #968 №12638 
12.png
>>12636
Нашел я производные, но что это дало-то? Ничего не получил, теперь при подставлении нуля имею sin(inf) и cos(inf) ааааа
Аноним 05/03/17 Вск 20:05:43 #969 №12639 
12.jpg
>>12638
>>12637
Аноним 05/03/17 Вск 20:14:49 #970 №12641 
>>12639
Да не надо ничего подставлять, хоспадя. Ты теорию вообще знаешь хоть немного? Определение предела там, Коши, Гейне, вся хуйня? Чтобы доказать, что lim f(x) при x -> 0 не существует, ебе нужно найти две последовательности xn и yn, стремящиеся к нулю, такие, что f(xn) и f(yn) будут стремиться к разным числам. Это и будет означать, что предела не существует.
Аноним 05/03/17 Вск 20:16:44 #971 №12642 
14887335434920.jpg
>>12639
Алсо, этот переход ты вообще откуда взял?
Аноним 05/03/17 Вск 20:21:08 #972 №12643 
>>12642
Ну по первому замечательному sin a(x)/a(x) = 1
И остается предел(x=>0) x^2
Или так нельзя?
Аноним 05/03/17 Вск 20:23:31 #973 №12644 
>>12643
Это работает, только если a(x) стремится к нулю. Кстати, чему оно у тебя равно? под синусом 1/x, но делишь ты не на 1/x, а на x.
Аноним 05/03/17 Вск 20:28:28 #974 №12645 
>>12644
Да пиздец, зачем я вообще полез в задания с тремя звездочками? Хуй с ним, пойду спать, спасибо за хоть какие то объяснения
Аноним 05/03/17 Вск 20:30:45 #975 №12646 
>>12645
Но ведь они оба решаются в уме за несколько секунд.
Аноним 05/03/17 Вск 20:32:34 #976 №12647 
>>12645
Тебе нужно почитать теорию. Вот просто без говна сесть и почитать. Иначе так и будешь впадать в прострацию, стоит заданию оказаться хоть чуть-чуть нестандартным. Семестр только начался, так что времени для этого навалом.
Аноним 05/03/17 Вск 20:34:15 #977 №12649 
12.png
>>12646
Возможно я просто дебил, который решает по алгоритму, предыдущие задание я прошел на ура, ты прав, нужно перечитать теорию.
Аноним 05/03/17 Вск 20:41:54 #978 №12650 
>>12649
Ты же уже сам почти написал решение осталось понять почему
второго предела >>12639 не существует и исправить в конце x^2 на х.
Аноним 05/03/17 Вск 21:17:22 #979 №12651 
>>12638
Первого вроде нету а второй равен единице.
Аноним 05/03/17 Вск 21:23:32 #980 №12652 
>>12638
Синус икс в пределе равен иксу, по первому замечательному пределу, синус один делить на икс не определен но находится между 1 и -1, икс квадрат обладает наибольшей малостью, значит предел равен нулю.

Во втором синус икс при бесконечности опять не определен, но находится между 1 и -1. Значит разница между числителем и знаменателем будет максимум 2. А так как икс бесконечен, этот предел равен одному.

Правильно, нет?
Аноним 05/03/17 Вск 22:20:16 #981 №12654 
Сап матемач. В одном математическом тексте встретилось словосочетание "существенное условие". Я понимаю, необходимое там или достаточное, но "существенное" - это как?
Аноним 05/03/17 Вск 22:25:47 #982 №12655 
>>12654
Давай контекст.
Аноним 05/03/17 Вск 22:30:08 #983 №12657 
>>12655
Условие монотонности в признаке Дирихле существенно
Аноним 05/03/17 Вск 22:33:25 #984 №12658 
>>12657
Это значит, что без этого условия теорема не работает. Можно привести пример, когда будут выполнены все условия теоремы, кроме этого, но ряд сходиться не будет.
Аноним 05/03/17 Вск 23:38:33 #985 №12660 
>>12658
Необходимость vs достаточность???
Аноним 05/03/17 Вск 23:46:05 #986 №12661 
>>12660
М? Это условие не является ни необходимым, ни достаточным. Оно является существенным в данной теореме.
Аноним 05/03/17 Вск 23:52:25 #987 №12662 
Как увеличить силу мат. абстракции ? Удариться в эйдетизм ? Не могу удержать долго образ какой-либо фигуры из банальной евклидовой геометрии. В какой-то момент и как услышанный шлягер или музыка - заедает. Постоянно нужно воспроизводить фигуру/ы на что уходит много времени.
Аноним 06/03/17 Пнд 00:49:06 #988 №12664 
>>12662
Представить треугольник не можешь или что?
Аноним 06/03/17 Пнд 00:51:07 #989 №12665 
Ананасы, подскажите, пожалуйста, какие книги/лекции почитать по теории устойчивости.
Аноним 06/03/17 Пнд 01:33:14 #990 №12667 
>>12624
Жизненно.

>>12626
Так создай модерируемый тред для нормальных пацанов, глядишь и подтянется кто.
Аноним 06/03/17 Пнд 13:18:50 #991 №12691 
сетка Багуа - копия.jpg
>>10508 (OP)
Почему производная неявной функции записывается так, отнимая и прибавляя дельта икс?
Аноним 06/03/17 Пнд 13:45:02 #992 №12693 
>>12664
С представленим все нормально, удержать не могу. Вообщем как шум накладывается на музыку со старых пластинок. И еще миллион всяких мыслей. Как пообороть этот хаос и с концентрироваться что ли ?
Аноним 06/03/17 Пнд 14:18:07 #993 №12694 
>>12691
Чтобы через частные производные F ответ выразить. У тебя теперь в правой части первые два слагаемые - это приращение вдоль y, а вторые два - вдоль x.
Аноним 06/03/17 Пнд 14:47:55 #994 №12695 
>>12662
Нофапон сроком два месяца
Аноним 06/03/17 Пнд 15:18:55 #995 №12696 
сетка Багуа - копия.jpg
>>12694
Я имел ввиду вот это. Нахуя его писать, результат же не изменится.
Аноним 06/03/17 Пнд 15:45:11 #996 №12697 
>>12696
Разумеется, не изменился, иначе нельзя было бы написать равенство. Я тебе больше скажу, всё это равно нулю, потому что мы остаёмся на гиперповерхности, определяемой уравнением.
>У тебя теперь в правой части первые два слагаемые - это приращение вдоль y, а вторые два - вдоль x.
Вот, собственно, за этим всё и сделано. Второй слагаемое теперь отличается от первого одним аргументом, как и третье от четвёртого. Их и группируют попарно, чтобы потом их них получились частные производные. Что непонятно?
Аноним 06/03/17 Пнд 22:16:31 #997 №12698 
>>10508 (OP)
Помогите пожалуйста. Я такой тупой, что не могу это решить
Основанием пирамиды MABCD служит прямоугольник ABCD, AB=a. Ребро MB перпендикулярно плоскости основания, а грани MAD и MCD составляют с ним соответственно углы 30 и 60 градусов. Найдите объем пирамиды.
Аноним 06/03/17 Пнд 22:29:07 #998 №12699 
>>12698
Ну и в чём твоя проблема? Ты не знаешь формулу объёма пирамиды? Или что такое угол между двумя плоскостями?
Аноним 06/03/17 Пнд 22:41:12 #999 №12700 
P70306-224101.jpg
7 и 8, пожалуйста
Аноним 07/03/17 Втр 00:18:00 #1000 №12701 
14860403889160.jpg
>>12700
Хорошие задачи. И всё-таки, что именно вызывает затруднения? Ты не можешь найти оптимальный алгоритм? Или не можешь доказать его оптимальность? Ты наверняка обдумывал их до того, как сюда запостить, так поделись своими мыслями, чтобы было от чего отталкиваться.

Вообще, надо бы наверное добавить в шапку что-нибудь насчёт того, что здесь не решают твою домашку вместо тебя. А то уже заебали постить условие и ждать решения на блюдечке, как манны небесной.
Аноним 07/03/17 Втр 02:16:05 #1001 №12702 
>>12700
>>12701
Если у нас два котенка, то нам же со второго придется начинать и идти до последнего этажа, разве нет?
Аноним 07/03/17 Втр 02:25:21 #1002 №12703 
>>12702
Неужели нельзя придумать целому дополнительному котёнку применение получше, чем разок бросить его с первого этажа и забыть?
Аноним 07/03/17 Втр 02:32:21 #1003 №12704 
>>12703
Да вроде нет, второго же в любом случае придется с первого начинать бросать, нет? Ну первого можем не со второго (третьего на самом деле, да), а с 50, например, но в худшем случае разницы вроде и не будет особой. Но я ужасно сонный, так что могу и ошибаться.
Аноним 07/03/17 Втр 02:45:59 #1004 №12705 
>>12704
А если эти падлы живучие и умирают только при падении со 101 этажа?
Аноним 07/03/17 Втр 02:49:14 #1005 №12706 
>>12705
То мы их в конце концов сбросим со 101 этажа и узнаем об этом. Не понял вопроса.
Аноним 07/03/17 Втр 02:58:13 #1006 №12707 
>>12706
А можно сбросить одного с 101 этажа сразу, чтобы исключить предельный случай.
А дальше кидать 2го, начиная с 1го этажа. Количество бросков будет равно 1 + N, N - номер этажа, где пиздец котяткам которые больше срать не будут.
3мя котятками можно покидаться, используя метод золотого сечения до тех пор, пока не останется только один - так тебе станет известна быстрей нижняя граница. А потом последнего кидать последовательно увеличивая номер этажа, пока и он не сдохнет.
Аноним 07/03/17 Втр 03:04:23 #1007 №12708 
>>12707
Ну так я про это и сказал сразу же и в следующем посте, ты чего. Можно первого сбросить с третьего, тогда то же, что и у тебя будет. Лучше в среднем с 50. Не очень понял, что ты хотел этим сказать.
Аноним 07/03/17 Втр 03:14:18 #1008 №12709 
>>12707
Хотя, наверное, с 2мя котятками лучше поступить по-другому.
Бросать 1го котёнка, увеличивая номер этажа по пропорции золотого сечения. Так быстрее найдётся верхняя граница.
2, 4, 6, 9, 15, 24, 39, 62, 101
И уже узнав нужный интервал, начинать с нижней границы.
Аноним 07/03/17 Втр 03:35:30 #1009 №12710 
>>12709
Для 3 котят надо начинать либо с 39 либо с 62 этажа.
Допустим, мы решили начинать сбрасывать с 39 этажа. Берём 1го котёнка и топаем на 39 этаж. Если сдох - делим интервал золотым сечением интервал 0...39, и берём следующего котёнка. Если не сдох, топаем на 62 этаж. Снова бросаем. Если не сдох, идём на 101, а если сдох - делим интервал 39..62 и берем следующего котёнка.
Аноним 07/03/17 Втр 04:03:10 #1010 №12711 
>>12710
Почему золотым сечением-то, лол?
Аноним 07/03/17 Втр 07:37:30 #1011 №12712 
>>12711
В большинстве случаев, он даёт быстрее результат, чем деление интервала пополам и проверка обеих половинок.
Аноним 07/03/17 Втр 12:22:00 #1012 №12715 
>>12712
Пруф? Разборчивого Гусейн-Заде не предлагать.
Аноним 07/03/17 Втр 13:36:55 #1013 №12716 
сетка Багуа - копия.jpg
Как эту хуйню решить? Распишите полностью, а то я не понимаю с какой стороны подступится.
Аноним 07/03/17 Втр 13:51:16 #1014 №12717 
>>12712
В этой задаче, делить в какой-то фиксированной пропорции - заведомый рак. Попробуй нарисовать дерево вариантов, хотя бы для простого случае. Ну, там, для 20 этажей, например.
>>12716
>для этого можно воспользоваться разбиением интервала от одного на m равных частей и сосчитать сумму
Тебе прямым текстом расписали, что нужно делать. Делай как написано. Если проблема в том, чтобы выбрать m, то бери m = 10. Больше всё равно никто не заставит вручную считать.
Аноним 07/03/17 Втр 14:42:49 #1015 №12721 
>>10508 (OP)
Посоветуйте что то про матрицы, чтобы там с объяснениями было какой смысл у умножения матриц друг на друга и почему нужно умножать и складывать ихние столбики для этого.
Аноним 07/03/17 Втр 14:45:46 #1016 №12722 
сетка Багуа - копия.jpg
>>12717
Мне предлагают воспользоваться "очевидным" свойством пикрелейтед для решения дальше по тексту.
Аноним 07/03/17 Втр 15:04:57 #1017 №12723 
>>12722
А, ну тогда можно и m = 1000 ебануть, действительно. Вот у тебя получается сумма в духе (1/1000)(1^2+1,001^2+1,002^2+...+1,999^2). Как для её вычисления воспользоваться приведённой формулой? Это ведь почти она и есть, нужно только причесать немного.
>>12721
Подойдёт любой учебник по линейной алгебре. Например, Кострикин (смотри список литературы в шапке). Если вкратце, то матрица - это линейный оператор, и произведение матриц соответствует композиции операторов. Поэтому и такое странное на первый взгляд правило.
Аноним 07/03/17 Втр 15:44:22 #1018 №12724 
>>12723
Ну вот у меня и не получается её причесать.
Я понимаю что принцип для дробных квадратов такой же, но в правой части формулы стоит количество степеней/последняя взятая степень.
Сесть посчитать сумму первых пяти дробных квадратов 0.2квадрат+0.4квадрат... и посмотреть как подогнать оно к другому, чтобы совсем не отупеить что ли?
Аноним 07/03/17 Втр 15:57:42 #1019 №12725 
Хочу завести себе красивые многообразия с ушками, чтобы их гладить :3
Аноним 07/03/17 Втр 16:17:14 #1020 №12726 
gif.latex.gif
>>12724
Всё предельно очевидно. Мог бы и сам догадаться.
Дальше сам.
07/03/17 Втр 16:29:24 #1021 №12727 
>>12725
Демидович уже прорешал?
Аноним 07/03/17 Втр 16:30:52 #1022 №12729 
Так лампово сычевать одному дома и изучать математику.
Аноним 07/03/17 Втр 17:22:11 #1023 №12730 
>>12624
Про тпоехавших забыл. А так годно.
Аноним 07/03/17 Втр 17:24:47 #1024 №12731 
>>12700
8 что-то там 2 в степени 2015 или типа того, короче кодировать двоичным кодом, 7 соебседования из гугла, гугли треды про программирование за август-сентябрь этого года.
Аноним 07/03/17 Втр 17:27:23 #1025 №12732 
>>12731
>2 в степени 2015 или типа того
Заорал чёт.
Аноним 07/03/17 Втр 17:45:16 #1026 №12733 
image.png
cgsFtrocFCk.jpg
Ребзя, есть вопрос, даже скорее просьба. Подставляю на место A разные матрицы 3х3, но равенство не выполняется. Препод говорит что оно правильное и шлет нахуй пересчитывать. Причем самое странное то, что не сходится у всей группы. Можете перепроверить на любой из этих матриц. Короче help!
Аноним 07/03/17 Втр 17:46:51 #1027 №12735 
>>12732
Имеется 1000 бочек с вином и 10 мышей. Известно, что одна из этих бочек отравлена ядом. Яд начинает действовать через час и мышь умирает.

Как за час определить какая бочка отравлена?

Считается, что время, за которое мышь пьет вино мгновенно, в мышь может влезть бесконечное количество вина и вина в бочках тоже бесконечно =)

нумеруем бочки : 1- 1024 (нумеруем мышей, даем каждой мыши какой то разряд в 2ичной системе) - бочки записаны в мышах (мышь пьет из той бочки, в номере которой где она значит 1) через час умершие мыши покажут номер бочки.
Аноним 07/03/17 Втр 17:48:54 #1028 №12736 
>>12735
Эта задача проще той, что выше.
Аноним 07/03/17 Втр 17:50:36 #1029 №12737 
>>12736
Тоже самое, только цифры другие.

У вас есть 2 яйца…

… и у вас есть доступ на 100-этажное здание. Яйца могут быть либо очень крепкими, либо очень хрупкими, это значит, что они могут разбиться, скинутые с первого этажа, либо не разбиться даже скинутые с 100-го этажа. Оба яйца абсолютно идентичные. Вам нужно выяснить самый высокий этаж 100-этажного здания, с которого яйца могут быть скинуты и не разбиться. Вопрос в том, сколько попыток вам надо сделать. Вы можете разбить только два яйца.
Вот 8, типа собес какой-то гугла.
Аноним 07/03/17 Втр 17:54:00 #1030 №12738 
>>12737
Цифры другие, из-за них больше тонкостей учитывать. Да и вопрос совсем по-другому стоит.
Аноним 07/03/17 Втр 18:06:18 #1031 №12739 
Посоветуйте, пожалуйста, что-то типа Алгебры Шеня и Гельфанда, но по геометрии.
Аноним 07/03/17 Втр 18:10:34 #1032 №12740 
>>12738
Да всё равно принцип тот же, это ж олимпиадозадачи, они все пол один шаблон всегда по идее. 2048, 2^11, 5*2=10 и ещё где-то один спрятан, мб в "наименьшее". А, дней 2 макс, а мин 1.
Короче, поим всех рабов 1024+512+256+128+64=1984, ждём сутки, так как повезло(наименьшее количество), то отравленное в одном из 2015-1984=31 бочонке. Повторяем 16+8+4+2+1=31, так как повезло(наименьшее), то подохнет последний раб, потратим одного раба и одну бочку. Мб я правда неверно понял наименьшее.
Аноним 07/03/17 Втр 18:11:36 #1033 №12741 
>>12740
Ну можно даже последнего раба не тратить.
Аноним 07/03/17 Втр 18:20:46 #1034 №12742 
FXiXxCI.png
>>12740
Мдеее...

Так можно вообще взять одного раба и дать ему одну рандомную бочку попробовать. А потом хуяк - и повезло наткнуться сразу на нужную, заебись задача, очень содержательная. Хуйню пишешь. Речь идёт о гарантированном исходе.
Не говоря уж о том, что предложенное тобой никакого отношения к двоичной системе не имеет.
Аноним 07/03/17 Втр 18:24:51 #1035 №12743 
>>12742
Ну и ладно.
Аноним 07/03/17 Втр 19:21:15 #1036 №12748 
>>12733
Препод ёбнулся опечатался. AA=(AA)*.
Аноним 07/03/17 Втр 19:25:33 #1037 №12749 
изображение.png
>>12733
>>12748
Блджад
Аноним 07/03/17 Втр 22:32:35 #1038 №12753 
>>10508 (OP)
Ребят, помогите с траблами, великовозврастный даун, только только начавший понимать таблицы умножения и десяток. Как развить умножение и счет в уме? Метод зубрежки не предлагать - он для задротов Магов.
Наткнулся в интернетах на методику когда сначала понимаешь и учишь принцип умножения на 2, а потом тебе объясняют принцип умножения на 9. Оба способа основаны на том - что 92 это 9+9, а 9 3 это 9+9+9 тоесть зная сумму первых двух девяток и принцип сложения с 9 мы будем легко ориентироваться в таблице .
Звучить мудренно - но объясняет принцип таблицы.
Кто знает методичек по этой хреновине?
Аноним 07/03/17 Втр 22:35:24 #1039 №12754 
>>12753
Один хер, точные расчеты все равно на калькуляторах делаются. но иногда кое что надо в уме раскинуть и понять "принцип".
Аноним 07/03/17 Втр 22:44:49 #1040 №12755 
>>12753
>Метод зубрежки не предлагать - он для задротов Магов.
Для всех он. Умножение на 2, на 5, на 9, квадраты чисел - это всё интуитивное говно. Умножение на 3 - в голове прекрасно складываются три числа одновременно. Пара приёмов, когда отталкиваешься от удобных известных произведений, добавляя/вычитая интересующий множитель. Это закрывает до 3/4 всеё таблицы умножений
Аноним 08/03/17 Срд 00:23:24 #1041 №12757 
blob
>>10508 (OP)
Понадобилось вспомнить производные. Как из первого выражения мы приходим ко второму? Явно не просто домножаем на знаменатель, т.к. появился минус перед дробью.
Аноним 08/03/17 Срд 00:25:55 #1042 №12758 
>>12757
Правило дифференцирования сложной функции.
Аноним 08/03/17 Срд 05:05:01 #1043 №12759 
>>12757
(y^n)'=ny^(n-1)y'
Аноним 08/03/17 Срд 12:57:06 #1044 №12766 
>>12753
>принцип умножения на 2
>принцип умножения на 9
Лучше заведи себе общее понятие об умножении, а не ментальными фокусами занимайся
Аноним 08/03/17 Срд 13:48:31 #1045 №12768 
Как научиться решать задачи? Постоянно впадаю в фрустрацию, когда вижу школьные задачи о работниках, деталях и времени. Пытаюсь их решить не собственно, построением какой-то модели, а тупо перебором возможных действий над данными.
Аноним 08/03/17 Срд 13:52:59 #1046 №12769 
>>12726
Действительно. Как то сам не догадался.
Аноним 08/03/17 Срд 14:18:46 #1047 №12770 
>>12768
Либо ты не усвоил смысл арифметических операций, либо ты не обращаешь внимания на наименования единиц измерения, которые должны ограничивать полёт твоей фантазии при расчётах.
Аноним 08/03/17 Срд 17:38:06 #1048 №12772 
Почему учебники такие скучные? Почему нет учебников математики с розовыми понями?
Аноним 08/03/17 Срд 18:10:54 #1049 №12774 
>>12772
Потому что это наука. Серьёзная наука.
Аноним 08/03/17 Срд 18:20:15 #1050 №12776 
>>12772
Так ведь есть же.
Аноним 08/03/17 Срд 18:27:34 #1051 №12777 
>>12776
Можно ссылки
Аноним 08/03/17 Срд 18:48:58 #1052 №12778 
>>12777
Погугли про онеме-учебники, обсуждали тут уже как-то раз.
Аноним 09/03/17 Чтв 09:02:02 #1053 №12780 
P70309-085852.jpg
Помогите решить что-нибудь, пожалуйста
Аноним 09/03/17 Чтв 09:23:50 #1054 №12781 
>>12780
Пятое не решайте, помогите решить две с конца - уже хорошо.
Аноним 09/03/17 Чтв 09:25:45 #1055 №12782 
>>12781
Или четвертую
Аноним 09/03/17 Чтв 11:58:04 #1056 №12783 
>>12780
Со вторую по пятую решил. А дальше?
Аноним 09/03/17 Чтв 12:02:47 #1057 №12784 
Ребята
Аноним 09/03/17 Чтв 13:50:33 #1058 №12785 
Только первую! Пожалуйста!
comments powered by Disqus

Отзывы и предложения